Set Four: Vents, Shock, Respiratory Failure

Ace your homework & exams now with Quizwiz!

A client is admitted to the emergency department after a motorcycle accident. Upon assessment, the client's vital signs reveal blood pressure of 80/60 mm Hg and heart rate of 145 beats per minute. The client's skin is cool and clammy. Which medical order for this client will the nurse complete first?

100% oxygen via a nonrebreather mask

In MODS, which of the following events contribute to organ failure? A. Microvascular clotting B. Interstitial edema C. Exhaustion of fuel supply D. All of the above

D. All of the above

A patient visits a health clinic because of urticaria and shortness of breath after being stung by several wasps. The nurse practitioner immediately administers which medication to reduce bronchospasm?

Epinephrine

The nursing instructor is discussing shock with the senior nursing students. The instructor tells the students that shock is a life-threatening condition. What else should the instructor tell the students about shock?

It occurs when arterial blood flow and oxygen delivery to tissues and cells are inadequate. pg. 285

A large volume of intravenous fluids is being administered to an elderly client who experienced hypovolemic shock following diarrhea. The nurse is evaluating the client's response to treatment and notes the following as a sign of an adverse reaction:

Jugular venous distention pg. 298

Baroreceptors are a primary mechanism of blood pressure regulation which results from the initial stimulation of what type of receptors?

Neural

The nurse is evaluating a client in the intensive care unit to identify improvement in the client's condition. Which outcome does the nurse note as the result of inadequate compensatory mechanisms?

Organ damage

The nurse assesses a BP reading of 80/50 mm Hg from a patient in shock. What stage of shock does the nurse recognize the patient is in?

Progressive pg. 290

When a client is in the compensatory stage of shock, which symptom occurs?

Tachycardia

When a client is in the compensatory stage of shock, which symptom occurs?

Tachycardia pg. 288

What priority intervention can the nurse provide to decrease the incidence of septic shock for patients who are at risk?

Use strict hand hygiene techniques. pg. 301

Which type of shock occurs from an antigen-antibody response?

anaphylactic

Pts receiving fluid replacement should frequently be monitored for?

adequate urinary out put, changes in mental status, and vital sign stability

Which of the following colloids is expensive but rapidly expands plasma volume?

albumin

Hematologic system changes in progressive shock would be characterized by all of the following except? a) generalized hypoxemia b) hypertension c) metabolic acidosis d) sluggish blood flow

b) hypertension

A common vasoactive agent used to improve cardiac contractility is?

dopamine

Elevating the patient's legs slightly to improve cerebral circulation is contraindicated in which of the following disease processes?

head injury

A nurse knows that the major clinical use of dobutamine (Dobutrex) is to:

increase cardiac output.

The drug of choice for cardiac pain relief is IV ?

morphine

When a patient is in the compensatory stage of shock which of the following symptoms occurs?

tachycardia

The primary goal in treating cardiogenic shock is to?

treat oxygenation needs of the heart muscle

11. A patient with acute respiratory distress syndrome (ARDS) is placed in the prone position. When prone positioning is used, which information obtained by the nurse indicates that the positioning is effective? a. The patient's PaO2 is 89 mm Hg, and the SaO2 is 91%. b. Endotracheal suctioning results in clear mucous return. c. Sputum and blood cultures show no growth after 48 hours. d. The skin on the patient's back is intact and without redness.

* a

10. Which statement by the nurse when explaining the purpose of positive end-expiratory pressure (PEEP) to the family members of a patient with ARDS is accurate? a. "PEEP will push more air into the lungs during inhalation." b. "PEEP prevents the lung air sacs from collapsing during exhalation." c. "PEEP will prevent lung damage while the patient is on the ventilator." d. "PEEP allows the breathing machine to deliver 100% oxygen to the lungs."

*ANSWER B

8. A patient develops increasing dyspnea and hypoxemia 2 days after heart surgery. To determine whether the patient has acute respiratory distress syndrome (ARDS) or pulmonary edema caused by heart failure, the nurse will plan to assist with a. obtaining a ventilation-perfusion scan. b. drawing blood for arterial blood gases. c. insertion of a pulmonary artery catheter. d. positioning the patient for a chest x-ray.

*ANSWER C

What can the nurse include in the plan of care to ensure early intervention along the continuum of shock to improve the patient's prognosis? (Select all that apply.)

- Assess the patient who is at risk for shock. - Administer intravenous fluids. - Monitor for changes in vital signs.

Stress ulcers occur frequently in acutely ill patient. Which of the following medications would be used to prevent ulcer formation? Select all that apply.

- Famotidine (Pepcid) - Ranitidine (Zantac) - Lansoprazole (Prevacid)

The nurse is caring for a client diagnosed with shock. During report, the nurse reports the results of which assessments that signal early signs of the decompensation stage? Select all that apply.

- Vital signs - Skin color - Urine output - Peripheral pulses

The nurse is caring for a patient who is experiencing cardiogenic shock as a result of myocardial infarction. Which nursing assessment finding is most concerning? 1 PaO2 60 mm Hg 2 Blood pressure 100/56 mm Hg 3 Urine output 260 mL in eight hours 4 Heart rate 96 beats/minute

1 A PaO2 of 60 is below the normal 80 to 100 mm Hg. The patient experiencing cardiogenic shock will exhibit hypotension and tachycardia, and therefore a blood pressure of 100/56 mm Hg and heart rate of 96 would not apply. A urine output of 260 cc/8 hrs is borderline but not reportable without a continued trending pattern. Text Reference - p. 1633

The nurse is providing care to a patient in the intensive care unit (ICU) who is being coded due to cardiac arrest. Why would it be appropriate for the patient's family to be present in this situation? Select all that apply. 1 It aids in the grieving process if the patient dies. 2 It decreases the likelihood of malpractice lawsuits. 3 It may reduce the fear and anxiety the family is feeling. 4 It allows the patient's family to support their loved one. 5 It may help the family overcome doubts about the patient's condition.

1, 3, 4, 5 Allowing a patient's family to be present during resuscitative efforts during a cardiac arrest can help in the grieving process if the patient dies. It also reduces the fear and anxiety that the family is feeling, allows the patient's family to support their loved one, and helps the family overcome any doubts about the patient's condition. Allowing the family to be in the room during resuscitative efforts has not been shown to decrease the likelihood of malpractice lawsuits. Text Reference - p. 1602

The nurse identifies that pulmonary artery catheterization is contraindicated for patients with a history of what? Select all that apply. 1 Coagulopathy 2 Cardiogenic shock 3 Fulminant myocarditis 4 Endocardial pacemaker 5 Mechanical tricuspid valve

1, 4, 5 Pulmonary artery catheterization helps to monitor and manage the care of patients who are at high risk for hemodynamic compromise. Pulmonary artery catheterization may cause trauma in the blood vessels and worsen symptoms of coagulopathy. Pulmonary artery catheterization increases the risk of trauma in patients with mechanical tricuspid valves and endocardial pacemakers; therefore, it is contraindicated in the patient with coagulopathy, mechanical tricuspid valve, and transvenous pacemaker. Pulmonary artery catheterization is performed in patients with cardiogenic shock and fulminant myocarditis to detect the risk of heart failure. Text Reference - p. 1607

List 4 indications for mechanical ventilation.

1. Apnea or impending inability to breathe 2. Acute respiratory failure 3. Severe hypoxia 4. Respiratory muscle fatigue

When vasoactive medications are administered, the nurse must monitor vital signs at least how often?

15 min

When vasoactive medications are administered, the nurse must monitor vital signs at least how often?

15 minutes

When vasoactive medications are administered, the nurse must monitor vital signs at least how often?

15 minutes pg. 294

Hypovolemic shock occurs when intravascular volume decreases by?

15-30%

What data are used to calculate stroke volume (SV) for a patient with arterial pressure-based cardiac output (APCO) monitoring? 1 Height, BMR, age, gender 2 Gender, age, height, weight 3 BMI, BMR, blood pressure, heart rate 4 Age, gender, blood pressure, heart rate

2 Arterial pressure-based cardiac output (APCO) monitoring uses the arterial waveform characteristics along with patient demographic data including gender, age, height, and weight to calculate stroke volume (SV). Basal metabolic rate (BMR), body mass index (BMI), blood pressure, and heart rate are not used to calculate stroke volume with this measuring device. Heart rate is used to calculate continuous cardiac output (COO) and continuous cardiac index (CCI). Text Reference - p. 1607

What results in surfactant dysfunction during the injury phase of acute respiratory distress syndrome (ARDS)? 1 Decrease in gas exchange capability 2 Damage to alveolar type I and II cells 3 Engorgement of the peribronchial space 4 Ventilation to perfusion (V/Q) mismatch

2 Damage to alveolar type I and II cells During the injury phase of acute respiratory distress syndrome (ARDS), the alveolar type I and II cells (which produce surfactant) will be damaged. Along with accumulation of fluid and proteins, this cell damage results in surfactant dysfunction. The hyaline membranes that line the alveoli lead to the decrease in gas exchange capability. An engorgement of the peribronchial and perivascular interstitial space results in interstitial edema. Ventilation to perfusion (V/Q) mismatch results in hypoxemia.

Which medication helps reduce the risk of stress ulcers in a patient with acute respiratory failure? 1 Propofol 2 Sucralfate 3 Fentanyl 4 Vancomycin

2 Sucralfate A patient with acute respiratory illness has a high risk of bleeding from stress ulcers due to decreased mucus production. Therefore administration of mucosal protective agents like sucralfate will be beneficial. Propofol is a sedative and analgesic that helps to alleviate pain. It does not help treat stress ulcers. Fentanyl is an opioid used to decrease anxiety but has no effect on the gastrointestinal tract. Vancomycin is an antibiotic used to treat bacterial infections.

Which system of the body is often the first to show signs of dysfunction in systemic inflammatory response syndrome (SIRS) and multiple organ dysfunction syndrome (MODS)? 1 Neurologic system 2 Respiratory system 3 Cardiovascular system 4 Gastrointestinal system

2 Systemic inflammatory response syndrome (SIRS) and multiple organ dysfunction syndrome (MODS) occur due to a systemic inflammatory response. Inflammatory mediators have a direct effect on the pulmonary vasculature. Thus, the respiratory system is often the first system to show signs of dysfunction. Signs of nervous system dysfunction, such as mental changes, can be early signs of SIRS and MODS. However, the nervous system is not the first system to show signs of SIRS and MODS. When the respiratory system is affected, ventilation-perfusion mismatch becomes worse. Tissue oxygen demands increase, leading to cardiovascular changes. Hence, cardiovascular changes occur after changes in the respiratory system. In the early stages of SIRS and MODS, there is shunting away of blood from the gastrointestinal tract, making it vulnerable to ischemic injury. However, such changes show signs of dysfunction later than does the respiratory system. Text Reference - p. 1649

A patient with a spinal cord injury requires negative pressure ventilation. What statements should the nurse say to the patient's caregiver about negative pressure ventilation? Select all that apply. 1 "It is a type of invasive ventilation." 2 "It is similar to the normal ventilation." 3 "It pulls the chest inward during inspiration." 4 "It uses intermittent subatmospheric pressure." 5 "It uses chambers that encase the upper airway." 6 "It reduces intrathoracic pressure during inspiration."

2, 4, 6 Negative pressure ventilation is similar to normal ventilation in that expiration is passive and decreased intrathoracic pressures produce inspiration. Negative pressure ventilation uses intermittent subatmospheric pressure around the chest wall. This pressure reduces the intrathoracic pressure during inspiration. Negative pressure ventilation is noninvasive because it does not require an artificial airway. During inspiration, the chest is pulled outward because of the intermittent negative pressure around the chest wall. Negative pressure ventilation uses chambers that encase the chest wall, but not the upper airway. Text Reference - p. 1618

What are the primary pathophysiologic changes that occur in the injury or exudative phase of ARDS (select all that apply)? a. Atelectasis d. Hyaline membranes line the alveoli b. Shortness of breath e. Influx of neutrophils, monocytes, and lymphocytes c. Interstitial and alveolar edema

22. a, c, d. The injury or exudative phase is the early phase of ARDS when atelectasis and interstitial and alveoli edema occur and hyaline membranes composed of necrotic cells, protein, and fibrin line the alveoli. Together, these decrease gas exchange capability and lung compliance. Shortness of breath occurs but it is not a physiologic change. The increased inflammation and proliferation of fibroblasts occurs in the reparative or proliferative phase of ARDS, which occurs 1 to 2 weeks after the initial lung injury.

A patient experiences a myocardial infarction (MI). The nurse closely monitors the patient for complications and recognizes that hypotension is a warning sign of: 1 A secondary MI 2 Pulmonary edema 3 Cardiogenic shock 4 Fatal dysrhythmias

3 One of the initial cardinal signs of cardiogenic shock after a myocardial infarction (MI) is a slow, steady drop in blood pressure. Hypotension after an MI may be an indirect sign of a secondary MI or a fatal dysrhythmia. Depending on the origin of pulmonary edema, patients may experience hypotension or hypertension. Text Reference - p. 1633

Which interventions should the nurse perform before suctioning a patient who has an endotracheal (ET) tube using open-suction technique? Select all that apply. 1 Put on clean gloves. 2 Administer a bronchodilator. 3 Perform a cardiopulmonary assessment. 4 Hyperoxygenate the patient for 30 seconds. 5 Insert a few drops of normal saline into the ET to break up secretions.

3, 4 Suctioning is preceded by a thorough assessment and hyperoxygenation for 30 seconds. Sterile, not clean, gloves are necessary and it is not necessary to administer a bronchodilator. Instillation of normal saline into the ET tube is not an accepted standard practice. Text Reference - p. 1616

The nurse is administering oxygen therapy to a patient in septic shock. What are the possible factors that directly affect oxygen delivery in the patient and should be monitored? Select all that apply. 1 Urine output 2 White blood cells 3 Cardiac output 4 Available hemoglobin 5 Arterial oxygen saturation

3, 4, 5 Oxygen delivery depends on cardiac output, available hemoglobin, and arterial oxygen saturation (SaO2). The amount of blood that the heart pumps to the body may decide the amount of oxygen delivered to the tissues. Hemoglobin, the protein content in red blood cells, is responsible for carrying oxygen molecules. Low hemoglobin means a low oxygen supply to the tissues. Arterial oxygen saturation indicates the total oxygen carried by the blood in the arteries and implies the level of tissue oxygenation. The urine output and white blood cells are also important parameters to be monitored in a patient who suffers from a shock. However, these do not impact the oxygen delivery directly. Text Reference - p. 1641

The nurse is providing care to a patient in the intensive care unit (ICU). Which pharmacologic interventions are appropriate to induce and maintain sleep? Select all that apply. 1 Propofol 2 Fentanyl 3 Zolpidem 4 Morphine 5 Temazepam

3, 5 Medications that are appropriate to induce and maintain sleep for the patient in the intensive care unit (ICU) environment include zolpidem and tamazepam. Propofol is a medication that is used to induce sedation not sleep. Fentanyl and morphine are opioid medications used to treat pain and are not appropriate to induce and maintain sleep for this patient. Text Reference - p. 1601

The nurse taking care of a patient evidencing signs of shock empties the urinary catheter drainage bag after her 12-hour shift. The nurse notes an indicator of renal hypoperfusion. What is the relevant urinary output for this condition?

300 ml

A patient is receiving mechanical ventilation after having a stroke. The nurse determines that the ventilator settings are based on which patient status? 1 Ideal body weight, vital signs, and family preference 2 Ethics committee results, current physiologic state, and ideal body weight 3 Respiratory muscle strength, ethics committee results, and family preference 4 Arterial blood gases (ABGs), current physiologic state, and respiratory muscle strength

4 Arterial blood gases (ABGs), current physiologic state, and respiratory muscle strength Settings on mechanical ventilators are based on the patient's physiologic status, such as ABGs, ideal body weight, current physiologic state, level of consciousness, and respiratory strength. Ethics committee results and family preference are psychosocial in nature and are not criteria used to determine mechanical ventilation settings.

A patient is diagnosed with an exacerbation of chronic pulmonary disease. What is an appropriate nursing action? 1 Consult with the agency's ethics committee. 2 Warn the family about the patient's need for ventilator support. 3 Discuss how critically ill the patient will become without ventilator support. 4 Discuss mechanical ventilation with the patient, family, and healthcare providers.

4 Discuss mechanical ventilation with the patient, family, and healthcare providers. The nurse should encourage all patients with chronic illnesses to discuss the possibility of mechanical ventilation with their families and health care providers. The patient may or may not become critically ill without ventilator support. The decision to use, withhold, or withdraw mechanical ventilation will be made carefully, respecting the wishes of the patient and caregiver; however, if disagreements occur, the agency's ethics committee may be consulted for assistance.

What type of medication does the nurse anticipate being prescribed by the health care provider to manage confusion, disorientation, and delirium in a patient with systemic inflammatory response syndrome (SIRS) and multiple organ dysfunction syndrome (MODS)? 1 Vasopressors 2 Loop diuretics 3 Proton pump inhibitors 4 Calcium channel blockers

4 Impaired perfusion of the brain may cause confusion, disorientation, and delirium in the patient. The health care provider is likely to prescribe calcium channel blockers to a patient exhibiting confusion, disorientation, and delirium to reduce cerebral vasospasm and improve perfusion of the brain. Vasopressors may be prescribed to combat cardiovascular dysfunction. Loop diuretics are prescribed if there is renal dysfunction. Proton pump inhibitors are prescribed to manage gastrointestinal symptoms. Text Reference - p. 1651

A massive gastrointestinal bleed has resulted in hypovolemic shock in an older patient. What is a priority nursing diagnosis? 1 Acute pain 2 Impaired tissue integrity 3 Decreased cardiac output 4 Ineffective tissue perfusion

4 The many deleterious effects of shock are all related to inadequate perfusion and oxygenation of every body system. Ineffective tissue perfusion supersedes acute pain, impaired tissue integrity, and decreased cardiac output, because circulation is a priority. Acute pain may occur, but is not a priority at this time. Tissue integrity is not impaired. Text Reference - p. 1633

What is the goal in the care of a systemic inflammatory response syndrome (SIRS) patient whose bilirubin level is 3 mg/dL? 1 Patient will be free of stress ulcers. 2 Patient will not feel abdominal distension. 3 Patient will maintain intraabdominal pressures. 4 Patient will maintain adequate tissue perfusion.

4 The nurse will plan to maintain adequate tissue perfusion for a systemic inflammatory response syndrome (SIRS) patient with bilirubin level of 3 mg/dL. An elevation of the bilirubin indicates impaired liver function. Stress ulcer prophylaxis is routine and would have been initiated before SIRS occurred. Abdominal distention and increased intraabdominal pressures are associated with impaired liver function. Monitoring these conditions is essential but is of a lower priority than maintaining tissue perfusion. Text Reference - p. 1651

What laboratory finding correlates with a medical diagnosis of cardiogenic shock? 1 Decreased liver enzymes 2 Increased white blood cells 3 Decreased red blood cells, hemoglobin, and hematocrit 4 Increased blood urea nitrogen (BUN) and serum creatinine levels

4 The renal hypoperfusion that accompanies cardiogenic shock results in increased BUN and creatinine levels. Impaired perfusion of the liver results in increased liver enzymes, whereas white blood cell levels typically do not increase in cardiogenic shock. Red blood cell indices are typically normal because of relative hypovolemia. Test-Taking Tip: Bring to your test prep with a positive attitude about yourself, your nursing knowledge, and your test-taking abilities. A positive attitude is achieved through self-confidence gained by effective study. This means (a) answering questions (assessment), (b) organizing study time (planning), (c) reading and further study (implementation), and (d) answering questions (evaluation). Text Reference - p. 1633

The nurse reviews the plan of care for a patient with multisystem organ dysfunction syndrome. What is the most desirable outcome for the patient? 1 The patient will be free of signs and symptoms of sepsis 2 The patient will maintain a balanced fluid intake and output 3 The patient will experience enhanced overall well-being and mental rest 4 The patient will demonstrate improved perfusion and oxygenation of organs

4 The underlying pathophysiology of multisystem organ dysfunction syndrome (MODS) is a lack of perfusion to organs, resulting in tissue and/or organ hypoxia. Interventions to improve perfusion with fluids or medications improve patient outcomes. The outcomes listed in the other answer options are appropriate and desirable for the patient with MODS, but they are secondary to improved perfusion and oxygenation. Text Reference - p. 1647

The nurse suspects that a patient has a pulmonary embolus. Which clinical manifestation confirms the nurse's suspicion? 1 Tachycardia 2 Muscle weakness 3 Morning headache 4 Ventilation to perfusion (V/Q) mismatch

4 Ventilation to perfusion (V/Q) mismatch The nurse conducts a computerized scan or ventilation to perfusion V/Q lung scan to confirm pulmonary embolus. Tachycardia is an early sign of respiratory failure consequent to physiologic stress. In hypercapnic respiratory failure, muscle weakness or paralysis is caused by altered neuromuscular conditions. The patient experiences deep tendon reflexes, tremors, as well as seizures at a later stage. Hypercapnia also causes cerebral vasodilation, increased cerebral blood flow, and a mild increase in intracranial pressure that produces a headache.

Which indirect lung condition predisposes a patient to acute respiratory distress syndrome (ARDS)? 1 Sepsis 2 Aspiration 3 Pneumonia 4 Severe massive trauma

4 Severe massive trauma Severe massive trauma An indirect lung injury that predisposes a patient to ARDS is a severe massive trauma. Aspiration, pneumonia, and sepsis are all examples of direct lung injuries that can predispose a patient to ARDS.

Which drug is used to treat pulmonary infection? 1 Albuterol 2 Lorazepam 3 Nitroglycerin 4 Azithromycin

4. Azithromycin Pulmonary infections can exacerbate acute respiratory failure. An intravenous antibiotic such as azithromycin is administered to treat pulmonary infections. Albuterol is a short-acting bronchodilator that reverses the effects of bronchospasm. Lorazepam is used to decrease anxiety, agitation, and pain. Medications used in the treatment of pulmonary congestion infections include diuretics like nitroglycerin that are administered intravenously.

The nurse receives an order to administer a colloidal solution for a patient experiencing hypovolemic shock. What common colloidal solution will the nurse most likely administer?

5% albumin

Which blood pressure (BP) reading would result in a pulse pressure indicative of shock?

90/70 mm Hg

The nurse is caring for a client at risk for impending shock. The nurse is assessing the vital signs frequently. What systolic blood pressure (BP) value would indicate impending shock?

91 mm Hg

Physiological Integrity 8. To evaluate both oxygenation and ventilation in a patient with acute respiratory failure, the nurse uses the findings revealed with a. arterial blood gas (ABG) analysis. b. hemodynamic monitoring. c. chest x-rays. d. pulse oximetry.

A Rationale: ABG analysis is useful because it provides information about both oxygenation and ventilation and assists with determining possible etiologies and appropriate treatment. The other tests may also provide useful information about patient status but will not indicate whether the patient has hypoxemia, hypercapnia, or both. Cognitive Level: Comprehension Text Reference: p. 1805 Nursing Process: Assessment

A client is experiencing septic shock and infrequent bowel sounds. To ensure adequate nutrition, the nurse administers

A continuous infusion of total parenteral nutrition

1 The distal lumen port (catheter tip), labeled A in the image, is within the pulmonary artery. This port is used to monitor pulmonary artery (PA) pressures. Choice B is the port used for infusions. Choice C is the port used for injecting medications. Choice D is the port used to inflate the balloon. Text Reference - p. 1607

A patient has the following device. Which port should be used to measure pulmonary artery pressure? 1. A 2. B 3. C 4. D

3 PaCO2 is the best indicator of alveolar hyperventilation or hypoventilation. Continuous PETCO2 monitoring can assess the patency of the airway and the presence of breathing. Continuous oxygen saturation (SpO2) provides objective data regarding tissue oxygenation. Central venous pressure (CVP) or pulmonary artery (PA) catheters with ScvO2 or SvO2 capability provide an indirect indication of the patient's tissue oxygenation status. Text Reference - p. 1615

A patient is admitted to the ICU and is on assisted ventilation. Which is the best indicator of inadequate alveolar oxygenation? 1 PETCO2 2 SpO2 3 PaCO2 4 ScvO2 or SvO2

6 The patient's airway pressure during expiration is 6 cm H2O. Normally during exhalation, the airway pressure drops to zero and exhalation occurs passively. The pressure in CPAP is delivered continuously during spontaneous breathing to prevent the patient's airway pressure from falling to zero. Therefore, if CPAP is 6 cm H2O, airway pressure during expiration is 6 cm H2O. Text Reference - p. 1621

A patient on mechanical ventilation is receiving a continuous positive airway pressure (CPAP) of 6 cm H2O. What is the patient's airway pressure during expiration? Record your answer using a whole number. ___________________ cm H2O

4 Intrathoracic pressure changes associated with positive pressure ventilation cause a decrease in production of atrial natriuretic peptide. Positive pressure ventilation also decreases cardiac output, which further decreases renal perfusion. A decrease in renal perfusion increases the production of renin, angiotensin and aldosterone, which results in sodium retention. Text Reference - p. 1623

A patient on positive pressure ventilation has increased sodium retention in the body. A decrease in production of which biologic factor may have caused sodium retention in the patient? 1 Renin 2 Angiotensin 3 Aldosterone 4 Atrial natriuretic peptide

1 The high- and low-pressure alarms are set based on the patient's current status. Since the patient's lowest auscultated systolic blood pressure was 118 mm Hg, the best setting to use would be systolic 100. Because the patient's lowest diastolic blood pressure was 78, the best setting to use would be diastolic 60. The setting of systolic 120 and diastolic 80 may cause the low pressure alarm to go off frequently. The settings of systolic 140, diastolic 80 and systolic 150, diastolic 90 would not be appropriate for low-pressure settings. Text Reference - p. 1606

A patient with an arterial invasive device has the following auscultated blood pressures. What setting should the nurse use for this patient's low pressure alarms? 1 Systolic 100; Diastolic 60 2 Systolic 120; Diastolic 80 3 Systolic 140; Diastolic 80 4 Systolic 150; Diastolic 90

3 A β-adrenergic blocking medication decreases stroke volume. Contractility is reduced by negative inotropes. An example of a negative inotrope is a β-adrenergic blocker. Since an increase in contractility increases stroke volume (SV) and myocardial oxygen requirements, a negative inotrope such as a β-adrenergic blocker will decrease stroke volume (SV). A β-adrenergic blocking medication will decrease rather than increase oxygen use. This medication does not affect myocardial cellular metabolism. Text Reference - p. 1604

A patient with hypertension is prescribed a β-adrenergic blocking medication. What effect should the nurse expect this medication to have on the patient's heart? 1 Increased oxygen use 2 Increased stroke volume 3 Decreased stroke volume 4 Decreased cellular metabolism

1 After insertion and before using the PA catheter, a chest x-ray must be taken to confirm the catheter's position. A hemoglobin level is not needed before using the catheter for fluid administration. Electrolyte levels do not need to be evaluated before using the catheter for fluid administration. A 12-lead electrocardiogram is not needed before using the catheter for fluid administration. Text Reference - p. 1608

A pulmonary artery catheter has just been inserted through a patient's internal jugular vein. What should be done before the catheter is used for fluid administration? 1 Obtain a chest x-ray 2 Draw a hemoglobin level 3 Evaluate electrolyte levels 4 Obtain a 12-lead electrocardiogram

You are the nurse caring for a client in septic shock. You know to closely monitor your client. What finding would you observe when the client's condition is in its initial stages?

A rapid, bounding pulse

You are the nurse caring for a client in septic shock. You know to closely monitor your client. What finding would you observe when the client's condition is in its initial stages?

A rapid, bounding pulse pg. 302

Which problem is a pathophysiological consequence common to all shock states? A. Hypoperfusion B. Vasoconstriction C. Pulmonary edema D. Hypertension

A. Hypoperfusion

During the first 24 hours after a severe burn injury, which of the following physiologic responses typically occurs? A. Increased capillary permeability B. Inhibition of stress hormone release C. Increased cardiac contractility and cardiac output D. Increased peripheral vascular resistance

A. Increased capillary permeability

Which of the following laboratory alterations would indicate that a patient is developing renal failure in MODS? A. Increased serum creatinine B. Decreased lactate dehydrogenase (LDH) levels C. Decreased blood urea nitrogen (BUN) D. Hypokalemia

A. Increased serum creatinine

Clinical manifestations of hypovolemic shock include all of the following except: A. pulmonary edema. B. tachycardia. C. hypotension. D. oliguria.

A. pulmonary edema.

When assisting with oral intubation of a patient who is having respiratory distress, in which order will the nurse take these actions? (Put a comma and a space between each answer choice [A, B, C, D, E].) a. Obtain a portable chest-x-ray. b. Position the patient in the supine position. c. Inflate the cuff of the endotracheal tube after insertion. d. Attach an end-tidal CO2 detector to the endotracheal tube. e. Oxygenate the patient with a bag-valve-mask device for several minutes.

ANS: E, B, C, D, A The patient is pre-oxygenated with a bag-valve-mask system for 3 to 5 minutes before intubation and then placed in a supine position. After the intubation, the cuff on the endotracheal tube is inflated to occlude and protect the airway. Tube placement is assessed first with an end-tidal CO2 sensor and then with chest x-ray examination.

11. Norepinephrine (Levophed) has been prescribed for a patient who was admitted with dehydration and hypotension. Which patient information indicates that the nurse should consult with the health care provider before administration of the norepinephrine? a. The patient's central venous pressure is 3 mm Hg. b. The patient is receiving low dose dopamine (Intropin). c. The patient is in sinus tachycardia at 100 to 110 beats/min. d. The patient has had no urine output since being admitted.

ANS: A Adequate fluid administration is essential before administration of vasopressors to patients with hypovolemic shock. The patient's low central venous pressure indicates a need for more volume replacement. The other patient data are not contraindications to norepinephrine administration. DIF: Cognitive Level: Application REF: 1733-1735 | 1736 TOP: Nursing Process: Implementation MSC: NCLEX: Physiological Integrity

Norepinephrine (Levophed) has been prescribed for a patient who was admitted with dehydration and hypotension. Which patient information indicates that the nurse should consult with the health care provider before administration of the norepinephrine? a. The patient's central venous pressure is 3 mm Hg. b. The patient is receiving low dose dopamine (Intropin). c. The patient is in sinus tachycardia at 100 to 110 beats/min. d. The patient has had no urine output since being admitted.

ANS: A Adequate fluid administration is essential before administration of vasopressors to patients with hypovolemic shock. The patient's low central venous pressure indicates a need for more volume replacement. The other patient data are not contraindications to norepinephrine administration. DIF: Cognitive Level: Application REF: 1733-1735 | 1736 TOP: Nursing Process: Implementation MSC: NCLEX: Physiological Integrity

15. A patient with septic shock has a BP of 70/46 mm Hg, pulse of 136 beats/min, respirations of 32 breaths/min, temperature of 104°F, and blood glucose of 246 mg/dL. Which intervention ordered by the health care provider should the nurse implement first? a. Give normal saline IV at 500 mL/hr. b. Give acetaminophen (Tylenol) 650 mg rectally. c. Start insulin drip to maintain blood glucose at 110 to 150 mg/dL. d. Start norepinephrine to keep systolic blood pressure above 90 mm Hg.

ANS: A Because of the decreased preload associated with septic shock, fluid resuscitation is the initial therapy. The other actions also are appropriate, and should be initiated quickly as well.

15. A patient with septic shock has a BP of 70/46 mm Hg, pulse 136, respirations 32, temperature 104° F, and blood glucose 246 mg/dL. Which of these prescribed interventions will the nurse implement first? a. Give normal saline IV at 500 mL/hr. b. Infuse drotrecogin- (Xigris) 24 mcg/kg. c. Start insulin drip to maintain blood glucose at 110 to 150 mg/dL. d. Titrate norepinephrine (Levophed) to keep mean arterial pressure (MAP) at 65 to 70 mm Hg.

ANS: A Because of the low systemic vascular resistance (SVR) associated with septic shock, fluid resuscitation is the initial therapy. The other actions also are appropriate and should be initiated quickly as well. DIF: Cognitive Level: Application REF: 1735-1737 OBJ: Special Questions: Prioritization TOP: Nursing Process: Implementation MSC: NCLEX: Physiological Integrity

A patient with septic shock has a BP of 70/46 mm Hg, pulse 136, respirations 32, temperature 104° F, and blood glucose 246 mg/dL. Which of these prescribed interventions will the nurse implement first? a. Give normal saline IV at 500 mL/hr. b. Infuse drotrecogin- (Xigris) 24 mcg/kg. c. Start insulin drip to maintain blood glucose at 110 to 150 mg/dL. d. Titrate norepinephrine (Levophed) to keep mean arterial pressure (MAP) at 65 to 70 mm Hg.

ANS: A Because of the low systemic vascular resistance (SVR) associated with septic shock, fluid resuscitation is the initial therapy. The other actions also are appropriate and should be initiated quickly as well. DIF: Cognitive Level: Application REF: 1735-1737 OBJ: Special Questions: Prioritization TOP: Nursing Process: Implementation MSC: NCLEX: Physiological Integrity

35. A patient who is receiving positive pressure ventilation is scheduled for a spontaneous breathing trial (SBT). Which finding by the nurse is most likely to result in postponing the SBT? a. New ST segment elevation is noted on the cardiac monitor. b. Enteral feedings are being given through an orogastric tube. c. Scattered rhonchi are heard when auscultating breath sounds. d. hydromorphone (Dilaudid) is being used to treat postoperative pain.

ANS: A Myocardial ischemia is a contraindication for ventilator weaning. The ST segment elevation is an indication that weaning should be postponed until further investigation and/or treatment for myocardial ischemia can be done. Ventilator weaning can proceed when opioids are used for pain management, abnormal lung sounds are present, or enteral feedings are being used.

During change-of-shift report, the nurse is told that a patient has been admitted with dehydration and hypotension after having vomiting and diarrhea for 4 days. Which finding is most important for the nurse to report to the health care provider? a. New onset of confusion b. Heart rate 112 beats/minute c. Decreased bowel sounds d. Pale, cool, and dry extremities

ANS: A The changes in mental status are indicative that the patient is in the progressive stage of shock and that rapid intervention is needed to prevent further deterioration. The other information is consistent with compensatory shock

14. Which information obtained by the nurse when caring for a patient who has cardiogenic shock indicates that the patient may be developing multiple organ dysfunction syndrome (MODS)? a. The patient's serum creatinine level is elevated. b. The patient complains of intermittent chest pressure. c. The patient has crackles throughout both lung fields. d. The patient's extremities are cool and pulses are weak.

ANS: A The elevated serum creatinine level indicates that the patient has renal failure as well as heart failure. The crackles, chest pressure, and cool extremities are all consistent with the patient's diagnosis of cardiogenic shock. DIF: Cognitive Level: Application REF: 1740-1741 TOP: Nursing Process: Assessment MSC: NCLEX: Physiological Integrity

32. A patient who is orally intubated and receiving mechanical ventilation is anxious and is "fighting" the ventilator. Which action should the nurse take next? a. Verbally coach the patient to breathe with the ventilator. b. Sedate the patient with the ordered PRN lorazepam (Ativan). c. Manually ventilate the patient with a bag-valve-mask device. d. Increase the rate for the ordered propofol (Diprivan) infusion.

ANS: A The initial response by the nurse should be to try to decrease the patient's anxiety by coaching the patient about how to coordinate respirations with the ventilator. The other actions may also be helpful if the verbal coaching is ineffective in reducing the patient's anxiety.

1. A patient with neurogenic shock has just arrived in the emergency department after a diving accident. He has a cervical collar in place. Which of the following actions should the nurse take (select all that apply)? a. Prepare to administer atropine IV. b. Obtain baseline body temperature. c. Prepare for intubation and mechanical ventilation. d. Administer large volumes of lactated Ringer's solution. e. Administer high-flow oxygen (100%) by non-rebreather mask.

ANS: A, B, C, E All of the actions are appropriate except to give large volumes of lactated Ringer's solution. The patient with neurogenic shock usually has a normal blood volume, and it is important not to volume overload the patient. In addition, lactated Ringer's solution is used cautiously in all shock situations because the failing liver cannot convert lactate to bicarbonate. DIF: Cognitive Level: Application REF: 1736-1737 OBJ: Special Questions: Alternate Item Format TOP: Nursing Process: Implementation MSC: NCLEX: Physiological Integrity

After change-of-shift report in the progressive care unit, who should the nurse care for first? a. Patient who had an inferior myocardial infarction 2 days ago and has crackles in the lung bases b. Patient with suspected urosepsis who has new orders for urine and blood cultures and antibiotics c. Patient who had a T5 spinal cord injury 1 week ago and currently has a heart rate of 54 beats/minute d. Patient admitted with anaphylaxis 3 hours ago who now has clear lung sounds and a blood pressure of 108/58 mm Hg

ANS: B Antibiotics should be administered within the first hour for patients who have sepsis or suspected sepsis in order to prevent progression to systemic inflammatory response syndrome (SIRS) and septic shock. The data on the other patients indicate that they are more stable. Crackles heard only at the lung bases do not require immediate intervention in a patient who has had a myocardial infarction. Mild bradycardia does not usually require atropine in patients who have a spinal cord injury. The findings for the patient admitted with anaphylaxis indicate resolution of bronchospasm and hypotension.

9. Which of these findings is the best indicator that the fluid resuscitation for a patient with hypovolemic shock has been successful? a. Hemoglobin is within normal limits. b. Urine output is 60 mL over the last hour. c. Pulmonary artery wedge pressure (PAWP) is normal. d. Mean arterial pressure (MAP) is 65 mm Hg.

ANS: B Assessment of end organ perfusion, such as an adequate urine output, is the best indicator that fluid resuscitation has been successful. The hemoglobin level, PAWP, and MAP are useful in determining the effects of fluid administration, but they are not as useful as data indicating good organ perfusion. DIF: Cognitive Level: Application REF: 1733-1735 TOP: Nursing Process: Evaluation MSC: NCLEX: Physiological Integrity

9. Which finding is the best indicator that the fluid resuscitation for a 90-kg patient with hypovolemic shock has been effective? a. Hemoglobin is within normal limits. b. Urine output is 65 mL over the past hour. c. Central venous pressure (CVP) is normal. d. Mean arterial pressure (MAP) is 72 mm Hg.

ANS: B Assessment of end organ perfusion, such as an adequate urine output, is the best indicator that fluid resuscitation has been successful. Urine output should be equal to or more than 0.5 mL/kg/hr. The hemoglobin level, CVP, and MAP are useful in determining the effects of fluid administration, but they are not as useful as data indicating good organ perfusion.

The nurse is caring for a patient who has septic shock. Which assessment finding is most important for the nurse to report to the health care provider? a. Blood pressure (BP) 92/56 mm Hg b. Skin cool and clammy c. Oxygen saturation 92% d. Heart rate 118 beats/minute

ANS: B Because patients in the early stage of septic shock have warm and dry skin, the patient's cool and clammy skin indicates that shock is progressing. The other information will also be reported, but does not indicate deterioration of the patient's status.

10. Which intervention will the nurse include in the plan of care for a patient who has cardiogenic shock? a. Check temperature every 2 hours. b. Monitor breath sounds frequently. c. Maintain patient in supine position. d. Assess skin for flushing and itching.

ANS: B Because pulmonary congestion and dyspnea are characteristics of cardiogenic shock, the nurse should assess the breath sounds frequently. The head of the bed is usually elevated to decrease dyspnea in patients with cardiogenic shock. Elevated temperature and flushing or itching of the skin are not typical of cardiogenic shock.

23. Which information about a patient who is receiving vasopressin (Pitressin) to treat septic shock is most important for the nurse to communicate to the heath care provider? a. The patient's heart rate is 108 beats/min. b. The patient is complaining of chest pain. c. The patient's peripheral pulses are weak. d. The patient's urine output is 15 mL/hr.

ANS: B Because vasopressin is a potent vasoconstrictor, it may decrease coronary artery perfusion. The other information is consistent with the patient's diagnosis and should be reported to the health care provider but does not indicate a need for a change in therapy. DIF: Cognitive Level: Application REF: 1735-1736 OBJ: Special Questions: Prioritization TOP: Nursing Process: Assessment MSC: NCLEX: Physiological Integrity

23. Which finding about a patient who is receiving vasopressin to treat septic shock indicates an immediate need for the nurse to report the finding to the health care provider? a. The patient's urine output is 18 mL/hr. b. The patient is complaining of chest pain. c. The patient's peripheral pulses are weak. d. The patient's heart rate is 110 beats/minute.

ANS: B Because vasopressin is a potent vasoconstrictor, it may decrease coronary artery perfusion. The other information is consistent with the patient's diagnosis, and should be reported to the health care provider but does not indicate an immediate need for a change in therapy.

16. To verify the correct placement of an oral endotracheal tube (ET) after insertion, the best initial action by the nurse is to a. obtain a portable chest x-ray. b. use an end-tidal CO2 monitor. c. auscultate for bilateral breath sounds. d. observe for symmetrical chest movement.

ANS: B End-tidal CO2 monitors are currently recommended for rapid verification of ET placement. Auscultation for bilateral breath sounds and checking chest expansion are also used, but they are not as accurate as end-tidal CO2 monitoring. A chest x-ray confirms the placement but is done after the tube is secured.

14. The nurse is caring for a patient who has an intraaortic balloon pump in place. Which action should be included in the plan of care? a. Avoid the use of anticoagulant medications. b. Measure the patient's urinary output every hour. c. Provide passive range of motion for all extremities. d. Position the patient supine with head flat at all times.

ANS: B Monitoring urine output will help determine whether the patient's cardiac output has improved and also help monitor for balloon displacement blocking the renal arteries. The head of the bed can be elevated up to 30 degrees. Heparin is used to prevent thrombus formation. Limited movement is allowed for the extremity with the balloon insertion site to prevent displacement of the balloon.

28. The nurse is caring for a patient who has an arterial catheter in the left radial artery for arterial pressure-based cardiac output (APCO) monitoring. Which information obtained by the nurse requires a report to the health care provider? a. The patient has a positive Allen test result. b. There is redness at the catheter insertion site. c. The mean arterial pressure (MAP) is 86 mm Hg. d. The dicrotic notch is visible in the arterial waveform.

ANS: B Redness at the catheter insertion site indicates possible infection. The Allen test is performed before arterial line insertion, and a positive test result indicates normal ulnar artery perfusion. A MAP of 86 mm Hg is normal, and the dicrotic notch is normally present on the arterial waveform.

29. When caring for a patient who has an arterial catheter in the left radial artery for arterial pressure-based cardiac output (APCO) monitoring, which information obtained by the nurse is most important to report to the health care provider? a. The patient has a positive Allen test. b. There is redness at the catheter insertion site. c. The mean arterial pressure (MAP) is 86 mm Hg. d. The dicrotic notch is visible in the arterial waveform.

ANS: B Redness at the catheter insertion site indicates possible infection. The Allen test is performed before arterial line insertion, and a positive test indicates normal ulnar artery perfusion. A MAP of 86 is normal and the dicrotic notch is normally present on the arterial waveform. DIF: Cognitive Level: Apply (application) REF: 1606 OBJ: Special Questions: Prioritization TOP: Nursing Process: Assessment MSC: NCLEX: Physiological Integrity

2. Which hemodynamic parameter best reflects the effectiveness of drugs that the nurse gives to reduce a patient's left ventricular afterload? a. Mean arterial pressure (MAP) b. Systemic vascular resistance (SVR) c. Pulmonary vascular resistance (PVR) d. Pulmonary artery wedge pressure (PAWP)

ANS: B SVR reflects the resistance to ventricular ejection, or afterload. The other parameters may be monitored but do not reflect afterload as directly.

2. Which hemodynamic parameter is most appropriate for the nurse to monitor to determine the effectiveness of medications given to a patient to reduce left ventricular afterload? a. Mean arterial pressure (MAP) b. Systemic vascular resistance (SVR) c. Pulmonary vascular resistance (PVR) d. Pulmonary artery wedge pressure (PAWP)

ANS: B Systemic vascular resistance reflects the resistance to ventricular ejection, or afterload. The other parameters will be monitored, but do not reflect afterload as directly.

7. A patient with cardiogenic shock has the following vital signs: BP 86/50, pulse 126, respirations 30. The PAWP is increased and cardiac output is low. The nurse will anticipate a. infusion of 5% human albumin. b. administration of furosemide (Lasix) IV. c. titration of an epinephrine (Adrenalin) drip. d. administration of hydrocortisone (SoluCortef).

ANS: B The PAWP indicates that the patient's preload is elevated and furosemide is indicated to reduce the preload and improve cardiac output. Epinephrine would further increase heart rate and myocardial oxygen demand. Normal saline infusion would increase the PAWP further. Hydrocortisone might be used for septic or anaphylactic shock. DIF: Cognitive Level: Application REF: 1735 | 1736 TOP: Nursing Process: Planning MSC: NCLEX: Physiological Integrity

20. A patient who has been involved in a motor vehicle crash arrives in the emergency department (ED) with cool, clammy skin; tachycardia; and hypotension. Which intervention ordered by the health care provider should the nurse implement first? a. Insert two large-bore IV catheters. b. Provide O2 at 100% per non-rebreather mask. c. Draw blood to type and crossmatch for transfusions. d. Initiate continuous electrocardiogram (ECG) monitoring.

ANS: B The first priority in the initial management of shock is maintenance of the airway and ventilation. ECG monitoring, insertion of IV catheters, and obtaining blood for transfusions should also be rapidly accomplished but only after actions to maximize O2 delivery have been implemented.

26. When evaluating a patient with a central venous catheter, the nurse observes that the insertion site is red and tender to touch and the patient's temperature is 101.8° F. What should the nurse plan to do next? a. Give analgesics and antibiotics as ordered. b. Discontinue the catheter and culture the tip. c. Change the flush system and monitor the site. d. Check the site more frequently for any swelling.

ANS: B The information indicates that the patient has a local and systemic infection caused by the catheter, and the catheter should be discontinued. Changing the flush system, giving analgesics, and continued monitoring will not help prevent or treat the infection. Administration of antibiotics is appropriate, but the line should still be discontinued to avoid further complications such as endocarditis. DIF: Cognitive Level: Apply (application) REF: 1611 TOP: Nursing Process: Planning MSC: NCLEX: Physiological Integrity

2. A nurse is caring for a patient whose hemodynamic monitoring indicates a blood pressure of 92/54 mm Hg, a pulse of 64 beats/min, and an elevated pulmonary artery wedge pressure (PAWP). Which intervention ordered by the health care provider should the nurse question? a. Elevate head of bed to 30 degrees. b. Infuse normal saline at 250 mL/hr. c. Hold nitroprusside if systolic BP is less than 90 mm Hg. d. Titrate dobutamine to keep systolic BP is greater than 90 mm Hg.

ANS: B The patient's elevated PAWP indicates volume excess in relation to cardiac pumping ability, consistent with cardiogenic shock. A saline infusion at 250 mL/hr will exacerbate the volume excess. The other actions will help to improve cardiac output, which should lower the PAWP and may raise the BP.

26. An 81-yr-old patient who has been in the intensive care unit (ICU) for a week is now stable and transfer to the progressive care unit is planned. On rounds, the nurse notices that the patient has new onset confusion. The nurse will plan to a. give PRN lorazepam (Ativan) and cancel the transfer. b. inform the receiving nurse and then transfer the patient. c. notify the health care provider and postpone the transfer. d. obtain an order for restraints as needed and transfer the patient.

ANS: B The patient's history and symptoms most likely indicate delirium associated with the sleep deprivation and sensory overload in the ICU environment. Informing the receiving nurse and transferring the patient is appropriate. Postponing the transfer is likely to prolong the delirium. Benzodiazepines and restraints contribute to delirium and agitation.

27. An 81-year-old patient who has been in the intensive care unit (ICU) for a week is now stable and transfer to the progressive care unit is planned. On rounds, the nurse notices that the patient has new onset confusion. The nurse will plan to a. give PRN lorazepam (Ativan) and cancel the transfer. b. inform the receiving nurse and then transfer the patient. c. notify the health care provider and postpone the transfer. d. obtain an order for restraints as needed and transfer the patient.

ANS: B The patient's history and symptoms most likely indicate delirium associated with the sleep deprivation and sensory overload in the ICU environment. Informing the receiving nurse and transferring the patient is appropriate. Postponing the transfer is likely to prolong the delirium. Benzodiazepines and restraints contribute to delirium and agitation. DIF: Cognitive Level: Apply (application) REF: 1601 TOP: Nursing Process: Planning MSC: NCLEX: Psychosocial Integrity

25. After reviewing the information shown in the accompanying figure for a patient with pneumonia and sepsis, which information is most important to report to the health care provider? Physical Assessment Laboratory Data Vital Signs · Petechiae noted on chest and legs · Crackles heard bilaterally in lung bases · No redness or swelling at central line IV site · Blood urea nitrogen (BUN) 34 mg/Dl · Hematocrit 30% · Platelets 50,000/ μL · Temperature 100°F (37.8°C) · Pulse 102/min · Respirations 26/min · BP 110/60 mm Hg · O2 saturation 93% on 2L O2 via nasal cannula a. Temperature and IV site appearance b. Oxygen saturation and breath sounds c. Platelet count and presence of petechiae d. Blood pressure, pulse rate, respiratory rate.

ANS: C The low platelet count and presence of petechiae suggest that the patient may have disseminated intravascular coagulation and that multiple organ dysfunction syndrome is developing. The other information will also be discussed with the health care provider but does not indicate that the patient's condition is deteriorating or that a change in therapy is needed immediately.

13. An intraaortic balloon pump (IABP) is being used for a patient who is in cardiogenic shock. Which assessment data indicate to the nurse that the goals of treatment with the IABP are being met? a. Urine output of 25 mL/hr b. Heart rate of 110 beats/minute c. Cardiac output (CO) of 5 L/min d. Stroke volume (SV) of 40 mL/beat

ANS: C A CO of 5 L/min is normal and indicates that the IABP has been successful in treating the shock. The low SV signifies continued cardiogenic shock. The tachycardia and low urine output also suggest continued cardiogenic shock.

The emergency department (ED) nurse receives report that a patient involved in a motor vehicle crash is being transported to the facility with an estimated arrival in 1 minute. In preparation for the patient's arrival, the nurse will obtain a. hypothermia blanket. b. lactated Ringer's solution. c. two 14-gauge IV catheters. d. dopamine (Intropin) infusion.

ANS: C A patient with multiple trauma may require fluid resuscitation to prevent or treat hypovolemic shock, so the nurse will anticipate the need for 2 large bore IV lines to administer normal saline. Lactated Ringer's solution should be used cautiously and will not be ordered until the patient has been assessed for possible liver abnormalities. Vasopressor infusion is not used as the initial therapy for hypovolemic shock. Patients in shock need to be kept warm not cool.

27. The family members of a patient who has been admitted to the intensive care unit (ICU) with multiple traumatic injuries have just arrived in the ICU waiting room. Which action should the nurse take first? a. Explain ICU visitation policies and encourage family visits. b. Escort the family from the waiting room to the patient's bedside. c. Describe the patient's injuries and the care that is being provided. d. Invite the family to participate in an interprofessional care conference.

ANS: C Lack of information is a major source of anxiety for family members and should be addressed first. Family members should be prepared for the patient's appearance and the ICU environment before visiting the patient for the first time. ICU visiting should be individualized to each patient and family rather than being dictated by rigid visitation policies. Inviting the family to participate in a multidisciplinary conference is appropriate but should not be the initial action by the nurse.

28. The family members of a patient who has just been admitted to the intensive care unit (ICU) with multiple traumatic injuries have just arrived in the ICU waiting room. Which action should the nurse take next? a. Explain ICU visitation policies and encourage family visits. b. Immediately take the family members to the patient's bedside. c. Describe the patient's injuries and the care that is being provided. d. Invite the family to participate in a multidisciplinary care conference.

ANS: C Lack of information is a major source of anxiety for family members and should be addressed first. Family members should be prepared for the patient's appearance and the ICU environment before visiting the patient for the first time. ICU visiting should be individualized to each patient and family rather than being dictated by rigid visitation policies. Inviting the family to participate in a multidisciplinary conference is appropriate but should not be the initial action by the nurse. DIF: Cognitive Level: Apply (application) REF: 1602 OBJ: Special Questions: Prioritization TOP: Nursing Process: Implementation MSC: NCLEX: Psychosocial Integrity

4. An older patient with cardiogenic shock is cool and clammy. Hemodynamic monitoring indicates a high systemic vascular resistance (SVR). Which intervention should the nurse anticipate? a. Increase the rate for the dopamine infusion. b. Decrease the rate for the nitroglycerin infusion. c. Increase the rate for the sodium nitroprusside infusion. d. Decrease the rate for the 5% dextrose in normal saline (D5/.9 NS) infusion.

ANS: C Nitroprusside is an arterial vasodilator and will decrease the SVR and afterload, which will improve cardiac output. Changes in the D5/.9 NS and nitroglycerin infusions will not directly decrease SVR. Increasing the dopamine will tend to increase SVR.

5. When caring for a patient with pulmonary hypertension, which parameter will the nurse use to directly evaluate the effectiveness of the treatment? a. Central venous pressure (CVP) b. Systemic vascular resistance (SVR) c. Pulmonary vascular resistance (PVR) d. Pulmonary artery wedge pressure (PAWP)

ANS: C PVR is a major contributor to pulmonary hypertension, and a decrease would indicate that pulmonary hypertension was improving. The other parameters may also be monitored but do not directly assess for pulmonary hypertension.

10. Which intervention will the nurse include in the plan of care for a patient who has cardiogenic shock? a. Avoid elevating head of bed. b. Check temperature every 2 hours. c. Monitor breath sounds frequently. d. Assess skin for flushing and itching.

ANS: C Since pulmonary congestion and dyspnea are characteristics of cardiogenic shock, the nurse should assess the breath sounds frequently. The head of the bed is usually elevated to decrease dyspnea. Elevated temperature and flushing or itching of the skin are not typical of cardiogenic shock. DIF: Cognitive Level: Application REF: 1721 TOP: Nursing Process: Implementation MSC: NCLEX: Physiological Integrity

Which intervention will the nurse include in the plan of care for a patient who has cardiogenic shock? a. Avoid elevating head of bed. b. Check temperature every 2 hours. c. Monitor breath sounds frequently. d. Assess skin for flushing and itching.

ANS: C Since pulmonary congestion and dyspnea are characteristics of cardiogenic shock, the nurse should assess the breath sounds frequently. The head of the bed is usually elevated to decrease dyspnea. Elevated temperature and flushing or itching of the skin are not typical of cardiogenic shock. DIF: Cognitive Level: Application REF: 1721 TOP: Nursing Process: Implementation MSC: NCLEX: Physiological Integrity

23. A nurse is weaning a 68-kg male patient who has chronic obstructive pulmonary disease (COPD) from mechanical ventilation. Which patient assessment finding indicates that the weaning protocol should be stopped? a. The patient's heart rate is 97 beats/min. b. The patient's oxygen saturation is 93%. c. The patient respiratory rate is 32 breaths/min. d. The patient's spontaneous tidal volume is 450 mL.

ANS: C Tachypnea is a sign that the patient's work of breathing is too high to allow weaning to proceed. The patient's heart rate is within normal limits, although the nurse should continue to monitor it. An oxygen saturation of 93% is acceptable for a patient with COPD. A spontaneous tidal volume of 450 mL is within the acceptable range. DIF: Cognitive Level: Apply (application) REF: 1627 TOP: Nursing Process: Evaluation MSC: NCLEX: Physiological Integrity

21. A patient who has neurogenic shock is receiving a phenylephrine infusion through a right forearm IV. Which assessment finding obtained by the nurse indicates a need for immediate action? a. The patient's heart rate is 58 beats/min. b. The patient's extremities are warm and dry. c. The patient's IV infusion site is cool and pale. d. The patient's urine output is 28 mL over the past hour.

ANS: C The coldness and pallor at the infusion site suggest extravasation of the phenylephrine. The nurse should discontinue the IV and, if possible, infuse the drug into a central line. An apical pulse of 58 beats/min is typical for neurogenic shock but does not indicate an immediate need for nursing intervention. A 28-mL urinary output over 1 hour would require the nurse to monitor the output over the next hour, but an immediate change in therapy is not indicated. Warm, dry skin is consistent with early neurogenic shock, but it does not indicate a need for a change in therapy or immediate action.

30. The nurse notes that a patient's endotracheal tube (ET), which was at the 22-cm mark, is now at the 25-cm mark, and the patient is anxious and restless. Which action should the nurse take next? a. Check the O2 saturation. b. Offer reassurance to the patient. c. Listen to the patient's breath sounds. d. Notify the patient's health care provider.

ANS: C The nurse should first determine whether the ET tube has been displaced into the right mainstem bronchus by listening for unilateral breath sounds. If so, assistance will be needed to reposition the tube immediately. The other actions are also appropriate, but detection and correction of tube malposition are the most critical actions.

5. After receiving 2 L of normal saline, the central venous pressure for a patient who has septic shock is 10 mm Hg, but the blood pressure is still 82/40 mm Hg. The nurse will anticipate an order for a. furosemide . c. norepinephrine . b. nitroglycerin . d. sodium nitroprusside .

ANS: C When fluid resuscitation is unsuccessful, vasopressor drugs are given to increase the systemic vascular resistance (SVR) and blood pressure and improve tissue perfusion. Furosemide would cause diuresis and further decrease the BP. Nitroglycerin would decrease the preload and further drop cardiac output and BP. Nitroprusside is an arterial vasodilator and would further decrease SVR.

15. While waiting for heart transplantation, a patient with severe cardiomyopathy has a ventricular assist device (VAD) implanted. When planning care for this patient, the nurse should anticipate a. preparing the patient for a permanent VAD. b. administering immunosuppressive medications. c. teaching the patient the reason for complete bed rest. d. monitoring the surgical incision for signs of infection.

ANS: D The insertion site for the VAD provides a source for transmission of infection to the circulatory system and requires frequent monitoring. Patients with VADs are able to have some mobility and may not be on bed rest. The VAD is a bridge to transplantation, not a permanent device. Immunosuppression is not necessary for nonbiologic devices such as the VAD.

8. The emergency department (ED) nurse receives report that a seriously injured patient involved in a motor vehicle crash is being transported to the facility with an estimated arrival in 5 minutes. In preparation for the patient's arrival, the nurse will obtain a. a dopamine infusion. c. lactated Ringer's solution. b. a hypothermia blanket. d. two 16-gauge IV catheters.

ANS: D A patient with multiple trauma may require fluid resuscitation to prevent or treat hypovolemic shock, so the nurse will anticipate the need for 2 large-bore IV lines to administer normal saline. Lactated Ringer's solution should be used cautiously and will not be ordered until the patient has been assessed for possible liver abnormalities. Vasopressor infusion is not used as the initial therapy for hypovolemic shock. Patients in shock need to be kept warm not cool.

13. Which assessment is most important for the nurse to make in order to evaluate whether treatment of a patient with anaphylactic shock has been effective? a. Pulse rate b. Orientation c. Blood pressure d. Oxygen saturation

ANS: D Because the airway edema that is associated with anaphylaxis can affect airway and breathing, the oxygen saturation is the most critical assessment. Improvements in the other assessments also will be expected with effective treatment of anaphylactic shock. DIF: Cognitive Level: Application REF: 1724-1725 | 1732 TOP: Nursing Process: Evaluation MSC: NCLEX: Physiological Integrity

Which assessment is most important for the nurse to make in order to evaluate whether treatment of a patient with anaphylactic shock has been effective? a. Pulse rate b. Orientation c. Blood pressure d. Oxygen saturation

ANS: D Because the airway edema that is associated with anaphylaxis can affect airway and breathing, the oxygen saturation is the most critical assessment. Improvements in the other assessments also will be expected with effective treatment of anaphylactic shock. DIF: Cognitive Level: Application REF: 1724-1725 | 1732 TOP: Nursing Process: Evaluation MSC: NCLEX: Physiological Integrity

13. Which assessment information is most important for the nurse to obtain when evaluating whether treatment of a patient with anaphylactic shock has been effective? a. Heart rate c. Blood pressure b. Orientation d. Oxygen saturation

ANS: D Because the airway edema that is associated with anaphylaxis can affect airway and breathing, the O2 saturation is the most critical assessment. Improvements in the other assessments will also be expected with effective treatment of anaphylactic shock.

Which assessment information is most important for the nurse to obtain to evaluate whether treatment of a patient with anaphylactic shock has been effective? a. Heart rate b. Orientation c. Blood pressure d. Oxygen saturation

ANS: D Because the airway edema that is associated with anaphylaxis can affect airway and breathing, the oxygen saturation is the most critical assessment. Improvements in the other assessments will also be expected with effective treatment of anaphylactic shock.

9. Which nursing action is needed when preparing to assist with the insertion of a pulmonary artery catheter? a. Determine if the cardiac troponin level is elevated. b. Auscultate heart sounds before and during insertion. c. Place the patient on NPO status before the procedure. d. Attach cardiac monitoring leads before the procedure.

ANS: D Dysrhythmias can occur as the catheter is floated through the right atrium and ventricle, and it is important for the nurse to monitor for these during insertion. Pulmonary artery catheter insertion does not require anesthesia, and the patient will not need to be NPO. Changes in cardiac troponin or heart and breath sounds are not expected during pulmonary artery catheter insertion.

9. Which action will the nurse need to do when preparing to assist with the insertion of a pulmonary artery catheter? a. Determine if the cardiac troponin level is elevated. b. Auscultate heart and breath sounds during insertion. c. Place the patient on NPO status before the procedure. d. Attach cardiac monitoring leads before the procedure.

ANS: D Dysrhythmias can occur as the catheter is floated through the right atrium and ventricle, and it is important for the nurse to monitor for these during insertion. Pulmonary artery catheter insertion does not require anesthesia, and the patient will not need to be NPO. Changes in cardiac troponin or heart and breath sounds are not expected during pulmonary artery catheter insertion. DIF: Cognitive Level: Apply (application) REF: 1608 TOP: Nursing Process: Planning MSC: NCLEX: Physiological Integrity

34. The nurse is caring for a patient with a subarachnoid hemorrhage who is intubated and placed on a mechanical ventilator with 10 cm H2O of peak end-expiratory pressure (PEEP). When monitoring the patient, the nurse will need to notify the health care provider immediately if the patient develops a. O2 saturation of 93%. b. green nasogastric tube drainage. c. respirations of 20 breaths/minute. d. increased jugular venous distention.

ANS: D Increases in jugular venous distention in a patient with a subarachnoid hemorrhage may indicate an increase in intracranial pressure (ICP) and that the PEEP setting is too high for this patient. A respiratory rate of 20, O2 saturation of 93%, and green nasogastric tube drainage are within normal limits.

4. A patient with cardiogenic shock is cool and clammy and hemodynamic monitoring indicates a high systemic vascular resistance (SVR). Which action will the nurse anticipate taking? a. Increase the rate for the prescribed dopamine (Intropin) infusion. b. Decrease the rate for the prescribed nitroglycerin (Tridil) infusion. c. Decrease the rate for the prescribed 5% dextrose in water (D5W) infusion. d. Increase the rate for the prescribed sodium nitroprusside (Nipride) infusion.

ANS: D Nitroprusside is an arterial vasodilator and will decrease the SVR and afterload, which will improve cardiac output. Changes in the D5W and nitroglycerin infusions will not directly increase SVR. Increasing the dopamine will tend to increase SVR. DIF: Cognitive Level: Application REF: 1733-1734 TOP: Nursing Process: Planning MSC: NCLEX: Physiological Integrity

When the nurse educator is evaluating the skills of a new registered nurse (RN) caring for patients experiencing shock, which action by the new RN indicates a need for more education? a. Placing the pulse oximeter on the ear for a patient with septic shock b. Keeping the head of the bed flat for a patient with hypovolemic shock c. Increasing the nitroprusside (Nipride) infusion rate for a patient with a high SVR d. Maintaining the room temperature at 66° to 68° F for a patient with neurogenic shock

ANS: D Patients with neurogenic shock may have poikilothermia. The room temperature should be kept warm to avoid hypothermia. The other actions by the new RN are appropriate.

During change-of-shift report, the nurse learns that a patient has been admitted with dehydration and hypotension after having vomiting and diarrhea for 3 days. Which finding is most important for the nurse to report to the health care provider? a. Decreased bowel sounds b. Apical pulse 110 beats/min c. Pale, cool, and dry extremities d. New onset of confusion and agitation

ANS: D The changes in mental status are indicative that the patient is in the progressive stage of shock and that rapid intervention is needed to prevent further deterioration. The other information is consistent with compensatory shock. DIF: Cognitive Level: Application REF: 1728-1729 OBJ: Special Questions: Prioritization TOP: Nursing Process: Assessment MSC: NCLEX: Physiological Integrity

31. The nurse educator is evaluating the care that a new registered nurse (RN) provides to a patient receiving mechanical ventilation. Which action by the new RN indicates the need for more education? a. The RN increases the FIO2 to 100% before suctioning. b. The RN secures a bite block in place using adhesive tape. c. The RN asks for assistance to resecure the endotracheal tube. d. The RN positions the patient with the head of bed at 10 degrees.

ANS: D The head of the patient's bed should be positioned at 30 to 45 degrees to prevent ventilator-associated pneumonia. The other actions by the new RN are appropriate.

21. Four hours after mechanical ventilation is initiated, a patient's arterial blood gas (ABG) results include a pH of 7.51, PaO2 of 82 mm Hg, PaCO2 of 26 mm Hg, and HCO3 - of 23 mEq/L (23 mmol/L). The nurse will anticipate the need to a. increase the FIO2 . c. increase the respiratory rate. b. increase the tidal volume. d. decrease the respiratory rate.

ANS: D The patient's PaCO2 and pH indicate respiratory alkalosis caused by too high a respiratory rate. The PaO2 is appropriate for a patient with COPD and increasing the respiratory rate and tidal volume would further lower the PaCO2 .

21. Four hours after mechanical ventilation is initiated for a patient with chronic obstructive pulmonary disease (COPD), the patient's arterial blood gas (ABG) results include a pH of 7.51, PaO2 of 82 mm Hg, PaCO2 of 26 mm Hg, and HCO3- of 23 mEq/L (23 mmol/L). The nurse will anticipate the need to a. increase the FIO2. b. increase the tidal volume. c. increase the respiratory rate. d. decrease the respiratory rate.

ANS: D The patient's PaCO2 and pH indicate respiratory alkalosis caused by too high a respiratory rate. The PaO2 is appropriate for a patient with COPD and increasing the respiratory rate and tidal volume would further lower the PaCO2. DIF: Cognitive Level: Analyze (analysis) REF: 1615-1616 TOP: Nursing Process: Planning MSC: NCLEX: Physiological Integrity

10. When assisting with the placement of a pulmonary artery (PA) catheter, the nurse notes that the catheter is correctly placed when the monitor shows a a. typical PA pressure waveform. b. tracing of the systemic arterial pressure. c. tracing of the systemic vascular resistance. d. typical PA wedge pressure (PAWP) tracing.

ANS: D The purpose of a PA line is to measure PAWP, so the catheter is floated through the pulmonary artery until the dilated balloon wedges in a distal branch of the pulmonary artery, and the PAWP readings are available. After insertion, the balloon is deflated and the PA waveform will be observed. Systemic arterial pressures are obtained using an arterial line and the systemic vascular resistance is a calculated value, not a waveform. DIF: Cognitive Level: Understand (comprehension) REF: 1608 TOP: Nursing Process: Assessment MSC: NCLEX: Physiological Integrity

3. A patient with respiratory failure has a respiratory rate of 6 breaths/minute and an oxygen saturation (SpO2) of 88%. The patient is increasingly lethargic. Which intervention will the nurse anticipate? * a. Administration of 100% oxygen by non-rebreather mask b. Endotracheal intubation and positive pressure ventilation c. Insertion of a mini-tracheostomy with frequent suctioning d. Initiation of continuous positive pressure ventilation (CPAP)

ANSWER B

7. A patient with acute respiratory distress syndrome (ARDS) and acute kidney injury has the following medications ordered. Which medication should the nurse discuss with the health care provider before giving? a. Pantoprazole (Protonix) 40 mg IV b. Gentamicin (Garamycin) 60 mg IV c. Sucralfate (Carafate) 1 g per nasogastric tube d. Methylprednisolone (Solu-Medrol) 60 mg IV

ANSWER B*

9. A nurse is caring for a patient with ARDS who is being treated with mechanical ventilation and high levels of positive end-expiratory pressure (PEEP). Which assessment finding by the nurse indicates that the PEEP may need to be reduced? a. The patient's PaO2 is 50 mm Hg and the SaO2 is 88%. b. The patient has subcutaneous emphysema on the upper thorax. c. The patient has bronchial breath sounds in both the lung fields. d. The patient has a first-degree atrioventricular heart block with a rate of 58.

ANSWER B*

A confused client exhibits a blood pressure of 112/84, pulse rate of 116 beats per minute, and respirations of 30 breaths per minute. The client's skin is cold and clammy. The nurse next

Administers oxygen by nasal cannula at 2 liters per minute

A confused client exhibits a blood pressure of 112/84, pulse rate of 116 beats per minute, and respirations of 30 breaths per minute. The client's skin is cold and clammy. The nurse next

Administers oxygen by nasal cannula at 2 liters per minute pg. 288

You are caring for a client in shock who is deteriorating. You are infusing IV fluids and giving medications as ordered. What type of medications are you most likely giving to this client?

Adrenergic

A client is admitted to the hospital with reports of chest pain. The nurse is monitoring the client and notifies the physician when the client exhibits

Adventitious breath sounds

Which colloid is expensive but rapidly expands plasma volume?

Albumin pg. 304

Which type of shock occurs from an antigen-antibody response?

Anaphylactic pg. 306

Which hematologic problem most significantly increases the risks associated with pulmonary artery (PA) catheter insertion? A. Leukocytosis B. Hypovolemia C. Hemolytic anemia D. Thrombocytopenia

Ans. D PA catheter insertion carries a significant risk of bleeding, a fact that is exacerbated when the patient has low levels of platelets. Leukocytosis, hypovolemia, and anemia are less likely to directly increase the risks associated with PA insertion

A gerontologic nurse educator is providing practice guidelines to unlicensed care providers. Because reaction to painful stimuli is sometimes blunted in older adults, what must be used with caution? A) Hot or cold packs B) Analgesics C) Anti-inflammatory medications D) Whirlpool baths

Ans: A Feedback: Reaction to painful stimuli may be decreased with age. Because pain is an important warning signal, caution must be used when hot or cold packs are used. The older patient may be burned or suffer frostbite before being aware of any discomfort. Any medication is used with caution in the elderly, but not because of the decreased sense of heat or cold. Whirlpool baths are generally not a routine treatment ordered for the elderly.

When teaching a client with newly diagnosed hypertension about the pathophysiology of this disease, the nurse states that arterial baroreceptors, which monitor arterial pressure, are located in the carotid sinus. Which other area should the nurse mention as a site of arterial baroreceptors?

Aorta pg. 286

A client admitted with a massive myocardial infarction rapidly develops cardiogenic shock. Ideally, the physician would use the intra-aortic balloon pump (IABP) to support the injured myocardium. However, this client has a history of unstable angina pectoris, aortic insufficiency, hypertension, and diabetes mellitus. Which condition is a contraindication for IABP use?

Aortic insufficiency

A client admitted with a massive myocardial infarction rapidly develops cardiogenic shock. Ideally, the physician would use the intra-aortic balloon pump (IABP) to support the injured myocardium. However, this client has a history of unstable angina pectoris, aortic insufficiency, hypertension, and diabetes mellitus. Which condition is a contraindication for IABP use?

Aortic insufficiency pg. 327

You are caring for a client with shock. You are concerned about hypoxemia and metabolic acidosis with your client. What finding should you analyze for evidence of hypoxemia and metabolic acidosis in a client with shock?

Arterial blood gas (ABG) findings

The nurse is caring for a client with shock. The nurse is concerned about hypoxemia and metabolic acidosis with the client. What finding should the nurse analyze for evidence of hypoxemia and metabolic acidosis in a client with shock?

Arterial blood gas (ABG) findings pg. 291

A patient scheduled for magnetic resonance imaging (MRI) has arrived at the radiology department. The nurse who prepares the patient for the MRI should prioritize which of the following actions? A) Withholding stimulants 24 to 48 hours prior to exam B) Removing all metal-containing objects C) Instructing the patient to void prior to the MRI D) Initiating an IV line for administration of contrast

B Patient preparation for an MRI consists of removing all metal-containing objects prior to the examination. Withholding stimulants would not affect an MRI; this relates to an electroencephalography (EEG). Instructing the patient to void is patient preparation for a lumbar puncture. Initiating an IV line for administration of contrast would be done if the patient was having a CT scan with contrast.

Physiological Integrity 9. A finding indicating to the nurse that a 22-year-old patient with respiratory distress is in acute respiratory failure includes a a. shallow breathing pattern. b. partial pressure of arterial oxygen (PaO2) of 45 mm Hg. c. partial pressure of carbon dioxide in arterial gas (PaCO2) of 34 mm Hg. d. respiratory rate of 32/min.

B Rationale: The PaO2 indicates severe hypoxemia and that the nurse should take immediate action to correct this problem. Shallow breathing, rapid respiratory rate, and low PaCO2 can be caused by other factors, such as anxiety or pain. Cognitive Level: Application Text Reference: p. 1806 Nursing Process: Assessment

Physiological Integrity 11. A patient with hypercapnic respiratory failure has a respiratory rate of 8 and an SpO2 of 89%. The patient is increasingly lethargic. Which collaborative intervention will the nurse anticipate? a. Administration of 100% oxygen by non-rebreather mask b. Endotracheal intubation and positive pressure ventilation c. Insertion of a mini-tracheostomy with frequent suctioning d. Initiation of bilevel positive pressure ventilation (BiPAP)

B Rationale: The patient's lethargy, low respiratory rate, and SpO2 indicate the need for mechanical ventilation with ventilator-controlled respiratory rate. Administration of high flow oxygen will not be helpful because the patient's respiratory rate is so low. Insertion of a mini-tracheostomy will facilitate removal of secretions, but it will not improve the patient's respiratory rate or oxygenation. BiPAP requires that the patient initiate an adequate respiratory rate to allow adequate gas exchange. Cognitive Level: Application Text Reference: pp. 1807-1808, 1810 Nursing Process: Planning

The nurse has admitted a new patient to the unit. One of the patient's admitting orders is for an adrenergic medication. The nurse knows that this medication will have what effect on the circulatory system? A) Thin, watery saliva B) Increased heart rate C) Decreased BP D) Constricted bronchioles

B The term "adrenergic" refers to the sympathetic nervous system. Sympathetic effects include an increased rate and force of the heartbeat. Cholinergic effects, which correspond to the parasympathetic division of the autonomic nervous system, include thin, watery saliva, decreased rate and force of heartbeat, and decreased BP.

What is the primary cause of hypotension in early stages of septic shock? A. Blood loss B. Arterial vasodilation C. Activation of the parasympathetic nervous system D. Heart failure

B. Arterial vasodilation

Anaphylactic shock manifests with the rapid onset of which set of symptoms? A. Bradycardia, decreased arterial pressure, and oliguria B. Dyspnea, hypotension, and urticaria C. Hypertension, anxiety, and tachycardia D. Fever, hypotension, and erythematous rash

B. Dyspnea, hypotension, and urticaria

Which of the following physiological alterations would you expect to see in the delayed response to a severe burn injury? A. Hypoglycemia B. Hypovolemia C. Hypometabolism D. Bleeding from wound beds

B. Hypovolemia

Which of the following shock states manifests with tachycardia, vasoconstriction, and movement of large volumes of interstitial fluid to the vascular compartment? A. Anaphylactic B. Hypovolemic C. Neurogenic D. Septic

B. Hypovolemic

Which of the following conditions is not a potential cause of cardiogenic shock? A. Tension pneumothorax B. Spinal cord injury C. Tamponade D. Cardiac arrhythmias

B. Spinal cord injury

Which burn injury is characterized by the immediate appearance of large water-filled blisters and a red wound bed? A. First degree B. Superficial partial thickness (second degree) C. Deep partial thickness (second degree) D. Third degree

B. Superficial partial thickness (second degree)

Anaphylactic shock occurs in response to severe: A. viral infections. B. allergic reactions. C. brain injuries. D. burn injuries.

B. allergic reactions.

The most common cause of multiple organ dysfunction syndrome (MODS) is: A. myocardial infarction. B. septic shock. C. chronic pulmonary disease. D. autoimmune disease.

B. septic shock.

During the compensation stage of shock, what is the consequence of the release of catecholamine sin the skeletal muscles?

Blood supply to the skeletal muscles increases.

During preshock, the compensatory stage of shock, the body, through sympathetic nervous system stimulation, will release catecholamines to shunt blood from one organ to another. Which of the following organs will always be protected?

Brain

During preshock, the compensatory stage of shock, the body, through sympathetic nervous system stimulation, will release catecholamines to shunt blood from one organ to another. Which of the following organs will always be protected?

Brain pg. 288

When beta-2 adrenergic receptors are stimulated which of the following occur?

Bronchioles relax

Physiological Integrity 17. All the following medications are ordered for a mechanically ventilated patient with acute respiratory distress syndrome (ARDS) and acute renal failure. Which medication should the nurse discuss with the health care provider before administration? a. IV ranitidine (Zantac) 50 mg IV b. sucralfate (Carafate) 1 g per nasogastric tube c. IV gentamicin (Garamycin) 60 mg d. IV methylprednisolone (Solu-Medrol) 40 mg

C Rationale: Gentamicin, which is one of the aminoglycoside antibiotics, is potentially nephrotoxic, and the nurse should clarify the drug and dosage with the health care provider before administration. The other medications are appropriate for the patient with ARDS. Cognitive Level: Application Text Reference: p. 1816 Nursing Process: Implementation

Individuals with severe burns are often at risk for becoming hypothermic. Which of the following descriptions best characterizes the underlying cause of this problem? A. Burn patients experience hypothermia as a rebound reaction from the heat damage. B. Burn patients tend to be hypothermic due to hypotension and ischemia. C. Evaporative heat loss from major burn wounds can lead to hypothermia. D. Blood coagulation limits the blood circulating to the body core, thereby causing hypothermia.

C. Evaporative heat loss from major burn wounds can lead to hypothermia.

Secondary MODS is defined as the progressive dysfunction of two or more organ systems resulting from: A. a drug overdose. B. severe hemorrhaging. C. an uncontrolled inflammatory response. D. myocardial depression.

C. an uncontrolled inflammatory response.

Causes of hypovolemic shock include all of the following except: A. dehydration. B. blood loss. C. brainstem injury. D. diuresis.

C. brainstem injury.

Arterial baroreceptors are located in the: A. renal artery. B. superior vena cava. C. carotid arteries. D. circle of Willis.

C. carotid arteries.

During hemorrhagic shock, the clinical manifestations of pale skin and cool extremities are most directly caused by: A. hypothermia. B. accumulation of toxic metabolites. C. vasoconstriction of peripheral arterioles. D. increased tissue oxygen demand.

C. vasoconstriction of peripheral arterioles.

A 74-year-old male client who is suffering a myocardial infarction is transported to the ED by ambulance. This client is at greatest risk for developing which type of shock?

Cardiogenic

Older adults are more likely to develop which type of shock?

Cardiogenic shock

The postanesthesia care unit (PACU) has several patients with endotracheal tubes. Which patient should receive the least amount of endotracheal suctioning? Transplantation of a kidney Replacement of aortic valve Cerebral aneurysm resection Formation of an ileal conduit

Cerebral aneurysm resection The nurse should avoid suctioning the patient after a craniotomy until it is necessary because suctioning will increase this patient's intracranial pressure. The patients with a kidney transplantation, aortic valve replacement, or formation of an ileal conduit will not be negatively affected by suctioning, although it should only be done when needed, not routinely.

You are a nurse in the Emergency Department (ED) caring for a client presenting with vasodilation. Your assessment indicates that the client's central blood flow is reduced and their peripheral vascular area is hypervolemic. You notify the physician that this client is in what kind of shock?

Circulatory (distributive)

You are caring for a client who is in neurogenic shock. You know that this is a subcategory of what kind of shock?

Circulatory (distributive)

You are caring for a client who is in neurogenic shock. You know that this is a subcategory of what kind of shock?

Circulatory (distributive) pg. 286

The nurse assesses a patient who experienced a reaction to a bee sting. The patient's clinical findings indicate a pre-shock condition, which is evidenced by:

Cold, clammy skin and tachycardia. pg. 288

The nurse obtains a blood pressure of 120/78 mm Hg from a patient in hypovolemic shock. Since the blood pressure is within normal range for this patient, what stage of shock does the nurse realize this patient is experiencing?

Compensatory stage

The nurse obtains a blood pressure of 120/78 mm Hg from a patient in hypovolemic shock. Since the blood pressure is within normal range for this patient, what stage of shock does the nurse realize this patient is experiencing?

Compensatory stage pg. 288

A vasoactive medication is prescribed for a patient in shock to help maintain MAP and hemodynamic stability. A medication that acts on the alpha-adrenergic receptors of the SNS is ordered. Its purpose is to:

Constrict blood vessels in the cardiorespiratory system.

A client who experienced shock remains unstable. Enteral nutritional supplements have been prescribed to prevent muscle wasting. The nurse

Consults with the physician about subsituting lansoprazole (Prevacid) for the prescribed dose of pantoprazole (Protonix)

15. The nurse evaluates that fluid resuscitation for a 70 kg patient in shock is effective on finding that the patient's a. urine output is 40 ml over the last hour. b. hemoglobin is within normal limits. c. CVP has decreased. d. mean arterial pressure (MAP) is 65 mm Hg.

Correct Answer: A Rationale: Assessment of end-organ perfusion, such as an adequate urine output, is the best indicator that fluid resuscitation has been successful. The hemoglobin level is not useful in determining whether fluid administration has been effective unless the patient is bleeding and receiving blood. A decrease in CVP indicates that more fluid is needed. The MAP is at the low normal range, but does not clearly indicate that tissue perfusion is adequate.

23. A patient in compensated septic shock has hemodynamic monitoring with a pulmonary artery catheter and an arterial catheter. Which information obtained by the nurse indicates that the patient is still in the compensatory stage of shock? a. The cardiac output is elevated. b. The central venous pressure (CVP) is increased. c. The systemic vascular resistance (SVR) is high. d. The PAWP is high.

Correct Answer: A Rationale: In the early stages of septic shock, the cardiac output is high. The other hemodynamic changes would indicate that the patient had developed progressive or refractory septic shock.

26. An assessment finding indicating to the nurse that a 70-kg patient in septic shock is progressing to MODS includes a. respiratory rate of 10 breaths/min. b. fixed urine specific gravity at 1.010. c. MAP of 55 mm Hg. d. 360-ml urine output in 8 hours.

Correct Answer: B Rationale: A fixed urine specific gravity points to an inability of the kidney to concentrate urine caused by acute tubular necrosis. With MODS, the patient's respiratory rate would initially increase. The MAP of 55 shows continued shock, but not necessarily progression to MODS. A 360-ml urine output over 8 hours indicates adequate renal perfusion.

22. A patient who has just been admitted with septic shock has a BP of 70/46, pulse 136, respirations 32, temperature 104.0° F, and blood glucose 246 mg/dl. Which order will the nurse accomplish first? a. Start insulin drip to maintain blood glucose at 110 to 150 mg/dl. b. Give normal saline IV at 500 ml/hr. c. Titrate norepinephrine (Levophed) to keep MAP at 65 to 70 mm Hg. d. Infuse drotrecogin- (Xigris) 24 mcg/kg.

Correct Answer: B Rationale: Because of the low systemic vascular resistance (SVR) associated with septic shock, fluid resuscitation is the initial therapy. The other actions are also appropriate and should be initiated quickly as well.

17. Norepinephrine (Levophed) has been ordered for the patient in hypovolemic shock. Before administering the drug, the nurse ensures that the a. patient's heart rate is less than 100. b. patient has received adequate fluid replacement. c. patient's urine output is within normal range. d. patient is not receiving other sympathomimetic drugs.

Correct Answer: B Rationale: If vasoconstrictors are given in a hypovolemic patient, the peripheral vasoconstriction will further decrease tissue perfusion. A patient with hypovolemia is likely to have a heart rate greater than 100 and a low urine output, so these values are not contraindications to vasoconstrictor therapy. Patients may receive other sympathomimetic drugs concurrently with Levophed.

8. The nurse caring for a patient in shock notifies the health care provider of the patient's deteriorating status when the patient's ABG results include a. pH 7.48, PaCO2 33 mm Hg. b. pH 7.33, PaCO2 30 mm Hg. c. pH 7.41, PaCO2 50 mm Hg. d. pH 7.38, PaCO2 45 mm Hg.

Correct Answer: B Rationale: The patient's low pH in spite of a respiratory alkalosis indicates that the patient has severe metabolic acidosis and is experiencing the progressive stage of shock; rapid changes in therapy are needed. The values in the answer beginning "pH 7.48" suggest a mild respiratory alkalosis (consistent with compensated shock). The values in the answer beginning "pH 7.41" suggest compensated respiratory acidosis. The values in the answer beginning "pH 7.38" are normal.

25. To monitor a patient with severe acute pancreatitis for the early organ damage associated with MODS, the most important assessments for the nurse to make are a. stool guaiac and bowel sounds. b. lung sounds and oxygenation status. c. serum creatinine and urinary output. d. serum bilirubin levels and skin color.

Correct Answer: B Rationale: The respiratory system is usually the system to show the signs of MODS because of the direct effect of inflammatory mediators on the pulmonary system. The other assessment data are also important to collect, but they will not indicate the development of MODS as early.

18. When the nurse is caring for a patient in cardiogenic shock who is receiving dobutamine (Dobutrex) and nitroglycerin (Tridil) infusions, the best evidence that the medications are effective is that the a. systolic BP increases to greater than 100 mm Hg. b. cardiac monitor shows sinus rhythm at 96 beats/min. c. PAWP drops to normal range. d. troponin and creatine kinase levels decrease.

Correct Answer: C Rationale: Because PAWP is increased in cardiogenic shock as a result of the increase in volume and pressure in the left ventricle, normalization of PAWP is the best indicator of patient improvement. The changes in BP and heart rate could occur with dobutamine infusion even if patient tissue perfusion was not improved. Troponin and creatine kinase (CK) levels are indicators of cardiac cellular death and are not used as indicators of improved tissue perfusion.

24. When caring for a patient with cardiogenic shock and possible MODS, which information obtained by the nurse will help confirm the diagnosis of MODS? a. The patient has crackles throughout both lung fields. b. The patient complains of 8/10 crushing chest pain. c. The patient has an elevated ammonia level and confusion. d. The patient has cool extremities and weak pedal pulses.

Correct Answer: C Rationale: The elevated ammonia level and confusion suggest liver failure in addition to the cardiac failure. The crackles, chest pain, and cool extremities are all consistent with cardiogenic shock and do not indicate that there are failures in other major organ systems.

The most accurate assessment parameters for the nurse to use to determine adequate tissue perfusion in the pt. with MODS are A) BP, HR, RR B) LS, BP, temp C) pulse pressure, LOC, and papillary response D) LOC, urine output, and skin color and temp

D) LOC, urine output, and skin color and temp

A pt. has a spinal cord injury at T4. VS include a falling BP with bradycardia. The nurse recognizes that the pt. is experiencing A) a relative hypervolemia B) an absolute Hypovolemia C) Neurogenic shock from low blood flow D) Neurogenic shock from massive vasodilation

D) Neurogenic shock from massive vasodilation

Which burn injury is essentially painless in the wound bed? A. First degree B. Superficial partial thickness (second degree) C. Deep partial thickness (second degree) D. Third degree

D. Third degree

Neurogenic shock is caused by: A. spinal cord injuries below T6. B. inhibition of the parasympathetic nervous system. C. injury to the cerebral cortex. D. a lack of sympathetic activity.

D. a lack of sympathetic activity.

All of the following alterations would indicate that a patient is developing liver failure in MODS except: A. increased serum ammonia levels. B. jaundice. C. increased levels of liver enzymes. D. increased serum urea levels.

D. increased serum urea levels.

The nurse assesses the patient for the negative effect of IV nitroglycerin (Tridil) for shock management which is:

Decreased blood pressure.

The nurse caring for a patient post epidural anesthesia notices that the patient is beginning to evidence symptoms of shock. The nurse assesses the patient for what type of shock?

Distributed

The nurse is caring for a client who is in neurogenic shock. The nurse knows that this is a subcategory of what kind of shock?

Distributive pg. 301

The community health nurse finds the client collapsed outdoors. The nurse assesses that the client is shallow breathing and has a weak pulse. A neighbor calls 911. Which nursing action is helpful while waiting for the ambulance?

Elevate the legs higher than the heart.

The community health nurse finds the client collapsed outdoors. The nurse assesses that the client is shallow breathing and has a weak pulse. The 911 is called by the neighbor. Which nursing action is helpful while waiting for the ambulance?

Elevate the legs higher than the heart.

A client who experienced shock is now nonresponsive and having cardiac dysrhythmias. The client is being mechanically ventilated, receiving medications to maintain renal perfusion, and is not responding to treatment. In this stage, it is most important for the nurse to

Encourage the family to touch and talk to the client. pg. 292

A patient visits a health clinic because of urticaria and shortness of breath after being stung by several wasps. The nurse practitioner immediately administers which medication to reduce bronchospasm?

Epi

A nurse is assisting with the orientation of a newly hired graduate. Which of the following behaviors of the graduate nurse would the other nurse identify as not adhering to strict infection control practices?

Hanging tape on the bedside table when changing a wet-to-dry sterile dressing

A nurse is assisting with the orientation of a newly hired graduate. Which of the following behaviors of the graduate nurse would the other nurse identify as not adhering to strict infection control practices?

Hanging tape on the bedside table when changing a wet-to-dry sterile dressing pg. 301

Elevating the patient's legs slightly to improve cerebral circulation is contraindicated in which of the following disease processes?

Head injury

You are caring for a client in the compensation stage of shock. You know that in this stage of shock epinephrine and norepinephrine are released into the circulation. What positive effect does this have on your client?

Increase myocardial contractility

The nurse is administering a medication to the client with a positive inotropic effect. Which action of the medication does the nurse anticipate?

Increase the force of myocardial contraction

A patient is in the progressive stage of shock with lung decompensation. What treatment does the nurse anticipate assisting with?

Intubation and mechanical ventilation

A patient is in the progressive stage of shock with lung decompensation. What treatment does the nurse anticipate assisting with?

Intubation and mechanical ventilation pg. 290

Depleted adenosine triphosphate (ATP) stores and multiple organ failure are characteristics of which stage of shock?

Irreversible

The nurse is aware that fluid replacement is a hallmark treatment for shock. Which of the following is the crystalloid fluid that helps treat acidosis?

Lactated Ringer's

The nurse is aware that fluid replacement is a hallmark treatment for shock. Which of the following is the crystalloid fluid that helps treat acidosis?

Lactated Ringer's pg. 293

A client has experienced hypovolemic shock and is being treated with 2 liters of lactated Ringer's solution. It is now most important for the nurse to assess

Lung sounds

A nurse practitioner visits a patient in a cardiac care unit. She assesses the patient for shock, knowing that the primary cause of cardiogenic shock is:

MI

The nurse is caring for a client newly diagnosed with sepsis. The client has a serum lactate concentration of 6 mmol/L and fluid resuscitation has been initiated. Which value indicates that the client has received adequate fluid resuscitation?

Mean arterial pressure of 70 mm Hg

A client experiencing vomiting and diarrhea for 2 days has a blood pressure of 88/56, a pulse rate of 122 beats/minute, and a respiratory rate of 28 breaths/minute. The nurse places the client in which position?

Modified Trendelenburg

A client experiencing vomiting and diarrhea for 2 days has a blood pressure of 88/56, a pulse rate of 122 beats/minute, and a respiratory rate of 28 breaths/minute. The nurse places the client in which position?

Modified Trendelenburg pg. 297

A client experiencing vomiting and diarrhea for 2 days has a blood pressure of 88/56, a pulse rate of 122 beats/minute, and a respiratory rate of 28 breaths/minute. The nurse places the client in which position?

Modified trandeleburg

Which positioning strategy should be utilized for the patient diagnosed with hypovolemic shock?

Modified trendelenburg

A patient arrives in the emergency department with complaints of chest pain radiating to the jaw. What medication does the nurse anticipate administering to reduce pain and anxiety as well as reducing oxygen consumption?

Morphine

A patient arrives in the emergency department with complaints of chest pain radiating to the jaw. What medication does the nurse anticipate administering to reduce pain and anxiety as well as reducing oxygen consumption?

Morphine pg. 299

The nurse determines that a patient in shock is experiencing a decrease in stroke volume when what clinical manifestation is observed?

Narrowed pulse pressure

The nurse determines that a patient in shock is experiencing a decrease in stroke volume when what clinical manifestation is observed?

Narrowed pulse pressure pg. 288

The nurse is evaluating a client in the intensive care unit to identify improvement in the client's condition. Which outcome does the nurse note as the result of inadequate compensatory mechanisms?

Organ damage pg. 292

A client experiences an acute myocardial infarction. Current blood pressure is 90/58, pulse is 118 beats/minute, and respirations are 30 breaths/minute. The nurse intervenes first by administering the following prescribed treatment:

Oxygen at 2 L/min by nasal cannula

A client experiences an acute myocardial infarction. Current blood pressure is 90/58, pulse is 118 beats/minute, and respirations are 30 breaths/minute. The nurse intervenes first by administering the following prescribed treatment:

Oxygen at 2 L/min by nasal cannula pg. 299

The nurse is caring for a 65-yr-old man with acute respiratory distress syndrome (ARDS) who is on pressure support ventilation (PSV), fraction of inspired oxygen (FIO2) at 80%, and positive end-expiratory pressure (PEEP) at 15 cm H2O. The patient weighs 72 kg. What finding would indicate that treatment is effective? PaO2 of 60 mm Hg Tidal volume of 700 mL Cardiac output of 2.7 L/min Inspiration to expiration ratio of 1:2

PaO2 of 60 mm Hg Severe hypoxemia (PaO2 less than 40 mm Hg) occurs with ARDS, and PEEP is increased to improve oxygenation and prevent oxygen toxicity by reducing FIO2. A PaO2 level of 60 mm Hg indicates that treatment is effective and oxygenation status has improved. Decreased cardiac output is a complication of PEEP. Normal cardiac output is 4 to 8 L/minute. Normal tidal volume is 6 to 10 mL/kg. PSV delivers a preset pressure but the tidal volume varies with each breath. I:E ratio is usually set at 1:2 to 1:1.5 and does not indicate patient improvement.

Which interventions should the nurse perform before using an open-suctioning technique for a patient with an endotracheal (ET) tube (select all that apply.)? Put on clean gloves. Administer a bronchodilator. Perform a cardiopulmonary assessment. Hyperoxygenate the patient for 30 seconds. Perform hand hygiene before performing the procedure. Insert a few drops of normal saline into the ET to break up secretions.

Perform a cardiopulmonary assessment. Hyperoxygenate the patient for 30 seconds. Suctioning is preceded by a thorough assessment and hyperoxygenation for 30 seconds. Sterile, not clean, gloves are necessary, and it is not necessary to administer a bronchodilator. Instillation of normal saline into the ET tube is not an accepted standard practice.

The nurse is caring for a client who is developing hypovolemic shock from a duodenal ulcer bleed. What is the first intervention the nurse can provide to facilitate blood flow to the brain?

Place the client in a modified Trendelenburg position.

Morphine sulfate has which of the following effects on the body?

Reduces preload

Morphine sulfate has which of the following effects on the body?

Reduces preload pg. 299

A 57-year-old client has been brought to your ED via squad. He is unresponsive, and his wife reports his symptoms of elevated temperature and flushed skin. Physical assessment reveals a rapid, bounding pulse. The high school where the client is employed has had a significant increase in cases of staphylococcal and streptococcal infections among student athletes. His labs show an elevated WBC; cultures are forthcoming. You suspect which of the following may be the cause of the client's present condition?

Septic

The nurse anticipates that an immunosuppressed client is at greatest risk for which type of shock?

Septic

The nurse anticipates that a client who is immunosuppressed is at the greatest risk for developing which type of shock?

Septic pg. 301

The client exhibits a blood pressure of 110/68 mm Hg, pulse rate of 112 beats/min, temperature of 102°F with skin warm and flushed. Respirations are 30 breaths/min. The nurse assesses the client may be exhibiting the early stage of which shock?

Septic pg. 302

The nurse in collaboration with respiratory therapy is determining a patient's readiness to wean from the ventilator. Which finding indicates the patient is not a candidate for weaning (select all that apply.)? Minute volume of 8 L/min Patient follow commands Serum hemoglobin of 6 g/dL Respirations of 28 breaths/min Mean arterial pressure (MAP) of 45 mm Hg Negative inspiratory force (NIF) of -15 cm H2O

Serum hemoglobin of 6 g/dL Mean arterial pressure (MAP) of 45 mm Hg Negative inspiratory force (NIF) of -15 cm H2O Findings that support readiness for weaning are minute volume of 8 L/min, patient is alert and follow commands, and respirations of 28 breaths/min. Findings that indicate the patient is not ready for weaning include serum hemoglobin of 6 g/dL, mean arterial pressure (MAP) of 45 mm Hg, and negative inspiratory force (NIF) of -15 cm H2O. Extubating a patient with severe anemia, poor perfusion, and weakened breathing effort will likely result in poor outcomes such as worsening of condition and reintubation.

The nursing instructor is talking with a group of senior nursing students about shock. When caring for a patient at risk for shock what assessment finding would the nurse consider a potential sign of shock?

Shallow, rapid respirations pg.

3 Certification as an advance care nurse practitioner (ACNP) is available through the American Association of Critical Care Nurses (AACN). The National League of Nurses (NLN), the American Association of Nurse Practitioners (AANP), and the National Council of State Boards of Nursing (NCSBN) are all nursing organizations but they do not offer certification to the advance practice registered nurse (APRN) as an ACNP. Text Reference - p. 1600

The advance practice registered nurse (APRN) is seeking certification as an advance care nurse practitioner (ACNP). Which professional organization can grant this certification to the APRN? 1 National League of Nurses (NLN) 2 American Association of Nurse Practitioners (AANP) 3 American Association of Critical Care Nurses (AACN) 4 National Council of State Boards of Nursing (NCSBN

You are talking with the family of a client who is in the irreversible stage of shock. They ask you why the physician has told the family that the client is going to die. What would you explain to this family?

The client is not responding to medical interventions.

You are caring for a client with a stage IV leg ulcer. You are closely monitoring the client for sepsis. What would indicate that sepsis has occurred and that you should notify the physician of immediately?

The client's heart rate is greater than 90 beats per minute.

1, 3, 4 In order to achieve the designation as a pediatric critical care nurse (CCRN) the nurse will require registered nurse licensure, successful completion of a written test, and pediatric intensive care clinical experience. An advance practice degree and recommendation from a pediatric physician are not required to achieve CCRN certification. Text Reference - p. 1599

The critical care nurse is pursuing certification as a pediatric critical care nurse (CCRN). What will the nurse need to accomplish to achieve this certification? Select all that apply. 1 Registered nurse licensure 2 An advanced practice degree 3 Successful completion of a written test 4 Pediatric intensive care clinical experience 5 Recommendation from a pediatric physician

The nurse is obtaining physician orders which include a pulse pressure. The nurse is most correct to report which of the following?

The difference between the systolic and diastolic pressure

4 Pulse oximetry is a noninvasive and continuous method of determining the oxygen saturation of SpO2. Monitoring SpO2 may reduce the frequency of arterial blood gas (ABG) sampling. SpO2 is normally 95% to 100%. A value less than that may indicate hypoperfusion. Text Reference - p. 1609

The health care provider requests constant hemoglobin (SpO2) monitoring for a patient. What is the normal range of SpO2, which indicates that the saturation pressure of oxygen in this patient is adequate? 1 80-85% 2 85-90% 3 90-95% 4 95-100%

1 Preload, afterload, and contractility determine stroke volume (SV). Cardiac output and heart rate are used to determine stroke volume; however, body surface area is used to determine cardiac index. Mean arterial pressure is used to determine afterload, not stroke volume. Cardiac index is a more precise measurement of the efficiency of the heart's pumping action; it is not used to determine stroke volume. Text Reference - p. 1603

The nurse is concerned about a patient's stroke volume. What determines stroke volume? 1 Preload, afterload, and contractility 2 Cardiac output, heart rate, and body surface area 3 Afterload, cardiac output, and mean arterial pressure 4 Cardiac index, mean arterial pressure, and blood pressure

3 The most commonly used mechanical circulatory assist device is the IABP, and it is used to decrease ventricular workload, increase myocardial perfusion, and augment circulation. Cardiopulmonary bypass provides circulation during open heart surgery. It is not used as an assist device after surgery. ICG is a noninvasive method to obtain cardiac output and assess thoracic fluid status. CVP measurement is an invasive measurement of right ventricular preload and reflects fluid volume problems. Text Reference - p. 1611

The patient has developed cardiogenic shock after a left anterior descending myocardial infection. Which circulatory assist device should the nurse expect to use for this patient? 1 Cardiopulmonary bypass 2 Impedance cardiography (ICG) 3 Intraaortic balloon pump (IABP) 4 Central venous pressure (CVP) measuremen

3 Critical care nurses provide care for patients with acute problems who are unstable. Patients with acute problems who are stable are often cared for on a medical-surgical unit versus the critical care unit. Patients with chronic problems who are stable may be cared for in the community setting. Patients with chronic problems who are unstable may require care in a rehabilitation or medical-surgical setting. Text Reference - p. 1598

The seasoned nurse is orienting a novice nurse to the critical care unit. When teaching the novice nurse about critical care nursing, which statement is most appropriate by the seasoned nurse? 1 "We care for patients with acute problems who are stable." 2 "We care for patients with chronic problems who are stable." 3 "We care for patients with acute problems who are unstable." 4 "We care for patients with chronic problems who are unstable."

The nurse is reviewing diagnostic lab work of a client developing shock. Which laboratory result does the nurse note as a key in determining the type of shock?

WBC: 42,000/mm3

The nurse is reviewing diagnostic lab work of a client developing shock. Which laboratory result does the nurse note as a key in determining the type of shock?

WBC: 42,000/mm3 pg. 302

2 Positive end-expiratory pressure (PEEP) therapy is given to patients with pulmonary edema to provide a counter pressure opposing fluid extravasation. PEEP is not used for patients with hypovolemia, low cardiac output and unilateral or nonuniform lung disease because in those patients, the adverse effects of PEEP may outweigh any benefits. Text Reference - p. 1621

What condition would the nurse infer that a patient undergoing positive end-expiratory pressure (PEEP) therapy has? 1 Hypovolemia 2 Pulmonary edema 3 Low cardiac output 4 Unilateral lung disease

37.5 The formula to calculate the rapid shallow breathing index is f/VT. Therefore, rapid shallow breathing index = 15/0.4 = 37.5 breaths/min/L. Text Reference - p. 1626

What is the rapid shallow breathing index of a patient whose spontaneous respiratory rate (f) is 15 breaths/min and spontaneous tidal volume (VT) is 0.4 L? Record your answer using one decimal place. ___________________breaths/min/L

1 Both negative pressure ventilation and positive pressure ventilation involve passive expiration. Negative pressure ventilation does not require an artificial airway. Positive pressure ventilation may require invasive mechanical ventilation through an artificial airway. Positive pressure ventilation is primarily used with acutely ill patients; negative pressure ventilation is not routinely used for acutely ill patients. In positive pressure ventilation, intrathoracic pressure is raised during lung inflation but in negative pressure ventilation, intrathoracic pressure is decreased during lung inflation. Text Reference - p. 1618

What is the similarity between negative pressure ventilation and positive pressure ventilation? 1 Both involve passive expiration. 2 Both require an artificial airway. 3 Both are routinely used for acutely ill patients. 4 Both raise intrathoracic pressure during lung inflation.

3 A dynamic response test is performed every 8 to 12 hours, as well as when the system is opened to air or the accuracy of the measurements is questioned. It involves activating the fast flush and checking that the equipment reproduces a distortion-free signal. A square wave indicates a normal response and requires no further action. This waveform does not indicate that the line needs to be flushed. Nor does it indicate that zero needs to be reconfirmed or that the wrist needs to be repositioned. Text Reference - p. 1605

When performing a dynamic response test, the nurse observes the following tracing. What action should the nurse perform based on this tracing?

3 Systemic vascular resistance is an index of left ventricular afterload. Central venous pressure is an index of preload. Pulmonary arterial pressure and peripheral vascular resistance are indices of right ventricular afterload. Text Reference - p. 1604

Which hemodynamic value should the nurse use to determine a patient's left ventricular afterload? 1 Central venous pressure 2 Pulmonary arterial pressure 3 Systemic vascular resistance 4 Peripheral vascular resistance

1 Patients with preexisting dementia, such as Alzheimer's disease, are at an increased risk for developing delirium when receiving care in the intensive care unit (ICU). Diabetes mellitus, Parkinson's disease, and multiple sclerosis are not known risk factors for developing delirium. Text Reference - p. 1601

Which is the recommendation from the American Association of Critical Care Nursing (AACN) regarding family visitation in the intensive care unit (ICU)? 1 Individualized visitation 2 Visitation on the evening shift 3 Visitation as prescribed by the physician 4 Hourly visitations occurring on each shift

1 In a patient with a head injury, positive pressure ventilation decreases the venous return because of the increase in intrathoracic pressure. Increased intrathoracic pressure causes jugular vein distension rather than compression. Positive pressure ventilation increases the cerebral volume. A decrease in venous return causes an increase in intracranial pressure. Text Reference - p. 1623

Which neurologic complication may occur in a patient with a head injury who is on positive pressure ventilation? 1 Decrease in venous return 2 Compression of jugular vein 3 Reduction of cerebral volume 4 Reduction of intracranial pressure

4 The electronic or teleICU assists the bedside ICU team by monitoring the patient from a remote location using informatics. The ICU, CCU, and PICU are traditional critical care units. Text Reference - p. 1599

Which type of critical care unit uses informatics to monitor a critically ill patient from a remote location? 1 Intensive care unit (ICU) 2 Coronary care unit (CCU) 3 Pediatric intensive care unit (PICU) 4 Electronic intensive care unit (teleICU)

19. The nurse is caring for a 33-year-old patient who arrived in the emergency department with acute respiratory distress. Which assessment finding by the nurse requires the most rapid action? * a. The patient's PaO2 is 45 mm Hg. b. The patient's PaCO2 is 33 mm Hg. c. The patient's respirations are shallow. d. The patient's respiratory rate is 32 breaths/minute.

a

Oliguria occurs in the progressive stage of shock because the kidneys decompensate. To verify this condition, the nurse should expect all of the following S&S except?

a mean arterial BP greater than 70 mmHG

A patient has a PaO2 of 50 mm Hg and a PaCO2 of 42 mm Hg because of an intrapulmonary shunt. Which therapy is the patient most likely to respond best to? a. Positive pressure ventilation b. Oxygen administration at a FIO2 of 100% c. Administration of O2 per nasal cannula at 1 to 3 L/min d. Clearance of airway secretions with coughing and suctioning

a. Patients with a shunt are usually more hypoxemic than patients with a V/Q mismatch because the alveoli are filled with fluid, which prevents gas exchange. Hypoxemia resulting from an intrapulmonary shunt is usually not responsive to high O2 concentrations and the patient will usually require positive pressure ventilation. Hypoxemia associated with a V/Q mismatch usually responds favorably to O2 administration at 1 to 3 L/min by nasal cannula. Removal of secretions with coughing and suctioning is generally not effective in reversing an acute hypoxemia resulting from a shunt.

A pressure (PEEP) of 10 cm H2O. A sign that alerts the nurse to undesirable effects of increased airway and thoracic pressure is a. decreased BP. b. decreased PaO2. c. increased crackles. d. decreased spontaneous respirations.

a. Rationale: Positive-pressure ventilation, especially with end-expiratory pressure, increases intrathoracic pressure with compression of thoracic vessels, resulting in decreased venous return to the heart, decreased left ventricular end-diastolic volume (preload), decreased CO, and lowered BP. None of the other factors is related to increased intrathoracic pressure.

In caring for the patient with ARDS, what is the most characteristic sign the nurse would expect the patient to exhibit? a. Refractory hypoxemia c. Progressive hypercapnia b. Bronchial breath sounds d. Increased pulmonary artery wedge pressure (PAWP)

a. Refractory hypoxemia, hypoxemia that does not respond to increasing concentrations of oxygenation by any route, is a hallmark of ARDS and is always present. Bronchial breath sounds may be associated with the progression of ARDS. PaCO2 levels may be normal until the patient is no longer able to compensate in response to the hypoxemia. Pulmonary artery wedge pressure (PAWP) that is normally elevated in cardiogenic pulmonary edema is normal in the pulmonary edema of ARDS.

Identify two precautions the nurse should take during mouth care and repositioning of an oral ET tube to prevent and detect tube dislodgement.

a. Use two nurses: one to hold the tube while it is untaped or the holder is loosened, and another to perform care. b. After completion of care, confirm the presence of bilateral breath sounds to ensure that the position of the tube was not changed and reconfirm cuff pressure.

Identify five problems associated with inadequate nutrition in the patient receiving prolonged mechanical ventilation.

a. anemia resulting in poor O2 transport; b. decreased respiratory strength; c. delayed weaning; d. decreased resistance to infection; e. prolonged recovery

Identify which of the common three reasons patients are admitted to the ICU apply in the following situations. a. Patient with diabetic ketoacidosis b. Patient with nondisplaced skull fracture who is alert and oriented c. Postoperative patient with mitral valve replacement d. Comatose patient who had an anaphylactic reaction with cardiopulmonary arrest at home with reestablishment of cardiac function

a. physiologically unstable; b. risk for serious complications; c. risk for serious complications; d. intensive nursing support

A client was admitted to the hospital unit with an elevated leukocyte count and a fever accompanied by warm, flushed skin. These symptoms suggest that the client has:

an overwhelming bacterial infection. pg. 301

Which physiologic mechanism of hypoxemia occurs with pulmonary fibrosis? a. Anatomic shunt c. Intrapulmonary shunt b. Diffusion limitation d. V/Q mismatch ratio of less than 1

b. Diffusion limitation in pulmonary fibrosis is caused by thickened alveolar-capillary interface, which slows gas transport.

A patient is receiving 35% O2 via a Venturi mask. To ensure the correct amount of O2 delivery, which action by the nurse is important? a. Teach the patient to keep the mask on during meals. b. Keep the air entrainment ports clean and unobstructed. c. Give a high enough flow rate to keep the bag from collapsing. d. Drain moisture condensation from the corrugated tubing every hour.

b. Keep the air entrainment ports clean and unobstructed. The air entrainment ports regulate the O2 percentage delivered to the patient, so they must be unobstructed. The other options refer to other types of O2 devices. A high O2 flow rate is needed when giving O2by partial rebreather or nonrebreather masks. Draining O2 tubing is necessary when caring for a patient receiving mechanical ventilation. The mask can be removed or changed to a nasal cannula at a prescribed setting when the patient eats.

The nurse suspects that a patient with PEEP is experiencing negative effects of this ventilatory maneuver when which of the following is assessed? a. Increasing PaO2 c. Decreasing heart rate (HR) b. Decreasing blood pressure d. Increasing central venous pressure (CVP)

b. PEEP increases intrathoracic and intrapulmonic pressures, compresses the pulmonary capillary bed, and reduces blood return to both the right and left sides of the heart. Increased PaO2 is an expected effect of PEEP. Preload (CVP) and cardiac output (CO) are decreased, often with a dramatic decrease in BP.

The nurse determines that alveolar hypoventilation is occurring in a patient on a ventilator when a. the patient develops cardiac dysrhythmias. b. auscultation reveals an air leak around the ET cuff. c. ABG results show a PaCO2 of 32 mm Hg and a pH of 7.47. d. the patient tries to breathe faster than the ventilator setting.

b. Rationale: A leaking cuff can lower tidal volume or respiratory rates. An SIMV rate that is too low, the presence of lung secretions, or obstruction can decrease tidal volume. A decreased PaCO2 and increased pH indicate a respiratory alkalosis from hyperventilation, and cardiac dysrhythmias can occur with either hyperventilation or hypoventilation.

A patient has SvO2 of 52%, CO of 4.8 L/min, SpO2 of 95%, and unchanged hemoglobin level. The nurse should assess the patient for a. dysrhythmias. b. pain on movement. c. pulmonary edema. d. signs of septic shock.

b. Rationale: The normal mixed venous oxygen saturation of 60% to 80% becomes decreased with decreased arterial oxygenation, low CO, low hemoglobin, or increased oxygen consumption. With normal CO, arterial oxygenation, and hemoglobin, the factor that is responsible for decreased SvO2 is increased oxygen consumption, which can result from increased metabolic rate, pain, movement, or fever.

Ventricular assist devices are designed to a. provide permanent, total circulatory support when the left ventricle fails. b. temporarily partially or totally support circulation until a donor heart can be obtained. c. support circulation only when patients cannot be weaned from cardiopulmonary bypass. d. reverse the effects of circulatory failure in patients with acute myocardial infarction (MI) in cardiogenic shock.

b. Rationale: Ventricular assist devices are temporary devices that can partially or totally support circulation until the heart recovers and can be weaned from cardiopulmonary bypass or when a donor heart can be obtained. The devices currently available do not permanently support circulation.

A patient with ARDS has a nursing diagnosis of risk for infection. To detect the presence of infections commonly associated with ARDS, what should the nurse monitor? a. Gastric aspirate for pH and blood c. Subcutaneous emphysema of the face, neck, and chest b. Quality, quantity, and consistency of sputum d. Mucous membranes of the oral cavity for open lesions

b. Ventilator-associated pneumonia (VAP) is one of the most common complications of ARDS. Early detection requires frequent monitoring of sputum smears and cultures and assessment of the quality, quantity, and consistency of sputum. Prevention of VAP is done with strict infection control measures, ventilator bundle protocol, and subglottal secretion drainage. Blood in gastric aspirate may indicate a stress ulcer and subcutaneous emphysema of the face, neck, and chest occurs with barotrauma during mechanical ventilation. Oral infections may result from prophylactic antibiotics and impaired host defenses but are not common.

To verify the correct placement of an oral endotracheal tube (ET) after insertion, the best initial action by the nurse is to a. obtain a portable chest x-ray. b. use an end-tidal CO2 monitor. c. auscultate for bilateral breath sounds. d. observe for symmetrical chest movement.

b. use an end-tidal CO2 monitor. End-tidal CO2 monitors are currently recommended for rapid verification of ET placement. Auscultation for bilateral breath sounds and checking chest expansion are also used, but they are not as accurate as end-tidal CO2 monitoring. A chest x-ray confirms the placement but is done after the tube is secured.

In cardiogenic shock, decreased cardiac contractility leads to all of the following compensatory responses except? a) decreased SV b) decreased tissue perfusion c) increased SV d) pulmonary congestion

c) increased stroke volume

The nurse notes thick, white secretions in the endotracheal tube (ET) of a patient who is receiving mechanical ventilation. Which intervention will most directly treat this finding? a. Reposition the patient every 1 to 2 hours. b. Increase suctioning frequency to every hour. c. Add additional water to the patient's enteral feedings. d. Instill 5 mL of sterile saline into the ET before suctioning.

c. Add additional water to the patient's enteral feedings. ANS: C Because the patient's secretions are thick, better hydration is indicated. Suctioning every hour without any specific evidence for the need will increase the incidence of mucosal trauma and would not address the etiology of the ineffective airway clearance. Instillation of saline does not liquefy secretions and may decrease the SpO2. Repositioning the patient is appropriate but will not decrease the thickness of secretions.

The nurse suspects the early stage of ARDS in any seriously ill patient who manifests what? a. Develops respiratory acidosis c. Exhibits dyspnea and restlessness b. Has diffuse crackles and rhonchi d. Has a decreased PaO2 and an increased PaCO2

c. Early signs of ARDS are insidious and difficult to detect but the nurse should be alert for any early signs of hypoxemia, such as dyspnea, restlessness, tachypnea, cough, and decreased mentation, in patients at risk for ARDS. Abnormal findings on physical examination or diagnostic studies, such as adventitious lung sounds, signs of respiratory distress, respiratory alkalosis, or decreasing PaO2 , are usually indications that ARDS has progressed beyond the initial stages.

A client who is suffering a myocardial infarction is transported to the ED by ambulance. This client is at greatest risk for developing which type of shock?

cardiogenic shock

16. After receiving change-of-shift report on a medical unit, which patient should the nurse assess first? * a. A patient with cystic fibrosis who has thick, green-colored sputum b. A patient with pneumonia who has crackles bilaterally in the lung bases c. A patient with emphysema who has an oxygen saturation of 90% to 92% d. A patient with septicemia who has intercostal and suprasternal retractions

d

What is the primary reason that hemodynamic monitoring is instituted in severe respiratory failure? a. To detect V/Q mismatches b. To continuously measure the arterial BP c. To evaluate oxygenation and ventilation status d. To evaluate cardiac status and blood flow to tissues

d. Hemodynamic monitoring with a pulmonary artery catheter is instituted in severe respiratory failure to determine the amount of blood flow to tissues and the response of the lungs and heart to hypoxemia. Continuous BP monitoring may be performed but BP is a reflection of cardiac activity, which can be determined by the pulmonary artery catheter findings. Arterial blood gases (ABGs) are important to evaluate oxygenation and ventilation status and V/Q mismatches.

The nurse notes premature ventricular contractions (PVCs) while suctioning a patient's endotracheal tube. Which next action by the nurse is indicated? a. Plan to suction the patient more frequently. b. Decrease the suction pressure to 80 mm Hg. c. Give antidysrhythmic medications per protocol. d. Stop and ventilate the patient with 100% oxygen.

d. Stop and ventilate the patient with 100% oxygen. Dysrhythmias during suctioning may indicate hypoxemia or sympathetic nervous system stimulation. The nurse should stop suctioning and ventilate the patient with 100% O2. There is no indication that more frequent suctioning is needed. Lowering the suction pressure will decrease the effectiveness of suctioning without improving the hypoxemia. Because the PVCs occurred during suctioning, there is no need for antidysrhythmic medications (which may have adverse effects) unless they recur when the suctioning is stopped and patient is well oxygenated.

The nurse educator is evaluating the care that a new registered nurse (RN) provides to a patient receiving mechanical ventilation. Which action by the new RN indicates the need for more education? a. The RN increases the FIO2 to 100% before suctioning. b. The RN secures a bite block in place using adhesive tape. c. The RN asks for assistance to resecure the endotracheal tube. d. The RN positions the patient with the head of bed at 10 degrees.

d. The RN positions the patient with the head of bed at 10 degrees. The head of the patient's bed should be positioned at 30 to 45 degrees to prevent ventilator-associated pneumonia. The other actions by the new RN are appropriate.

Four hours after mechanical ventilation is initiated, a patient's arterial blood gas (ABG) results include a pH of 7.51, PaO2 of 82 mm Hg, PaCO2 of 26 mm Hg, and HCO3- of 23 mEq/L (23 mmol/L). The nurse will anticipate the need to a. increase the FIO2. c. increase the respiratory rate. b. increase the tidal volume. d. decrease the respiratory rate.

d. decrease the respiratory rate. The patient's PaCO2 and pH indicate respiratory alkalosis caused by too high a respiratory rate. The PaO2 is appropriate for a patient with COPD and increasing the respiratory rate and tidal volume would further lower the PaCO2.

Sympathomimetic drugs increase CO by all of the following measure except? a) decreasing preload and afterload b) increasing myocardial contractility c) increasing stroke volume d) increasing cardiac output

decreasing preload and afterload

A nurse knows that the major clinical use of dobutamine (Dobutrex) is to:

increase cardiac output. pg. 299

The nurse is caring for a critically ill client. Which of the following is the nurse correct to identify as a positive effect of catecholamine release during the compensation stage of shock?

increase in arterial ozygenation

The nurse assesses for negative effect of IV nitroglycerin (Tridil) for shock management , which is?

increased BP

A nurse knows that the major clinical use of dobutamine (Dobutrex) is to:

increased cardiac output

A client with a history of depression is brought to the ED after overdosing on Valium. This client is at risk for developing which type of distributive shock?

neurogenic shock

A client is experiencing vomiting and diarrhea for 2 days. Blood pressure is 88/56, pulse rate is 122 beats/minute, and respirations are 28 breaths/minute. The nurse starts intravenous fluids. Which of the following prescribed prn mediciations would the nurse administer next?

ondansetron (Zofran)

Morphine sulfate has which of the following effects on the body?

reduces preload

Stress ulcers occur frequently in acutely ill patient. Which of the following medications would be used to prevent ulcer formation? Select all that apply.

• Lansoprazole (Prevacid) • Famotidine (Pepcid) • Ranitidine (Zantac)

The nurse is planning care for a client diagnosed with cardiogenic shock. Which nursing intervention is most helpful to decrease myocardial oxygen consumption?

Maintain activity restriction to bedrest.

The nurse is caring for a patient newly diagnosed with sepsis. The patient has a serum lactate level of 6 mmol/L and fluid resuscitation has been initiated. Which of the following indicates that the fluid resuscitation received by the patient is adequate?

Mean arterial pressure (MAP) of 70 mm Hg

You are the nurse caring for a client in septic shock. You know to closely monitor your client. What finding would you observe when the client's condition is in its initial stages?

Rapid, bounding pulse

23. Which information about a patient who is receiving cisatracurium (Nimbex) to prevent asynchronous breathing with the positive pressure ventilator requires immediate action by the nurse? * a. Only continuous IV opioids have been ordered. b. The patient does not respond to verbal stimulation. c. There is no cough or gag when the patient is suctioned. d. The patient's oxygen saturation fluctuates between 90% to 93%.

a

26. The nurse reviews the electronic medical record for a patient scheduled for a total hip replacement. Which assessment data shown in the accompanying figure increase the patient's risk for respiratory complications after surgery? * a. Albumin level and recent weight loss b. Mild confusion and recent weight loss c. Age and recent arthroscopic procedure. d. Anemia and recent arthroscopic procedure

a

The nurse is caring for a motor vehicle accident client who is unresponsive on arrival to the emergency department. The client has numerous fractures, internal abdominal injuries, and large lacerations on the head and torso. The family arrives and seeks update on the client's condition. A family member asks, "What causes the body to go into shock?"Given the client's condition, which statement is most correct?

"The client is in shock because the blood volume has decreased in the system."

17. A patient with chronic obstructive pulmonary disease (COPD) arrives in the emergency department complaining of shortness of breath and dyspnea on minimal exertion. Which assessment finding by the nurse is most important to report to the health care provider? a. The patient has bibasilar lung crackles. b. The patient is sitting in the tripod position. c. The patient's respirations have decreased from 30 to 10 breaths/minute. d. The patient's pulse oximetry indicates an O2 saturation of 91%.

*

MULTIPLE RESPONSE 1. Which actions should the nurse initiate to reduce the risk for ventilator-associated pneumonia (VAP) (select all that apply)? a. Obtain arterial blood gases daily. b. Provide a "sedation holiday" daily. c. Elevate the head of the bed to at least 30°. d. Give prescribed pantoprazole (Protonix). e. Provide oral care with chlorhexidine (0.12%) solution daily.

* ANS: B, C, D, E

13. A nurse is caring for a patient who is orally intubated and receiving mechanical ventilation. To decrease the risk for ventilator-associated pneumonia, which action will the nurse include in the plan of care? a. Elevate head of bed to 30 to 45 degrees. b. Suction the endotracheal tube every 2 to 4 hours. c. Limit the use of positive end-expiratory pressure. d. Give enteral feedings at no more than 10 mL/hr.

* a

22. A nurse is caring for a patient with acute respiratory distress syndrome (ARDS) who is receiving mechanical ventilation using synchronized intermittent mandatory ventilation (SIMV). The settings include fraction of inspired oxygen (FIO2) 80%, tidal volume 450, rate 16/minute, and positive end-expiratory pressure (PEEP) 5 cm. Which assessment finding is most important for the nurse to report to the health care provider? * a. Oxygen saturation 99% b. Respiratory rate 22 breaths/minute c. Crackles audible at lung bases d. Heart rate 106 beats/minute

* a

12. The nurse documents the vital signs for a patient admitted 2 days ago with gram-negative sepsis: temperature 101.2° F, blood pressure 90/56 mm Hg, pulse 92, respirations 34. Which action should the nurse take next? a. Give the scheduled IV antibiotic. b. Give the PRN acetaminophen (Tylenol). c. Obtain oxygen saturation using pulse oximetry. d. Notify the health care provider of the patient's vital signs.

* c

2. While caring for a patient who has been admitted with a pulmonary embolism, the nurse notes a change in the patient's oxygen saturation (SpO2) from 94% to 88%. Which action should the nurse take next? a. Increase the oxygen flow rate. b. Suction the patient's oropharynx. c. Instruct the patient to cough and deep breathe. d. Help the patient to sit in a more upright position.

*ANSWER A

5. A nurse is caring for an obese patient with right lower lobe pneumonia. Which position will be best to improve gas exchange? a. On the left side b. On the right side c. In the tripod position d. In the high-Fowler's position

*ANSWER A

6. When admitting a patient with possible respiratory failure with a high PaCO2, which assessment information should be immediately reported to the health care provider? a. The patient is somnolent. b. The patient complains of weakness. c. The patient's blood pressure is 164/98. d. The patient's oxygen saturation is 90%.

*ANSWER A

4. The oxygen saturation (SpO2) for a patient with left lower lobe pneumonia is 90%. The patient has rhonchi, a weak cough effort, and complains of fatigue. Which action is a priority for the nurse to take? a. Position the patient on the left side. b. Assist the patient with staged coughing. c. Place a humidifier in the patient's room. d. Schedule a 2-hour rest period for the patient.

*ANSWER B

1. To evaluate the effectiveness of ordered interventions for a patient with ventilatory failure, which diagnostic test will be most useful to the nurse? a. Chest x-ray b. Oxygen saturation c. Arterial blood gas analysis d. Central venous pressure monitoring

*ANSWER C

When a patient in shock is receiving fluid replacement, what should the nurse monitor frequently? (Select all that apply.)

- Urinary output - Mental status - Vital signs pg. 293

A nurse is caring for a patient undergoing mechanical ventilation who is also receiving positive end-expiratory pressure (PEEP). What is the outcome that the nurse hopes to achieve with PEEP? 1 Expand collapsed alveoli 2 Decrease alveolar volume 3 Decrease bronchospasms 4 Prevent spontaneous breathing

1 Expand collapsed alveoli Positive end-expiratory pressure (PEEP) expands collapsed alveoli and improves resting lung volume by keeping the alveoli open and preventing them from collapsing during expiration. PEEP increases, not decreases, alveolar volume by keeping the alveoli expanded. PEEP has no direct effect on bronchospasms. PEEP allows, not prevents, spontaneous breathing of a patient undergoing mechanical respiration.

A feeding tube is placed in a patient receiving positive pressure ventilation to prevent inadequate nutrition. What should the nurse avoid while verifying the placement of the feeding tube? 1 Listening for air after injection 2 X-ray confirmation before initial use 3 Review of routine x-rays and aspirate 4 Marking and assessing the tube's exit site

1 Listening for air after injection While verifying the placement of feeding tube, the nurse should avoid listening for air after injection, because it is not a reliable method for verification of its placement. X-ray confirmation before initial use, review of routine x-rays, and marking and assessing the tube's exit site are all reliable methods for verifying the feeding tube placement.

A patient in the intensive care unit has been intubated for the relief of airway obstruction. What nursing actions should be performed to prevent complications after intubation? Select all that apply. 1 Obtain a chest x-ray exam to confirm the placement. 2 Immediately catheterize the patient and check for urine output. 3 Obtain a computed tomography (CT) scan to note the placement. 4 Auscultate lungs bilaterally and also epigastrium for breath sounds. 5 Use an end-tidal carbon dioxide detector to note presence of exhaled carbon dioxide.

1 Obtain a chest x-ray exam to confirm the placement. 4 Auscultate lungs bilaterally and also epigastrium for breath sounds. 5 Use an end-tidal carbon dioxide detector to note presence of exhaled carbon dioxide. Following an intubation, it is important to confirm the placement of the endotracheal (ET) tube. This confirmation is obtained by x-ray after visualizing the ET tube correctly placed in the trachea. Auscultating lungs for breath sounds confirms that air is going into the lungs and not in the stomach. If the sounds are heard over the epigastrium, it indicates that the ET tube has gone in the stomach. Presence of carbon dioxide in exhaled air also confirms that the tube has gone into the lungs, and the breathing effort is normal. In this case, a CT scan is redundant. However, an x-ray is sufficient to confirm the placement of the ET tube. The patient may require a urinary catheter, but it is not an immediate intervention and can be done after intubation.

patient is weaned from artificial ventilation. What interventions should the nurse perform during weaning? Select all that apply. 1 Obtain baseline vital signs and respiratory parameters. 2 Make the patient walk a bit and then proceed with the trial. 3 Anesthetize or restrain the patient to avoid any resistance. 4 Place the patient in a comfortable sitting or semirecumbent position. 5 Closely monitor for signs and symptoms that may signal intolerance and a need to end the trial.

1 Obtain baseline vital signs and respiratory parameters. 4 Place the patient in a comfortable sitting or semirecumbent position. 5 Closely monitor for signs and symptoms that may signal intolerance and a need to end the trial. During a weaning trial, the patient is placed in a comfortable sitting or semirecumbent position. Baseline vital signs and respiratory parameters should be obtained. The patient should be closely monitored for signs and symptoms that may signal intolerance and a need to end the trial. The patient should not be anesthetized or made to walk

What therapy is provided to a patient with acute respiratory distress syndrome (ARDS)? 1 Mechanical ventilation 2 Oxygen via a Venturi mask 3 Oxygen via a non-rebreather mask 4 Small volume nebulizer treatments

1 A patient with acute respiratory distress syndrome (ARDS) would be intubated and receive mechanical ventilation. Small volume nebulizer treatments would open airways; however, this intervention will not sufficiently treat ARDS. Oxygen via a Venturi mask or a non-rebreather mask would be insufficient to promote oxygenation and perfusion. Text Reference - p. 1651

What occurs when the inflammatory response is activated in a patient with systemic inflammatory response syndrome (SIRS)? 1 Release of mediators 2 Decrease in metabolism 3 Damage of the mesothelium 4 Decrease in vascular permeability

1 A release of mediators occurs when the inflammatory response is activated. Other changes that occur include an increase in metabolism or hypermetabolism, direct damage to the endothelium, and an increase in vascular permeability. Text Reference - p. 1649

A patient has acute respiratory distress syndrome (ARDS) as a result of sepsis. Which measure most likely would be implemented to maintain cardiac output? 1 Administer crystalloid fluids or colloid solutions. 2 Position the patient in the Trendelenburg position. 3 Place the patient on fluid restriction and administer diuretics. 4 Perform chest physiotherapy and assist with staged coughing.

1 Administer crystalloid fluids or colloid solutions. Low cardiac output may necessitate crystalloid fluids or colloid solutions in addition to lowering positive end-expiratory pressure (PEEP) or administering inotropes. The Trendelenburg position (not recommended to treat hypotension) and chest physiotherapy are unlikely to relieve decreased cardiac output, and fluid restriction and diuresis would be inappropriate interventions.

How is systemic inflammatory response syndrome (SIRS) different from multiple organ dysfunction syndrome (MODS)? Correct1 Shock leads to SIRS, and SIRS causes MODS. 2 MODS is reversible, and SIRS has irreversible changes. Incorrect3 SIRS is caused by shock, and MODS is caused by perfusion deficits. 4 Homeostasis fails before SIRS, and homeostasis is maintained in MODS.

1 Any type of shock triggers the systemic inflammatory response. Generalized inflammation in organs remote from the initial shock is systemic inflammatory response syndrome (SIRS). Multiple organ dysfunction syndrome (MODS) results from SIRS. Both SIRS and MODS are reversible in the early stages. SIRS is caused by some kind of injury to the body such as sepsis, ischemia, infarction, and injury; SIRS can lead to MODS if not treated. In SIRS, a type of shock triggers a systemic inflammatory response, after which the body's homeostasis fails. In MODS, homeostasis fails and medical intervention is needed. Test-Taking Tip: Identify option components as correct or incorrect. This may help you identify a wrong answer. Text Reference - p. 1649

The nurse is providing care for an older adult patient who is experiencing low partial pressure of oxygen in arterial blood (PaO2) as a result of worsening left-sided pneumonia. Which intervention should the nurse use to help the patient mobilize secretions? 1 Augmented coughing or huff coughing 2 Positioning the patient side-lying on the left side 3 Frequent and aggressive nasopharyngeal suctioning 4 Application of noninvasive positive pressure ventilation (NIPPV)

1 Augmented coughing or huff coughing Augmented coughing and huff coughing techniques may aid the patient in the mobilization of secretions. If positioned side-lying, the patient should be positioned on the right side (good lung down) for improved perfusion and ventilation. Suctioning may be indicated but always should be performed cautiously because of the risk of hypoxia. NIPPV is inappropriate in the treatment of patients with excessive secretions.

The nurse is caring for a patient receiving mechanical ventilation with high levels of positive end-expiratory pressure (PEEP). What complication should the nurse monitor for in this patient? 1 Barotrauma 2 Oxygen toxicity 3 Pneumoperitoneum 4 Ventilator-associated pneumonia (VAP)

1 Barotrauma A high level of positive end-expiratory pressure (PEEP) leads to barotrauma due to the extreme inspiratory pressure. A patient is at risk for oxygen toxicity when there are respiratory complications caused by oxygen overdose. Pneumoperitoneum is a gastrointestinal complication. Prolonged mechanical ventilation also causes respiratory complication, such as ventilator-associated pneumonia (VAP).

Which medication helps to decrease heart rate and improve cardiac output in the patient with respiratory failure and atrial fibrillation? 1 Diltiazem 2 Nitroglycerin 3 Metaproterenol 4 Methylprednisolone

1 Diltiazem Diltiazem is a calcium channel blocker and potent vasodilator. It increases the blood flow through the arteries and decreases the heart rate. Calcium channel blockers reduce blood pressure and increase cardiac output. Administration of nitroglycerin decreases pulmonary congestion caused by heart failure. Metaproterenol is a bronchodilator. It improves breathing by relaxing the muscles in the airways. Methylprednisolone is administered in conjunction with bronchodilators to treat bronchospasm and asthma.

What are the clinical manifestations of respiratory failure associated with hypoxemia? Select all that apply. 1 Fatigue 2 Confusion 3 Restlessness 4 Muscle weakness 5 Morning headache

1 Fatigue 2 Confusion 3 Restlessness Hypoxemia refers to the decrease in arterial oxygen and may manifest as fatigue, confusion, and restlessness. In hypercapnic respiratory failure, the neuromuscular conditions are affected, resulting in muscle weakness or paralysis. The patient experiences deep tendon reflexes and tremors, as well as seizures at a later stage. Hypercapnia also causes cerebral vasodilation, increased cerebral blood flow, and a mild increase in intracranial pressure that produces a headache.

Which statement describes anatomic pulmonary shunt? 1 It occurs when there is a mismatch of ventilation to perfusion. 2 It occurs when the partial pressure of oxygen falls sufficiently. 3 It occurs when ventilatory demand exceeds ventilatory supply. 4 It occurs when gas exchange across the alveolar-capillary interface is compromised.

1 It occurs when there is a mismatch of ventilation to perfusion. Pulmonary shunt is an extreme ventilation to perfusion (V/Q) mismatch, such as when parts of the lungs are perfused with blood but not ventilated with air. Hypoxia occurs when the partial pressure of oxygen (PaO2) falls sufficiently to cause signs and symptoms of inadequate oxygenation. In cases of hypercapnia, the ventilatory demand exceeds ventilatory supply and PaCO2 cannot be sustained within normal limits. During diffusion limitation, there is gas exchange across the alveolar-capillary interface. The gas exchange is compromised by a process that thickens, damages, or destroys the alveolar membrane or affects blood flow through the pulmonary capillaries.

The health care provider has prescribed a corticosteroid for a patient with acute asthma. What nursing actions are appropriate during and after administration of this medication? 1 Monitor potassium levels 2 Monitor for cardiac ischemia 3 Observe for anxiety and restlessness 4 Administer via inhalation for fast results

1 Monitor potassium levels Potassium levels of patients on corticosteroids should always be monitored because corticosteroid administration can worsen hypokalemia caused by usage of diuretics. When consumed by inhalation, corticosteriods require at least four to five days for optimum therapeutic effects. However, when administered intravenously in cases of acute asthma, corticosteriods speed up the resolution of airway inflammation and edema. Anxiety and restlessness result from hypoxia. Fear caused by the inability to breathe and a sense of loss of control, not the medication in itself, may increase anxiety. Patients being treated for bronchospasm run the risk of high cardiac ischemia with prolonged use of a beta-adrenergic drug.

A patient diagnosed with acute respiratory distress syndrome is being mechanically ventilated with 12 cm of positive end-expiratory pressure (PEEP). Upon assessment, the nurse notes deterioration of vital signs and absent breath sounds in the right lung field. What is the most likely cause for this finding? 1 Pneumothorax 2 Decreased cardiac output 3 Deterioration of the disease 4 Obstructed endotracheal tube

1 Pneumothorax A complication of PEEP may be a pneumothorax as a result of overdistention of the alveoli. If deterioration of the disease were the cause, both lung sounds would be decreased equally. Decreased cardiac output would affect vital signs, but not breath sounds. An obstructed endotracheal tube would affect both lung fields.

The intensive care unit nurse is caring for a patient who is ventilated mechanically. To prevent sepsis in this patient, which nursing intervention does the nurse include in the plan of care? 1 Provide oral care every two to four hours. 2 Turn patient from side to side every eight hours. 3 Position patient in a supine position every two hours. 4 Use clean gloves when suctioning the endotracheal tube.

1 Providing oral care every two to four hours is correct, because research has found that the oral flora of critically ill patients are predominately gram-negative organisms that can potentially cause ventilator-associated pneumonia. Oral care will help reduce the organisms. Turning the patient from side to side every eight hours is incorrect, because patients need to be turned at least every two hours to prevent accumulation of mucus, which could lead to pneumonia. Positioning the patient in a supine position is incorrect, because patients should have the head of the bed elevated during mechanical ventilation. The nurse should use sterile gloves when conducting endotracheal suctioning of the patient. Test-Taking Tip: Never leave a question unanswered. Even if answering is no more than an educated guess on your part, go ahead and mark an answer. You might be right, but if you leave it blank, you will certainly be wrong and lose precious points. Text Reference - p. 1648

The nurse reviews the medical record of a patient with pneumonia and notes that the patient has hypotension, hypothermia, leukocytosis, and hypoxemia. What should the nurse infer from these findings? 1 The patient has septic shock. 2 The patient has neurogenic shock. 3 The patient has cardiogenic shock. 4 The patient has hypovolemic shock.

1 Septic shock is most commonly found in the patient having gram-negative bacterial infections, such as pneumonia. Because it is characterized by hypertension, hypothermia, leukocytosis, and hypoxemia in patients with infections, the nurse concludes that the patient has septic shock. Neurogenic shock is most commonly seen in the patient who has an injury. Cardiogenic shock is caused by cardiovascular disorders, such as a myocardial infarction and cardiomyopathy. Hypovolemic shock is caused by hemorrhage or trauma. Test-Taking Tip: Multiple-choice questions can be challenging, because students think that they will recognize the right answer when they see it or that the right answer will somehow stand out from the other choices. This is a dangerous misconception. The more carefully the question is constructed, the more each of the choices will seem like the correct response. Text Reference - p. 1637

The primary health care provider prescribes antibiotics and vasopressors for a patient. Which type of shock does the nurse expect to be treating? 1 Septic shock 2 Cardiogenic shock 3 Neurogenic shock 4 Anaphylactic shock

1 Septic shock occurs in response to infection. Therefore, antibiotics are prescribed for a patient with septic shock. Cardiogenic shock occurs when systolic or diastolic function of the heart is impaired. Sympathomimetic drugs are used for the treatment of cardiogenic shock. Injury to the spinal cord at the fifth thoracic vertebra or above causes neurogenic shock. Vasconstricting medications are prescribed to prevent vasodilation for a patient in septic shock. Anaphylactic shock is a life-threatening allergic reaction to a sensitizing substance. Antihistamines, bronchodilators, and corticosteroids are used in the treatment of anaphylactic shock. Test-Taking Tip: Do not worry if you select the same numbered answer repeatedly, because there usually is no pattern to the answers. Text Reference - p. 1645

A patient is showing signs of anaphylactic shock from an insect sting. Which primary health care provider's prescription does the nurse implement first? 1 Epinephrine 1:1000, 0.5 mg subcutaneous (SQ) 2 Normal saline intravenous (IV) to run at 150 mL/hr 3 Diphenhydramine 50 mg IV 4 Oxygen via nasal cannula at 3 L

1 The patient in anaphylaxis experiences bronchial spasm and constriction. The administration of epinephrine is necessary to reverse this process and facilitate an open airway. Although administering normal saline, diphenhydramine, and oxygen are appropriate, they must be done after an airway has been established. Text Reference - p. 1646

The nurse concludes that a patient is experiencing severe respiratory distress based on what assessment finding? 1 The patient sits in tripod position. 2 The patient reports difficulty sleeping. 3 The patient walks restlessly in the room. 4 The patient uses long sentences when conversing.

1 The patient sits in tripod position. A patient with severe respiratory distress most commonly uses the tripod position to help decrease the work of breathing and reduces respiratory distress. The patient with severe respiratory distress has severe dyspnea; therefore, the patient would only be able to speak two to three words. The patient with severe respiratory distress would experience shortness of breath and may not be able to walk. Difficulty in sleeping can have many causes and does not necessarily indicate respiratory distress.

The intensive care unit (ICU) nurse is providing care to a patient requiring IV polypharmacy. Which prescriptions is this patient receiving? Select all that apply. 1 Sedation 2 Thrombolytics 3 Vasopressor titration 4 Mechanical ventilation 5 Hemodynamic monitoring

1, 2, 3 ICU patients often require intensive and complicated nursing support related to the use of IV polypharmacy and advanced technology. Prescriptions that are classified as IV polypharmacy include sedation, thrombolytics, and vasopressor titration. Mechanical ventilation and hemodynamic monitoring are advanced technology prescriptions. Text Reference - p. 1600

The nurse is caring for a patient receiving intraaortic balloon therapy. Which nursing interventions are appropriate for this patient? Select all that apply. 1 Monitoring oxygen deprivation levels 2 Monitoring hemodynamic parameters 3 Monitoring neurovascular complications 4 Including spinach in the patient's diet 5 Providing warfarin therapy

1, 2, 3 Intraaortic balloon therapy destroys platelets and may cause thrombocytopenia. Monitoring hemodynamic parameters helps estimate the loss of platelets and development of thrombus. Enlarged blood clots reduce blood flow to the tissues and result in hypoxemia. Therefore, measurement of oxygen deprivation levels assesses for thrombus formation in the patient. Peripheral neurovascular damage is generally observed as a result of intraaortic balloon therapy and should be monitored. Spinach is rich in vitamin K and increases the formation of clotting factors. Warfarin is an anticoagulant used to prevent future development of blood clots. Spinach and warfarin should be avoided for patients with thrombocytopenia. Text Reference - p. 1611

When examining a patient with cardiogenic shock, which signs of peripheral hypoperfusion does the nurse expect? Select all that apply. 1 Cyanosis 2 Cold skin 3 Weak pulse 4 Bradycardia 5 Hypertension

1, 2, 3 Cyanosis, cold skin, and a weak pulse are the signs of peripheral hypoperfusion in cardiogenic shock. Bradycardia and hypertension are not seen in cardiogenic shock; instead, tachycardia and low blood pressure are noted. Text Reference - p. 1633

Which are the reasons why caregivers play a valuable role in the intensive care unit (ICU) patient's recovery? Select all that apply. 1 They provide loving support for the patient. 2 They provide a link to the patient's personal life. 3 They help the patient with activities of daily living. 4 They function as the patient's decision-maker during the hospitalization. 5 They make the financial decision for the patient during the hospitalization.

1, 2, 3, 4 Caregivers play a valuable role in the intensive care unit (ICU) patient's recovery because they provide the patient with support. They are also a link to the patient's personal life, help the patient with activities of daily living, and function as the decision maker during the hospitalization. Although the caregiver may make financial decision for the patient during the hospitalization, this is not an important role the caregiver plays in the patient's recovery. Text Reference - p. 1601

The nurse is evaluating a patient receiving intraaortic balloon pump (IABP) therapy. Which findings indicate that the pump is improving the patient's health status? Select all that apply. 1 Warm and dry skin 2 Urine output 50 mL/hr 3 Breath sounds clear bilaterally 4 Blood pressure 168/88 mm Hg 5 Oriented to person, place, and time

1, 2, 3, 5 Hemodynamic effects of intraaortic balloon pump therapy (IABP) include increased stroke volume leading to warm skin and increased urine output. The decrease in afterload improves breath sounds. Improved stroke volume also improves mentation. The pump has no direct effect on blood pressure regulation. Text Reference - p. 1611

Which types of shock may cause reduced urinary output in a patient? Select all that apply. 1 Septic shock 2 Hypovolemic shock 3 Obstructive shock 4 Neurogenic shock 5 Anaphylactic shock 6 Cardiogenic shock.

1, 2, 3, 6 Decreased urine output is a clinical manifestation of septic, hypovolemic, obstructive and cardiogenic shock. Neurogenic shock is associated with bladder dysfunction. Anaphylactic shock is associated with urinary incontinence. Test-Taking Tip: Make certain that the answer you select is reasonable and obtainable under ordinary circumstances and that the action can be carried out in the given situation. Text Reference - p. 1635

A patient is placed on a ventilator for assisted ventilation. What precautions should a nurse take to prevent the patient from ventilator-assisted pneumonia (VAP)? Select all that apply. 1 Wash hands before and after suctioning. 2 Use an endotracheal (ET) tube with a dorsal lumen above the cuff. 3 Change the patient's ventilator circuit tubing every two to three hours. 4 Wear gloves when in contact with the patient, and change gloves between activities. 5 Maintain the head-of-bed elevation at a minimum of 90 degrees unless medically contraindicated.

1, 2, 4 Patients have a higher risk for hospital-acquired pneumonia when they require mechanical ventilation. This is because the ET or tracheostomy tube bypasses the normal upper airway defenses. Additionally, poor nutritional state, immobility, and the underlying disease process make the patient more prone to infection. VAP is pneumonia that occurs 48 hours or more post-ET intubation. To prevent VAP, the health care team should strictly wash their hands before and after suctioning. An ET tube with a dorsal lumen above the cuff should be used to allow continuous suctioning of secretions in the subglottic area. Gloves should be worn whenever the nurse is in contact with the patient, and the nurse should change them frequently between activities to avoid cross-infection. If the ventilator circuit tubing is changed frequently, there is more risk of exposing the patient to various infections. Therefore, there should be no routine changes in the patient's ventilator circuit tubing. In addition to this, the head-of-bed should be elevated at a minimum of 30 to 45 degrees, unless medically contraindicated, to prevent pooling of secretions and facilitate suctioning. Text Reference - p. 1623

The health care provider prescribes a dose of dobutamine for a patient in cardiogenic shock due to myocardial infarction. What appropriate actions should the nurse perform for safely administering the medication? Select all that apply. 1 Monitor heart rate and blood pressure. 2 Stop infusion if tachydysrhythmias develop. 3 Always administer with sodium bicarbonate. 4 Administer through a central line. 5 Use a glass bottle for infusion.

1, 2, 4 Doubutamine is a sympathomimetic medication. When used in therapy with dobutamine, the patient's heart rate and blood pressure should be continuously monitored, as they may worsen hypotension, requiring the addition of a vasopressor. The infusion should be stopped if tachydysrhythmias develop. The administration through a central line is recommended, because infiltration leads to tissue sloughing. The drug should not be administered with sodium bicarbonate, because it can get deactivated. Because dobutamine is not adsorbed in plastic containers, it is not necessary to administer the drug in glass bottles. Test-Taking Tip: Key words or phrases in the stem of the question such as first, primary, early, or best are important. Similarly, words such as only, always, never, and all in the alternatives are frequently evidence of a wrong response. As in life, no real absolutes exist in nursing; however, every rule has its exceptions, so answer with care. Text Reference - p. 1643

A patient with severe respiratory distress is brought to the medical facility. The health care provider prescribes rapid-sequence intubation (RSI) to be done. What information should the nurse include when explaining the procedure to the family members? Select all that apply. 1 A sedative and a paralytic medication is administered so that the patient sleeps and does not feel the pain. 2 After intubation, 100% oxygen is given to the patient, and placement of the tube is confirmed. 3 After giving paralytic, an opening is made in the throat through which a tube is introduced. 4 After establishing the opening in the throat, the tube is placed and dressed properly. 5 A tube will be introduced in the patient's throat through the mouth so that ventilation can be established.

1, 2, 5 In RSI, a sedative and paralytic are administered to the patient. Usually a sedative-hypnotic-amnesic agent like midazolam is used to make the patient unconscious. A rapid-onset opioid like fentanyl is also given to blunt the pain of the procedure. A paralytic drug like succinylcholine is then given to produce skeletal muscle paralysis. Before this, the patient is 100% oxygenated. After intubation, the patient is again 100% oxygenated, and tube placement is confirmed. Auscultation of the chest bilaterally and x-ray are some of the methods of confirming the placement of the tube. During intubation, the endotracheal (ET) tube is inserted through the nose or mouth. While performing RSI, no opening is made in the throat. However, an opening in the throat is made in a procedure called tracheostomy. Text Reference - p. 1614

A nurse is caring for a patient in ICU who is taking sedatives. What are the steps that a nurse should take in order to prevent delirium in this patient? Select all that apply. 1 Keep the noise in the ICU to a minimum. 2 Use clocks and a calendar to keep the patient oriented. 3 Ensure that there is minimal communication with the patient. 4 Give regular sponge baths to the patient, and monitor the urinary output. 5 Carry out frequent assessment for delirium by using the Confusion Assessment Method.

1, 2, 5 It is extremely essential to monitor all ICU patients and prevent delirium. Sensory overload can lead to patient distress and anxiety. The nurse should limit noise in the ICU and help the patient to understand that some noises in the ICU cannot be prevented, for example, beeping of a cardiac monitor. The nurse can also limit noise levels by muting phones, setting alarms based on the patient's condition, and reducing unnecessary alarms. The use of clocks and calendars can help orient the patient to time and date. Regular assessment should be carried out using tools like the Confusion Assessment Method for the ICU and the Intensive Care Delirium Screening Checklist. Seeing a familiar face may make the patient comfortable; therefore, the presence of a caregiver is important. Giving regular sponge baths helps to maintain hygiene but doesn't affect delirium directly. Text Reference - p. 1601

When examining a patient with septic shock, what symptoms would the nurse expect to find? Select all that apply. 1 Paralytic ileus 2 Gastrointestinal (GI) bleeding 3 Pulsus paradoxus 4 Distended jugular vein 5 Decreased urinary output

1, 2, 5 Patients suffering from septic shock may experience decreased tissue perfusion, which may result in a paralytic ileus, GI bleeding, and decreased urinary output. Pulsus paradoxus and jugular vein distension are found in obstructive shock and are mainly the result of compromised hemodynamics. Text Reference - p. 1637

A patient is prescribed milrinone. What effects on the patient's hemodynamic parameters should the nurse expect? Select all that apply. 1 Decreased preload 2 Increased heart rate 3 Decreased afterload 4 Increased blood pressure 5 Decreased cardiac output

1, 3 Milrinone is a vasodilator. Vasodilation decreases preload and afterload. This medication does not directly affect the heart rate. Vasodilators cause the blood pressure to decrease. Vasodilators will improve cardiac output. Test-Taking Tip: Be alert for details about what you are being asked to do. In this question type, you are asked to select all options that apply to a given situation or patient. All options likely relate to the situation, but only some of the options may relate directly to the situation. Text Reference - p. 1604

The nurse is caring for a patient with an artificial airway. Which nursing interventions ensure endotracheal tube patency? Select all that apply. 1 Provide adequate hydration 2 Provide gluten rich food 3 Promote postural drainage 4 Provide supplemental humidification 5 Instilling normal saline into the endotracheal tube

1, 3, 4 Administering IV fluids helps thin respiratory secretions and facilitates suctioning. Postural drainage and percussion every two hours helps move secretions into larger airways and promotes the removal through suctioning. Supplemental humidification helps thin secretions and promotes suctioning. The patient has an endotracheal tube and will not be given an oral diet. Instilling normal saline into the endotracheal tube can cause asphyxia. Text Reference - p. 1617

A patient is advised to have a pulmonary artery (PA) catheter inserted for pulmonary artery pressure monitoring. What precautions are necessary prior to insertion of the catheter? Select all that apply. 1 Place the patient in the supine and flat position. 2 Position the patient sitting and with head turned laterally. 3 Explain the procedure to the patient and get consent. 4 Note the patient's electrolyte levels and oxygenation and coagulation status. 5 Do not cover the catheter insertion site with any dressings.

1, 3, 4 Before PA catheter insertion, the patient is positioned supine and flat. The procedure is explained to the patient, and informed consent is obtained. The patient's electrolyte, acid-base, oxygenation, and coagulation status are noted. Imbalances such as hypokalemia, hypomagnesemia, hypoxemia, or acidosis can make the heart more irritable and increase the risk of ventricular dysrhythmia during catheter insertion. Coagulopathy increases the risk of hemorrhage. The procedure is never performed in a sitting position. The PA catheter is inserted through a sheath percutaneously into the internal jugular, subclavian, antecubital, or femoral vein using surgical asepsis. The insertion sites have to be dressed with occlusive dressings. Text Reference - p. 1608

A nurse is suctioning a patient. Which signs indicate that the patient is not tolerating suctioning? Select all that apply. 1 Decreased SpO2 2 Absence of coughing 3 Increased blood pressure (BP) 4 Development of dysrhythmias 5 Shivering and convulsions of the entire body

1, 3, 4 It is extremely important for a nurse to closely assess the patient before, during, and after the suctioning procedure. If the patient is unable to tolerate suctioning, stop the procedure and hyperoxygenate until equilibration occurs before attempting next suction pass. Decreased SpO2, increased or decreased BP, and development of dysrhythmias are indicators that the patient is not tolerating suction. Sustained coughing rather than absence of coughing also indicates that the patient is not tolerating suctioning. Presence of shivering and convulsions is not related to suctioning. Text Reference - p. 1616

An endotracheal (ET) tube is placed for a patient. What interventions should the nurse perform to maintain oral hygiene and care? Select all that apply. 1 Reposition and retape the ET tube at least every 24 hours. 2 Keep the buccal cavity and nasal cavity anesthetized. 3 Replace the bite block and reconfirm proper cuff inflation and tube placement regularly. 4 Provide oral hygiene with repositioning of the ET tube to the opposite side of the mouth after care. 5 Keep the patient restrained to avoid dislodging the tube.

1, 3, 4 Whenever a patient is intubated, repositioning and retaping the tube after 24 hours are essential. For the orally intubated patient, remove the bite block and the old tape or ties. Provide oral hygiene, and then reposition the ET tube to the opposite side of the mouth. There is no need to give anesthesia for maintaining oral care. There is no point in physically restraining the patient, because it will cause discomfort and anxiety. Text Reference - p. 1617

A patient is being prepared for intubation using a nasal intubation technique. What absolute contraindications for nasal intubation should the nurse be aware of? Select all that apply. 1 Recent cranial surgery 2 Suspected spine injury 3 Fracture of facial bones 4 Deviated nasal septum 5 Fracture of the base of the skull

1, 3, 5 Nasal intubation is a blind procedure. There are chances that the tube may be misdirected. It may cause complications if there was a recent cranial surgery and fracture of the facial bones or the base of the skull. Suspected spinal fracture is a contraindication for oral intubation because it requires some movement of the neck and head. A deviated nasal septum can cause some difficulty in nasal intubation, although it is not a contraindication. STUDY TIP: Begin studying by setting goals. Make sure they are realistic. A goal of scoring 100% on all exams is not realistic, but scoring an 85% may be a better goal. Text Reference - p. 1614

A patient in ICU has been intubated for the relief of airway obstruction. What nursing actions should be performed to prevent complications after intubation? Select all that apply. 1 Obtain a chest x-ray to confirm the placement. 2 Obtain a computed tomography (CT) scan to note the placement. 3 Immediately catheterize the patient and check for urine output. 4 Auscultate lungs bilaterally and also epigastrium for breath sounds. 5 Use an end-tidal carbo

1, 4, 5 Following an intubation, it is important to confirm the placement of the endotracheal (ET) tube. This confirmation is obtained by x-ray after visualizing the ET tube correctly placed in the trachea. Auscultating lungs for breath sounds confirms that air is going into the lungs and not in the stomach. If the sounds are heard over the epigastrium, it indicates that the ET tube has gone in the stomach. Presence of carbon dioxide in exhaled air also confirms that the tube has gone into the lungs, and the breathing effort is normal. In this case, a CT scan is redundant. However, an x-ray is sufficient to confirm the placement of the ET tube. The patient may require a urinary catheter, but it is not an immediate intervention and can be done after intubation. Text Reference - p. 1614

A patient admitted to the ICU is being intubated. What are the steps that a nurse should ensure for a safe intubation? Select all that apply. 1 Remove the dentures of the patient. 2 Ensure that the patient is not wearing any metallic objects. 3 Oxygenate the patient using a bag-valve-mask (BVM) and 95% oxygen before the procedure. 4 Inform the patient that brief restraint will be necessary. 5 Explain the procedure to the patient and also the patient's role in the procedure.

1, 4, 5 It is important to remove the dentures of the patient during the process of oral intubation because the dentures can obstruct the airway. Inform the patient that brief restraint will be necessary for safety purposes. It is also necessary to brief the patient about the procedure to avoid any type of resistance during intubation. Metallic objects on the body do not interfere with the procedure of intubation, and, therefore, need not be removed. Before intubating, it is extremely important to preoxygenate the patient with 100% oxygen. This is because the patient will not get any oxygen supply during intubation for a short period. STUDY TIP: Avoid planning other activities that will add stress to your life between now and the time you take the licensure examination. Enough will happen spontaneously; do not plan to add to it. Text Reference - p. 1614

A nurse caring for a patient with multiple organ dysfunction syndrome understands that the patient may be at increased risk of bleeding. What nursing interventions should the nurse perform to manage this patient? Select all that apply. 1 Observe bleeding sites. 2 Decrease fluid intake. 3 Provide enteral feedings. 4 Administer platelets and clotting factors. 5 Minimize traumatic interventions.

1, 4, 5 The patient with multiple organ dysfunction syndrome is at a risk of bleeding due to increased bleeding time, thrombocytopenia, and dysfunctional clotting process. The nursing interventions should be aimed at preventing potential bleeding and replacing factors being lost. The patient should be observed for frank or occult bleeding from potential sites. The factors like platelets and clotting factors should be replaced if deficient. Traumatic interventions such as intramuscular injections or multiple venipunctures should be avoided. Decreasing the fluid intake and providing enteral feedings will not help in minimizing hematologic complications. Text Reference - p. 1650

A patient is treated in the emergency department (ED) for shock of unknown etiology. The first action by the nurse should be to a. check the blood pressure. b. obtain an oxygen saturation. c. attach a cardiac monitor. d. check level of consciousness.

1.Correct Answer: B Rationale: The initial actions of the nurse are focused on the ABCs, and assessing the airway and ventilation is necessary. The other assessments should be accomplished as rapidly as possible after the oxygen saturation is determined and addressed.

A client is admitted to the emergency department after a motorcycle accident. Upon assessment, the client's vital signs reveal blood pressure of 80/60 mm Hg and heart rate of 145 beats per minute. The client's skin is cool and clammy. Which medical order for this client will the nurse complete first?

100% oxygen via a nonrebreather mask pg. 292

An endotracheal (ET) tube is inserted in a patient. The nurse inflates the cuff to stabilize the tube. How much cuff pressure should be maintained to keep it inflated and ensure adequate tracheal perfusion? 1 10-15 cm H2O 2 20-25 cm H2O 3 30-35 cm H2O 4 40-45 cm H2O

2 20-25 cm H2O To ensure adequate tracheal perfusion, the nurse should maintain cuff pressure at 20 to 25 cm H2O. Excess cuff pressure can damage the tracheal mucosa. Lesser cuff pressure may cause the ET tube to become destabilized and extubate.

An older adult patient reports having used an "iron lung" after contracting polio as a child. The nurse knows this patient is referring to which type of mechanical ventilation? 1 Positive pressure 2 Negative pressure 3 Volume ventilation 4 Pressure ventilation

2 Negative pressure The "iron lung" was the first form of negative pressure ventilation, developed during the polio epidemic. Negative pressure uses the chambers encasing the chest and surrounding it with intermittent negative pressure; expiration is passive, and the negative pressure is delivered by noninvasive measures. Pressure ventilation means the peak inspiratory pressure is predetermined and the tidal volume delivered varies based on the patient. Volume ventilation has a predetermined tidal volume delivered with each inspiration and the pressure varies based on the patient. Positive pressure ventilation is the primary method used with acutely ill patients, where air is pushed into the lungs during inspiration under positive pressure.

The nurse is caring for a group of patient's in the intensive care unit. Which patient is a candidate for bilevel positive airway pressure (BiPAP)? 1 Patient with shock 2 Patient with sleep apnea 3 Patient with altered mental status 4 Patient with increased airway secretions

2 Patient with sleep apnea Bilevel positive airway pressure (BiPAP) is used for patients with sleep apnea. Patients with shock, altered mental status and/or increased airway secretions are not candidates for BiPAP because of the risk of aspiration and the inability to remove the mask.

The nurse is assessing a patient placed on mechanical ventilation and hears breath sounds on the right but not on the left side of the chest. What common complication should the nurse immediately notify the health care provider about? 1 Hypertension 2 Pneumothorax 3 Electrolyte imbalance 4 Increased cardiac output

2 Pneumothorax Mechanical ventilation can cause pneumothorax as a result of excessive pressure applied to lung tissue. Hypertension is not a direct complication; however, a patient undergoing mechanical ventilation may be anxious and fearful, resulting in high blood pressure; sedation should be considered in this event. Electrolyte imbalance is not a related complication. Mechanical ventilation does increase intrathoracic pressure, which may then increase cardiac output, causing a beneficial secondary effect.

While evaluating a mechanically ventilated patient, the nurse notes that the auto-PEEP has been activated. Which mode of mechanical ventilation does the nurse suspect? 1 Volume mode 2 Pressure mode 3 Continuous positive airway pressure 4 Positive end-expiratory pressure mode

2 Pressure mode The auto-PEEP is a pressure mode that provides a pressure-limited breath delivered at a set rate that may permit spontaneous breathing. The positive end-expiratory pressure (PEEP) mode is a ventilator maneuver in which positive pressure is applied to the airway during exhalation. Continuous positive airway pressure (CPAP) is similar to PEEP, but the pressure is delivered continuously during spontaneous breathing, preventing the patient's airway pressure from falling to zero. Volume modes require that rate, tidal volume, inspiratory time, sensitivity, and/or PEEP are set for the patient

The nurse working in the intensive care unit (ICU) is taking care of a patient on a mechanical ventilator who had a motor vehicle accident two weeks ago. What does the nurse know about this situation? 1 The patient has severe hypoxia due to acute respiratory failure. 2 The ventilator will support the patient until he or she can breathe on his or her own. 3 The patient suffered from a chronic pulmonary disease before the accident. 4 The patient will be on long-term ventilation until the family decides to withdraw ventilator support.

2 The ventilator will support the patient until he or she can breathe on his or her own. Mechanical ventilation is not curative. It is a means of supporting patients until they recover the ability to breathe independently. The decision to use, withhold, or withdraw mechanical ventilation will be made carefully, involving the agency's ethics committee for assistance. The patient's medical history or diagnosis is not known; the nurse does not know if the patient suffered from chronic pulmonary disease or if he has severe hypoxia.

The nurse working in a critical care unit understands that tidal volume is an important setting in a mechanical ventilator. Which statement appropriately describes tidal volume? 1 Number of breaths the ventilator delivers per minute 2 Volume of gas delivered to patient during each ventilator breath 3 Positive pressure used to augment patient's inspiratory pressure 4 Positive pressure applied at the end of expiration of ventilator breaths

2 Volume of gas delivered to patient during each ventilator breath Tidal volume is the volume of gas delivered to a patient during each ventilator breath. The number of breaths the ventilator delivers per minute is called the respiratory rate. The positive pressure used to augment the patient's inspiratory pressure is called pressure support. The positive pressure applied at the end of expiration of ventilator breaths is called positive end-expiratory pressure.

A patient presents to the emergency department (ED) in a state of shock. On assessment, the nurse finds that the patient is cyanotic and has crackles on auscultation of the lungs. As which type of shock will the nurse classify this? 1 Neurogenic shock 2 Cardiogenic shock 3 Hypovolemic shock 4 Anaphylactic shock

2 A patient with cardiogenic shock shows peripheral hypoperfusion presenting as cyanosis and has crackles on auscultation of the lungs due to pulmonary congestion. In neurogenic shock, the patient demonstrates symptoms related to the injury such as hypotension and bradycardia. The patient in hypovolemic shock may experience tachycardia as a late sign. In anaphylactic shock, the patient may experience wheezing and stridor. Text Reference - p. 1633

What is a manifestation of the irreversible stage of shock? 1 Delirium 2 Areflexia 3 Restlessness 4 Alterations in the level of consciousness

2 Areflexia or loss of reflexes is a manifestation of the irreversible stage of shock. The progressive state of shock is associated with delirium. Restlessness and altered levels of consciousness indicate that the patient is in the compensatory stage of shock. Test-Taking Tip: Identifying content and what is being asked about that content is critical to your choosing the correct response. Be alert for words in the stem of the item that are the same or similar in nature to those in one or two of the options. Text Reference - p. 1639

A patient in shock is receiving 0.9 % NaCl (normal saline solution-NSS). Which nursing intervention is appropriate for this patient? 1 Monitor the patient's vital signs 2 Monitor for the signs of circulatory overload 3 Monitor for signs of hypernatremia in the patient 4 Monitor for allergic reactions and acute renal failure

2 Circulatory overload occurs due to fluid overload. Continuous infusion of 0.9% NaCl increases the fluid volume in the body and may cause circulatory overload. The patient's vital signs must be checked during transfusion of blood or blood products because they could cause an infection or an allergic reaction. Hypernatremia occurs when the patient is on 1.8%, 3%, and 5% NaCl infusions. Infusion of dextran-40 has a tendency to precipitate allergic reactions and acute renal failure. Test-Taking Tip: Do not worry if you select the same numbered answer repeatedly, because there usually is no pattern to the answers. Text Reference - p. 1642

When caring for a critically ill patient who is being mechanically ventilated, the nurse will astutely monitor for which clinical manifestation of multiple organ dysfunction syndrome (MODS)? 1 Increased serum albumin 2 Decreased respiratory compliance 3 Increased gastrointestinal (GI) motility 4 Decreased blood urea nitrogen (BUN)/creatinine ratio

2 Clinical manifestations of MODS include symptoms of respiratory distress, signs and symptoms of decreased renal perfusion, decreased serum albumin and prealbumin, decreased GI motility, acute neurologic changes, myocardial dysfunction, disseminated intravascular coagulation (DIC), and changes in glucose metabolism. Serum albumin is not increased, GI motility decreases in MODS, and the BUN/Creatinine ratio likely will increase. Text Reference - p. 1649

Which drug helps manage renal manifestations in a patient with systemic inflammatory response syndrome (SIRS) and multiple organ dysfunction syndrome (MODS)? 1 Sucralfate 2 Furosemide 3 Omeprazole 4 Acetaminophen

2 Furosemide is a loop diuretic that helps to manage renal manifestations in a patient with systemic inflammatory response syndrome (SIRS) and multiple organ dysfunction syndrome (MODS). Sucrafate is administered for prophylaxis against stress ulcers, which are gastrointestinal manifestations of SIRS and MODS. Omeprazole is a proton pump inhibitor that has the same action. Acetaminophen is an antipyretic drug given as an acute intervention to manage fevers in patients who are in shock. Text Reference - p. 1651

What is included in the subjective data the nurse obtains after the diagnosis of a patient with respiratory failure? 1 Vital signs 2 Health history 3 Neurologic findings 4 Diagnostic test results

2 Health history The nursing diagnosis for a patient includes subjective and objective data. Subjective data are inclusive of health information like health history, medications, surgery, or other treatments. Vital signs are objective rather than subjective data. Diagnostic tests are performed prior to a diagnosis. A neurologic examination assesses for any slurred speech, restlessness, or delirium.

Which type of shock is associated with hyperglycemia, presence of pulmonary infiltrates in chest x-ray and increased levels of blood urea nitrogen (BUN)? 1 Septic 2 Cardiogenic 3 Obstructive 4 Hypovolemic

2 Increased blood levels of glucose, nitrogen, cardiac markers and presence of pulmonary infiltrates are seen in cardiogenic shock. Increased blood levels of lactate, glucose, and positive blood cultures are signs of septic shock. Manifestations of obstructive shock are specific to the area or organ of obstruction. Electrolyte imbalances and decreased hemoglobin and hematocrit are seen in hypovolemic shock. Test-Taking Tip: Identify option components as correct or incorrect. This may help you identify a wrong answer. Text Reference - p. 1635

Which type of shock is associated with bradycardia? 1 Septic shock 2 Neurogenic shock 3 Hypovolemic shock 4 Anaphylactic shock

2 Neurogenic shock is associated with bradycardia. Myocardial dysfunction and changes in body temperature are signs of septic shock. Reduction in preload, capillary refill and stroke volume are clinical manifestations of hypovolemic shock. Chest pain is seen in anaphylactic shock. Test-Taking Tip: Make certain that the answer you select is reasonable and obtainable under ordinary circumstances and that the action can be carried out in the given situation. Text Reference - p. 1635

Which type of shock can be treated by minimizing spinal cord trauma with stabilization? 1 Septic shock 2 Neurogenic shock 3 Anaphylactic shock 4 Hypovolemic shock

2 Neurogenic shock is caused by severe injury to the spinal cord and results in loss of sympathetic stimulation of blood vessels. Apart from administering vasoconstrictor agents, minimizing the spinal cord trauma with stabilization is a supporting therapy for neurogenic shock. Septic shock occurs in response to a systemic infection. Obtaining the cultures before starting antibiotics is appropriate care for septic shock. A life-threatening allergic reaction to a sensitizing substance causes anaphylactic shock. Avoiding exposure to allergens is supportive therapy for anaphylactic shock. Excessive loss of intravascular fluid causes hypovolemic shock. Besides restoring fluid volume, correcting the cause of fluid loss is supportive therapy. Test-Taking Tip: If the question asks for an immediate action or response, all of the answers may be correct, so base your selection on identified priorities for action. Text Reference - p. 1645

Which cardiovascular change is commonly found in patients with systemic inflammatory response syndrome (SIRS)? 1 Decrease in heart rate 2 Decrease in capillary refill 3 Decrease in central venous pressure 4 Decrease in pulmonary artery wedge pressure

2 Patients with systemic inflammatory response syndrome (SIRS) have decreased capillary refill. Other cardiovascular changes include increases rather than decreases in heart rate, central venous pressure, and pulmonary artery wedge pressure. Test-Taking Tip: Multiple-choice questions can be challenging, because students think that they will recognize the right answer when they see it or that the right answer will somehow stand out from the other choices. This is a dangerous misconception. The more carefully the question is constructed, the more each of the choices will seem like the correct response. Text Reference - p. 1649

A patient has developed acute respiratory distress syndrome (ARDS) in the intensive care unit. What nursing action will assist in the treatment of this disorder? 1 Effective coughing 2 Positioning strategy 3 Chest physiotherapy 4 Hydration and humidification

2 Positioning strategy Positioning strategy during oxygenation has helped patients with acute respiratory distress syndrome (ARDS) show significant improvement. Effective coughing and adequate hydration and humidification help patients mobilize secretions. Chest physiotherapy is suggested for patients with acute respiratory failure who produce sputum of more than 30 mL per day.

The nurse attending to an older adult patient hears the patient coughing. On auscultation, the nurse finds retained pulmonary secretions in the lungs. What nursing interventions should the nurse perform to mobilize secretions? Select all that apply. 1 Limit fluid intake. 2 Provide humidified air. 3 Provide chest physiotherapy. 4 Advise bed rest for the patient. 5 Encourage the patient to cough.

2 Provide humidified air 3 Provide chest physiotherapy 5 Encourage the patient to cough Retained pulmonary secretions increase the risk of respiratory failure. It is important to mobilize the secretions and clear the airway to facilitate ventilation. Effective coughing is an important measure to move up the secretions and relieve obstruction of the airway. Chest physiotherapy can be used in patients who produce a copious amount of sputum and who have a collapsed lung. Humidified air helps to keep secretions liquefied and eases in coughing them out. The patient should not be limited to bed rest; ambulation should be done when possible, because it helps to expand the lungs and clear the secretions. Fluid intake should not be restricted. Adequate fluid intake is required to prevent the secretions from thickening.

A patient admitted to the hospital after a motor vehicle accident (MVA) is in hypovolemic shock. On examination, the nurse finds that the patient is becoming anxious, and the urine output is decreasing. What appropriate action should the nurse perform? 1 Prepare for administering blood products. 2 Begin crystalloid fluid replacement. 3 Start fluids only if deterioration occurs. 4 Wait for the patient to compensate naturally.

2 When the volume of blood loss is less than 30 percent, crystalloid fluid replacements are performed to reverse tissue dysfunction. Blood products are administered when the blood volume loss is more than 30 percent. The nurse should not wait for deterioration to occur to start the fluid replacement therapy. This is because the body can typically compensate naturally for a blood volume loss up to 15 percent. Test-Taking Tip: Be alert for grammatical inconsistencies. If the response is intended to complete the stem (an incomplete sentence) but makes no grammatical sense to you, it might be a distractor rather than the correct response. Question writers typically try to eliminate these inconsistencies. Text Reference - p. 1633

The nurse wants to provide culturally competent care to patients requiring care in the intensive care unit (ICU). Which actions by the nurse are appropriate? Select all that apply. 1 Prioritizing cultural needs over physiologic needs 2 Asking the patient who he or she wants in the room at the time of death 3 Asking the family about cultural traditions regarding death and dying 4 Assuming that the patient follows cultural customs for the documented ethnicity 5 Telling the family members it is not possible for last rites to be administered

2, 3 Providing culturally competent care to critically ill patients and caregivers is challenging. The nurse who wants to provide culturally competent care to critically ill patients in the intensive care unit (ICU) should ask the patient or family members who is wanted in the room at the time of death. The nurse should also ask the patient or family members about cultural traditions regarding death and dying. The nurse should not prioritize cultural needs over physiologic needs. Often physiologic needs are the priority in the ICU. The nurse should not assume that the patient follows cultural customs for the documented ethnicity. The nurse should not tell the family that last rites are not possible and should advocate for the patient to receive last rites if this is the patient's wish. Text Reference - p. 1602

The nurse reviews medical records of several patient and concludes that which patients are appropriate candidates for parenteral nutrition? Select all that apply. 1 A patient with arthritis 2 A patient with pancreatitis 3 A patient with paralytic ileus 4 A patient with severe diarrhea 5 A patient with hypothyroidism

2, 3, 4 When enteral nutrition is contraindicated in patients, the primary health care provider would prescribe parenteral nutrition to provide adequate nutrients to the patients. Patients with gastrointestinal disorders such as pancreatitis, paralytic ileus, and severe diarrhea will receive parenteral nutrition. Enteral nutrition is not contraindicated in the patient with arthritis and hypothyroidism. Text Reference - p. 1600

A nurse is examining a patient with anaphylactic shock due to an insect bite. What types of skin manifestations would the nurse expect to find? Select all that apply. 1 Pallor 2 Pruritus 3 Flushing 4 Urticaria 5 Cold, clammy skin

2, 3, 4 Insect bites may cause allergic reactions and anaphylactic shock. The skin manifestations may include pruritus, flushing, and urticaria caused by massive vasodilation, release of vasoactive mediators, and an increase in capillary permeability. Pallor and cold, clammy skin changes are usually seen in cardiogenic, hypovolemic, and obstructive shock caused by decreased circulatory volume and tissue perfusion. Text Reference - p. 1636

A patient has received intraaortic balloon pump (IABP) therapy. In this case, what precautions should a nurse take to prevent any infection at the site? Select all that apply. 1 Replace the lines every two to three hours. 2 Cover all insertion sites with occlusive dressings. 3 Use strict aseptic technique line insertion and dressing changes. 4 Send culture swabs from the insertion site regularly. 5 Administer prophylactic antibiotics for the entire course of therapy

2, 3, 5 It is extremely important to follow aseptic measures to avoid infection following an IABP. Covering all the insertion sites with occlusive dressings avoids infections. Aseptic techniques should be followed during insertion and dressing changes to prevent infection. Prophylactic antibiotics prevent infections. Infection doesn't breed in the dressings if aseptic precautions are followed and the dressings are cleaned and replaced regularly. Replacing the lines every two hours is not necessary, but following aseptic conditions is. A culture swab helps to identify the presence of infection but d

While planning the management of oxygen delivery in a patient with shock, what appropriate measures should the nurse undertake? Select all that apply. 1 Encourage the patient to move around to increase lung expansion. 2 Space activities that increase oxygen consumption. 3 Monitor continuously by using a central venous catheter. 4 Space activities that decrease oxygen consumption. 5 Administer supplemental oxygen as prescribed.

2, 3, 5 To optimize oxygen supply and ventilation in a patient suffering from shock, the activities that increase oxygen consumption should be evenly spaced. Mixed venous oxygen saturation should be monitored through a central venous catheter. The patient should not exert energy by excessive moving around because it increases oxygen demand. Text Reference - p. 1641

A patient is being considered for ventricular assist device (VAD) therapy. Which criteria indicate that this patient is an appropriate candidate for implantation of this device? Select all that apply. 1 Body surface area 1.1 m2 2 Placed on the heart transplantation list 3 Diagnosed with alcoholic liver failure 4 Diagnosed with Class IV heart disease 5 Unable to wean from the cardiopulmonary bypass (CPB) machine

2, 4, 5 Appropriate patient selection for ventricular assist device (VAD) includes patients who are waiting for heart transplantation, who are diagnosed with Class IV heart disease, and have failed medical therapy, and who have failed to wean from cardiopulmonary bypass (CPB). Body surface area less than 1.3 m2 is a contraindication for ventricular assist device (VAD) therapy. Liver failure unrelated to a cardiac event is a contraindication for ventricular assist device (VAD) therapy. Text Reference - p. 1612

When managing a patient with shock, which appropriate actions should the nurse take as part of nutritional therapy? Select all that apply. 1 Plan enteral feeding to meet at least 50 percent of calorie requirements. 2 Start enteral nutrition within the first 24 hours. 3 Wait until the patient recovers to start with enteral nutrition. 4 Start a slow continuous drip of small amounts of enteral feedings. 5 Start parenteral nutrition if enteral feedings are contraindicated.

2, 4, 5 Enteral nutrition should ideally begin within the first 24 hours. It is important because it enhances the perfusion of the (gastrointestinal) GI tract and helps to maintain the integrity of the gastrointestinal mucosa. Enteral feeding should be started with a slow continuous drip of small amounts of enteral feedings. If enteral feedings are contraindicated, parenteral feedings can be started. Enteral feeding should be planned to meet at least 80 percent of the total calorie requirements, but if it is not feasible, parenteral feeding should be started. Nutritional therapy should start as early as possible. The nurse should not wait for the patient to recover before starting nutritional therapy. Text Reference - p. 1644

When examining a patient in the progressive stage of shock, which factors related to the gastrointestinal (GI) system should the nurse consider? Select all that apply. 1 Increased motility and peristalsis 2 Increased likelihood of GI ulcers 3 Increased ability to absorb nutrients 4 Increased risk of GI bleeding 5 Increased risk of bacterial migration from the GI tract to the bloodstream.

2, 4, 5 In the progressive stage of shock, the GI system gets affected by prolonged decreased tissue perfusion. As the blood supply to the GI tract is decreased, the normally protective mucosal barrier becomes ischemic. This ischemia predisposes the patient to ulcers and GI bleeding. It also increases the risk of bacterial migration from the GI tract to the blood. The decreased perfusion to the GI tract also results in a decreased ability to absorb nutrients, decreased motility, and slowed peristalsis. Text Reference - p. 1639

A patient is placed on mechanical ventilation. A nurse notices that the alarm for the high-pressure limit has been set off. What are the possible conditions that could give rise to this alarm? Select all that apply. 1 Oversedation 2 Secretions, coughing, or gagging 3 Loss of airway through total or partial extubation 4 Patient fighting the ventilator 5 Decreased compliance due to pulmonary edema 6 Kinked or compressed tubing

2, 4, 5, 6 The possible causes for a high-pressure limit alarm to go off include secretions, coughing, or gagging. It may also be set off in case of ventilator asynchrony if the patient is fighting the ventilator. Decreased compliance due to conditions like pulmonary edema may also cause the setting off of the high-pressure alarm. It can also be due to kinked or compressed tubing, which usually happens when the patient is biting on the endotracheal tube. Oversedation and loss of airway cause an apnea alarm and can also set off the low-pressure limit alarm. Text Reference - p. 1620

The nurse is reporting the current nursing assessment to the physician. Vital signs: temperature, 97.2° F; pulse, 68 beats/minute, thready; respiration, 28 breaths/minute, blood pressure, 102/78 mm Hg; and pedal pulses, palpable. The physician asks for the pulse pressure. Which would the nurse report?

24 pg. 288

The nurse knows to report and early indicator of compensatory shock that would be the pulse pressure of?

25 mmHg

A patient is to be intubated for respiratory failure. Which factor indicates that tracheotomy would be preferable to endotracheal intubation? 1 The patient is at high risk for aspiration. 2 The patient is unable to clear secretions. 3 A long-term airway is probably necessary. 4 An upper airway obstruction is impairing the patient's ventilation.

3 A long-term airway is probably necessary. A tracheotomy is indicated when the need for an artificial airway is expected to be long term. Aspiration risk, an inability to clear secretions, and upper airway obstruction are indications for an artificial airway, but these are not specific indications for tracheotomy.

A patient is being mechanically ventilated. A high-pressure ventilation alarm sounds. The nurse should assess for what cause of this type of alarm? 1 Power failure 2 Insufficient gas flow 3 Condensate in tubing 4 Tracheotomy cuff leak

3 Condensate in tubing Presence of condensate or water in tubing triggers a high-pressure ventilation alarm. Power failure triggers ventilator inoperative or low battery alarm. Insufficient gas flow and tracheotomy cuff leak triggers low tidal volume or minute ventilation alarm.

When planning care for a patient on a mechanical ventilator, the nurse understands that the application of positive end-expiratory pressure (PEEP) to the ventilator settings has which therapeutic effect? 1 Increased inflation of the lungs 2 Prevention of barotrauma to the lung tissue 3 Prevention of alveolar collapse during expiration 4 Increased fraction of inspired oxygen concentration (FIO2) administration

3 Prevention of alveolar collapse during expiration PEEP is positive pressure that is applied to the airway during exhalation. This positive pressure prevents the alveoli from collapsing, improving oxygenation and enabling a reduced FIO2 requirement. PEEP does not cause increased inflation of the lungs or prevent barotrauma. Auto-PEEP resulting from inadequate exhalation time may contribute to barotrauma.

When assessing the settings of a patient's ventilator, the nurse knows that which parameter is abnormal? 1 PaO2 of 66 mm Hg 2 PEEP of 5 cm H 2O 3 Tidal volume of 12 mL/kg 4 Respiratory rate of 20 breaths/minute

3 Tidal volume of 12 mL/kg Usual tidal volume is 6-10 mL/kg; a tidal volume of 12 ml/kg is abnormally high. A respiratory rate of 20 breaths/minute is within normal; usual settings are 6-20 breaths/minute. A PaO2 level of 66 mm Hg is normal; the usual is greater than 60 mm Hg. PEEP of 5 cm H2O is the usual setting.

Which medication would the nurse administer to treat renal complications associated with systemic inflammatory response syndrome (SIRS)? 1 Antacids 2 Sucralfate 3 Furosemide 4 Omeprazole

3 A SIRS-affected (systemic inflammatory response syndrome) patient is administered a loop diuretic such as furosemide to address renal complications related to hypoperfusion and a decreased glomerular filtration rate. Antacids, sucralfate, and omeprazole act as a stress ulcer prophylaxis for the gastrointestinal system. Text Reference - p. 1651

The nurse assesses a patient with multisystem organ dysfunction syndrome. What assessment finding is most indicative of deterioration? 1 Arterial PO2 of 95% 2 Pulse rate of 108 beats/minute 3 Total urine output of 120 mL over the past eight hours 4 Auscultation of fine bilateral crackles and a moist cough

3 A decrease in urine output to less than 30 mL/hr in an adult is an early indication of hypoperfusion to the kidneys, as well as other vital organs. This may or may not be accompanied by changes in vital signs. An arterial PO2 of 95%, a pulse rate of 108 beats/minute, and auscultation of crackles and a moist cough may also be indications of early deterioration but are not as specific as a low urinary output in identifying deterioration. Text Reference - p. 1649

The nurse is admitting a patient with asthma in acute respiratory distress. The nurse auscultates the patient's lungs and notes cessation of the inspiratory wheezing. The patient has not yet received any medication. What should this finding most likely suggest to the nurse? 1 Spontaneous resolution of the acute asthma attack 2 An acute development of bilateral pleural effusions 3 Airway constriction requiring intensive interventions 4 Overworked intercostal muscles resulting in poor air exchange

3 Airway constriction requiring intensive interventions When the patient in respiratory distress has inspiratory wheezing and then he or she ceases, it is an indication of airway obstruction. This finding requires emergency action to restore the airway. Cessation of inspiratory wheezing does not indicate spontaneous resolution of the acute asthma attack, bilateral pleural effusion development, or overworked intercostal muscles in this asthmatic patient who is in acute respiratory distress.

A patient is brought to the emergency department (ED) after multiple bee stings. On assessment, the nurse finds that the patient has edema on the lips and tongue as well as chest pain, dizziness, wheezing, and stridor. What type of shock should the nurse document this as? 1 Septic shock 2 Neurogenic shock 3 Anaphylactic shock 4 Obstructive shock

3 Anaphylactic shock is an acute, life-threatening hypersensitivity reaction to a sensitizing substance that, in this case, is insect venom. The reaction quickly causes massive vasodilation, release of vasoactive mediators, and an increase in capillary permeability. As capillary permeability increases, fluid leaks from the vascular space into the interstitial space. The consequences of these pathophysiologic processes include edema on the lips and tongue, chest pain, wheezing, and stridor. Sepsis is a systemic inflammatory response to a documented or suspected infection. Neurogenic shock is a hemodynamic phenomenon that can occur within 30 minutes of a spinal cord injury at the fifth thoracic (T5) vertebra or above. Obstructive shock develops when a physical obstruction to blood flow occurs with decreased cardiac output. Text Reference - p. 1636

Why is there a loss of lean body mass in patients with systemic inflammatory response syndrome (SIRS) and multiorgan dysfunction syndrome (MODS)? 1 Hypoglycemia occurs. 2 Glucose is converted to glycogen. 3 Fatty acids are mobilized for fuel. 4 Glucose is converted to amino acids.

3 Both SIRS and MODS trigger a hypermetabolism response leading to mobilization of fatty acids for fuel. Such a catabolic state leads to loss of lean body mass. Because catecholamines and glucocorticoids are released, hyperglycemia occurs, not hypoglycemia. Hyperglycemia also occurs, because glycogen stores are converted into glucose. Once glycogen is depleted, amino acids are converted into glucose and there is a reduction in protein stores. Text Reference - p. 1649

The nurse recalls that cardiogenic shock is differentiated from other forms of shock because the patient with cardiogenic shock typically experiences: 1 Hypotension 2 Dysrhythmias 3 Volume excess 4 Volume depletion

3 In cardiogenic shock the heart fails as a pump. This usually results in fluid retention and poor perfusion of organs, including the kidneys, which adds to the fluid volume excess. As cardiogenic shock progresses, hypotension will develop, but it is not the cause of cardiogenic shock. Dysrhythmias may or may not be present with cardiogenic shock. Volume depletion is not generally seen with cardiogenic shock. It is, however, seen more with hypovolemic or distributive forms of shock. Text Reference - p. 1633

What causes dysrhythmias in patients with systemic inflammatory response syndrome (SIRS) and multiple organ dysfunction syndrome (MODS)? 1 Metabolic acidosis resulting in increase in lactate levels 2 Increased capillary permeability in the cardiovascular system 3 Hypokalemia due to activities of aldosterone and catecholamines 4 Aldosterone-mediated sodium and water reabsorption in kidneys

3 In systemic inflammatory response syndrome (SIRS) and multiple organ dysfunction syndrome (MODS), there are hormonal and metabolic changes and fluid shifts, including the release of aldosterone and catecholamines. Aldosterone increases urinary potassium loss and catecholamines cause potassium to move into the cell, resulting in hypokalemia, which causes dysrhythmias. Dysrhythmias are not associated with an increased lactate level caused by metabolic acidosis. Increased capillary permeability causes tachycardia rather than dysrhythmias. The increase in sodium reabsorption raises the serum osmolality and stimulates the release of the antidiuretic hormone (ADH). Increased sodium is not related to the development of dysrhythmias. Text Reference - p. 1649

A patient is in the exudative phase of acute respiratory distress syndrome (ARDS). What does the nurse determine the function of surfactant will be in this phase? 1 Attract neutrophils 2 Decrease tidal volume 3 Maintain alveolar stability 4 Release cellular mediators

3 Maintain alveolar stability During the injury, or exudative phase, of ARDS, the alveolar type I and II cells produce surfactant to prevent alveolar collapse. The inflammatory and immune systems attract neutrophils to the pulmonary interstitium. A decrease in tidal volume is caused by stimulation of the juxtacapillary receptors. The neutrophils release biochemical, humoral, and cellular mediators to produce changes in the lungs.

A patient is receiving respiratory therapy for the treatment of acute respiratory failure. What are the related interventions to maximize oxygen delivery? 1 Administering IV antibiotics 2 Reducing pain, anxiety, and restlessness 3 Maintaining adequate hemoglobin concentration 4 Maintaining the caloric requirements of the body

3 Maintaining adequate hemoglobin concentration Adequate hemoglobin concentration is maintained by transfusion of packed red blood cells. The hemoglobin increases the oxygen-carrying capacity of the blood and thus supports oxygenation. Pulmonary infections are reduced by administering IV antibiotics, such as azithromycin. Sedation and analgesia, along with drug therapy, help to decrease anxiety, agitation, and pain. Nutritional therapy focuses on the maintenance of caloric requirement of patients with acute respiratory failure.

A nurse is caring for a patient diagnosed with acute respiratory distress syndrome. The nurse is aware that these patients often will require which intervention? 1 Peritoneal dialysis 2 Frequent suctioning 3 Mechanical ventilation 4 Creatinine and blood urea nitrogen (BUN) testing

3 Mechanical ventilation Patients with acute respiratory distress syndrome likely will require mechanical ventilation to support their respiratory status. Frequent suctioning is not required often, but some suctioning may be required. Peritoneal dialysis and creatinine and BUN testing might be necessary with some level of kidney failure, not respiratory compromise.

Which medical emergency is caused by the failure of two or more organ systems? 1 Crush syndrome 2 Toxic shock syndrome 3 Multiple organ dysfunction syndrome (MODS) 4 Systemic inflammatory response syndrome (SIRS)

3 Multiple organ dysfunction syndrome is caused by the failure of two or more organ systems in an acutely ill patient. A crushing injury to the skeletal muscle causes crush syndrome, which is characterized by shock and renal failure. Bacterial toxins cause toxic shock syndrome, which is characterized by high fever, hypotension and malaise. Systemic inflammatory response syndrome (SIRS) is a systemic inflammatory response caused by infection, ischemia, infarction and injury. Test-Taking Tip: Identifying content and what is being asked about that content is critical to your choosing the correct response. Be alert for words in the stem of the item that are the same or similar in nature to those in one or two of the options. Text Reference - p. 1649

The nurse is caring for a patient who developed cardiogenic shock. Which medical diagnosis does the nurse suspect? 1 Urosepsis 2 Hemorrhage 3 Myocardial infarction 4 Tension pneumothorax

3 Myocardial infarction may produce necrotic areas of cardiac tissue that lead to impaired contractility and decreased cardiac output. This may lead to a cardiogenic shock state. Hemorrhage may lead to a hypovolemic shock state, tension pneumothorax may lead to an obstructive shock state, and urosepsis may lead to a septic shock state. Text Reference - p. 1633

A patient's localized infection has progressed to the point where septic shock now is suspected. What medication is an appropriate treatment modality for this patient? 1 Insulin infusion 2 Intravenous (IV) administration of epinephrine 3 Aggressive IV crystalloid fluid resuscitation 4 Administration of nitrates and β-adrenergic blockers

3 Patients in septic shock require large amounts of crystalloid fluid replacement. Epinephrine is indicated in anaphylactic shock, and insulin infusion is not normally necessary in the treatment of septic shock (but can be). Nitrates and β-adrenergic blockers are used most often in the treatment of patients in cardiogenic shock. Text Reference - p. 1636

A patient in neurogenic shock is receiving phenylephrine. Which nursing actions are appropriate when caring for this patient? 1 Monitoring for signs of dyspnea and pulmonary edema 2 Monitoring for signs of hypokalemia and hyperglycemia 3 Monitoring for signs of reflex bradycardia and restlessness 4 Monitoring for signs of hypothyroidism and Addison's disease

3 Phenylephrine is α-adrenergic agonist and may cause bradycardia and restlessness due to central nervous system stimulation. β-adrenergic agonists such as epinephrine cause dyspnea and pulmonary edema. Phenylephrine does not increase the elimination of potassium levels or blood glucose levels. Therefore, the patient does not have risk of hypokalemia and hyperglycemia. Phenylephrine does not impair thyroid and adrenal gland functioning. Therefore, the nurse will not monitor for the signs of hypothyroidism and Addison's disease. Test-Taking Tip: Identify option components as correct or incorrect. This may help you identify a wrong answer. Text Reference - p. 1643

The nurse is caring for a patient who is experiencing acute respiratory distress due to accumulation of nasal secretions in the airways. What is appropriate to include in the patient's plan of care? Select all that apply. 1 Administer fentanyl. 2 Restrict fluids to 1 L daily. 3 Encourage the patient to cough. 4 Administer nebulized acetylcysteine. 5 Elevate the patient's lower limbs to 45 degrees.

3 Encourage the patient to cough. 4 Administer nebulized acetylcysteine. Coughing helps to expel the nasal secretions and clears the airways; therefore, the nurse should encourage the patient to cough. Administering nebulized acetylcysteine with a bronchodilator helps to reduce the thickness of nasal secretions and increases their elimination. The patient should be provided 2 to 3 L of fluids daily in order to reduce the thickness of nasal secretions. The nurse should elevate the patient's head by raising the bed to 45 degrees to prevent breathlessness. Elevating the lower limbs will not be helpful for the patient. Fentanyl helps to reduce pain but does not help to clear airways.

The nurse is planning care for a patient receiving intraaortic balloon pump (IABP) therapy. What interventions should be included to reduce this patient's risk of developing an infection? Select all that apply. 1 Keep lower extremities extended 2 Turn and reposition every two hours 3 Cover the site with an occlusive dressing 4 Administer prophylactic antibiotics as prescribed 5 Use strict aseptic technique with dressing changes

3, 4, 5 Interventions to reduce the risk of infection in a patient with an intraaortic balloon pump (IABP) include covering the site with an occlusive dressing, administering prophylactic antibiotics as prescribed, and using strict aseptic technique with dressing changes. Keeping the lower extremities extended would help prevent arterial trauma caused by insertion or displacement of the balloon. Turning and repositioning every two hours would help prevent the development of stasis pneumonia. Text Reference - p. 1612

A patient is admitted to the hospital with a suspected diagnosis of obstructive shock. What could be the possible causes of this type of shock? Select all that apply. 1 Hypersensitivity to antibiotics 2 Spinal cord injury 3 Cardiac tamponade 4 Tension pneumothorax 5 Superior vena cava syndrome

3, 4, 5 Obstructive shock develops when a physical obstruction to blood flow occurs resulting in decreased cardiac output. This can be caused by restricted diastolic filling of the right ventricle from compression caused by cardiac tamponade, tension pneumothorax, or superior vena cava syndrome. Hypersensitivity to antibiotics may cause anaphylactic shock. Spinal cord injury may lead to neurogenic shock. Text Reference - p. 1637

The nurse is teaching the patient's caregiver about receiving positive pressure ventilation. What movements should the nurse tell the caregiver to avoid doing to the patient? 1 Arm circles 2 Knee bends 3 Quadriceps setting 4 External rotation of the hip

4 External rotation of the hip The nurse should advise the caregiver to avoid external rotation of the patient's hip; this movement can be avoided by properly positioning the patient and by the use of specialized mattresses and beds. Simple maneuvers such as arm circles, knee bends and quadriceps setting should be performed, because they maintain the muscle tone in the upper and lower extremities of the patient.

The negative inspiratory force (NIF) is measured in a patient who is on positive pressure ventilation. Among the values given, which NIF value is the best indication for weaning? 1 -30 cm H2O 2 -40 cm H2O 3 -50 cm H2O 4 -60 cm H2O

4 Negative inspiratory force (NIF) is the amount of negative pressure that a patient is able to generate to initiate spontaneous respirations. An NIF of less than -20 cm H2O is an indicator for weaning but the more negative the number, the better the indication for weaning. Therefore -60 cm H2O is the best indication for weaning. Text Reference - p. 1626

When taking care of a patient diagnosed with respiratory failure on a mechanical ventilator, the nurse hears the apnea alarm beeping. What assessment data should be gathered to determine the cause of the alarm? 1 Pain or anxiety 2 Partial ventilator disconnect 3 Secretions, coughing, or gagging 4 Oversedation with opioid analgesics

4 Oversedation with opioid analgesics The apnea alarm on mechanical ventilation may be caused by respiratory arrest, oversedation, change in patient condition, or loss of airway (total or partial extubation). The high-pressure limit alarm is caused by secretions, coughing, or gagging. The low tidal volume alarm can be caused by partial ventilator disconnect. The high tidal volume alarm can be caused by pain or anxiety.

Which laboratory finding in a patient with multiple organ dysfunction syndrome (MODS) suggests prerenal manifestations of renal dysfunction? 1 Urine Na+ is 22 mEq/L. 2 Urine specific gravity is 1.010. 3 Urine osmolality is decreased. 4 Urine specific gravity is increased.

4 An increase in urine specific gravity suggests prerenal manifestations in patients with MODS. This occurs due to renal hypoperfusion. A urine Na+ level above 20 mEq/L is a sign of intrarenal manifestations in MODS. It occurs due to acute tubular necrosis. Other intrarenal manifestations of MODS are urine specific gravity of around 1.010 and a decrease of the urine osmolality.

What causes gut bacteria to move into circulation in patients with systemic inflammatory response syndrome (SIRS)? 1 Toxic effects of medicines 2 Effect of inflammatory mediators 3 Decreased gastrointestinal motility 4 Decreased perfusion of gut mucosa

4 Decreased perfusion in the gastrointestinal (GI) tract leads to a breakdown of the normally protective mucosal barrier, which causes the bacterial movement from the GI tract into circulation. In order to control these bacteria, antibiotics are administered. Antibiotics are nephrotoxic medicines that can cause acute kidney injury. The breakdown of the mucosal barrier is the direct effect of hypoperfusion rather than the inflammatory mediators. In critical illnesses, GI motility is often decreased causing abdominal distension and paralytic ileus. Test-Taking Tip: Make certain that the answer you select is reasonable and obtainable under ordinary circumstances and that the action can be carried out in the given situation. Text Reference - p. 1649

Which pathophysiologic change occurs in the fibrotic phase of acute respiratory distress syndrome (ARDS)? 1 Diseased lung is characterized by dense, fibrous tissue 2 Increased fluid accumulation and decreased lung compliance 3 Engorgement of the peribronchial and perivascular interstitial space 4 Diseased lung is completely remodeled by collagenous and fibrous tissues

4 Diseased lung is completely remodeled by collagenous and fibrous tissues The fibrotic phase is the late phase of acute respiratory distress syndrome (ARDS), which occurs two to three weeks after the lung injury. During this phase, the lung is completely remodeled by collagenous and fibrous tissues. During the reparative phase, which begins one to two weeks from the lung injury, the diseased lung appears dense with fibrous tissue, there is an increase in the fluid accumulations, and lung compliance decreases. Engorgement of the peribronchial and perivascular interstitial space occurs within 24 to 48 hours of the lung injury.

A patient is diagnosed with multiple organ dysfunction syndrome. While aggressive treatment is continued, the nurse suspects infection. What is the most appropriate action that the nurse should perform? 1 Discontinue the aggressive treatment. 2 Reduce oxygen delivery to the patient. 3 Wait for laboratory reports to confirm the suspicion. 4 Obtain a prescription for broad-spectrum antibiotic therapy.

4 If an infection is suspected, broad-spectrum antibiotics should be started immediately to limit the infection. Aggressive treatment for infection control should be carried out in parallel. These patients are usually hypoxemic. Therefore, oxygen should be administered strictly as prescribed. Cultures can be sent, and based on the reports, specific antibiotics can be added. Test-Taking Tip: Identify option components as correct or incorrect. This may help you identify a wrong answer. Example: If you are being asked to identify a diet that is specific to a certain condition, your knowledge about that condition would help you choose the correct response (e.g., cholecystectomy = low-fat, high-protein, low-calorie diet). Text Reference - p. 1650

Following coronary artery bypass graft surgery a patient has postoperative bleeding that requires returning to surgery to repair the leak. During surgery, the patient has a myocardial infarction (MI). After restoring the patient's body temperature to normal, which patient assessment is the most important for planning nursing care? 1 Cardiac index (CI) 5 L/min/m2 2 Central venous pressure (CVP) 8 mm Hg 3 Mean arterial pressure (MAP) 86 mm Hg 4 Pulmonary artery pressure (PAP) 28/14 mm Hg

4 Pulmonary hypertension as indicated by an elevated PAP indicates impaired forward flow of blood because of left ventricular dysfunction or hypoxemia. Both can be caused by the MI. The CI, CVP, and MAP readings are normal. Text Reference - p. 1639

A patient with significant right-sided pneumonia is receiving respiratory therapy. Which position is best suited for this patient? 1 Prone position 2 Tripod position 3 Supine position 4 Side-lying position

4 Side-lying position A patient with a medical condition involving only one lung requires focused intervention. A lateral or side-lying position is used for patients whose condition involves only one lung, because it allows for improved ventilation to perfusion that matches with the affected lung. This position also optimizes pulmonary blood flow and ventilation to the dependent lung areas. When a patient is in the prone position, air-filled, nonatelectatic alveoli in the ventral (anterior) portion of the lung become dependent, and perfusion may be better matched to ventilation. However, not all patients respond well to prone positioning, and there is no reliable way of predicting who will respond. Tripod positioning helps to increase chest and lung expansion and decrease the effort needed to breathe for patients with chronic obstructive pulmonary disease, not patients with conditions affecting only one lung. The supine position changes the pleural pressure and predisposes the patient to atelectasis.

What is the clinical manifestation of systemic inflammatory response syndrome (SIRS) and multiple organ dysfunction syndrome (MODS) on the respiratory system? 1 Pulmonary edema 2 Pulmonary fibrosis 3 Pulmonary embolism 4 Pulmonary hypertension

4 Systemic inflammatory response syndrome (SIRS) and multiple organ dysfunction syndrome (MODS) affect the respiratory system and lead to pulmonary hypertension. Pulmonary edema is caused by fluid accumulation in the air spaces. Pulmonary fibrosis is a respiratory disorder caused by scars in the lung tissues or inhalation of airborne toxins. A pulmonary embolism is caused by blood clots or blockage in the pulmonary arteries. Text Reference - p. 1651

Which sign of neurologic dysfunction is commonly seen in both systemic inflammatory response syndrome (SIRS) and multiple organ dysfunction syndrome (MODS)? 1 Increased heart rate 2 Increased liver enzymes 3 Difficulty breathing 4 Confusion, agitation, and lethargy

4 The sign of neurologic dysfunction commonly seen in systemic inflammatory response syndrome (SIRS) and multiple organ dysfunction syndrome (MODS) is change in mental status, which may cause the patient to become confused, agitated, and lethargic. The patient's heart rate increases due to changes in the cardiovascular system. The patient's liver enzymes increase due to dysfunction in the hepatic system and finally the dysfunction results in hepatic encephalopathy. The patient's dyspnea is caused by changes in the respiratory system by inflammatory mediators. Test-Taking Tip: Identify option components as correct or incorrect. This may help you identify a wrong answer. Text Reference - p. 1649

Bed rest is prescribed for a patient during the acute phase of respiratory failure. What is the rationale for the recommendation of bed rest and limitation of other activity in the plan of care? 1 To prevent further alveolar collapse 2 To decrease the basal metabolic rate 3 To promote the clearance of secretions 4 To reduce the cellular demand for oxygen

4 To reduce the cellular demand for oxygen To reduce the cellular demand for oxygen Respiratory failure interferes with ventilation and oxygenation. It is essential to reduce the body's need for oxygen at the cellular level. Bed rest is an essential and effective means of reducing the need for oxygen. Bed rest and limitation of activity do not prevent alveolar collapse, clear secretions, or decrease the basal metabolic rate.

The most commonly used colloid solution to treat hypovolemic shock is?

5% albumin

If the patient in shock is to receive 1000 mL of normal saline in two hours, at what rate should the infusion pump be set? Fill in the blank. ___mL/hour

500 For the 1000 mL of normal saline to be infused in two hours, the infusion pump should be set at 500 mL per hour (1000 mL divided by two hours). Test-Taking Tip: When taking the NCLEX exam, an on-screen calculator will be available for you to determine your response, which you will then type in the provided space. Text Reference - p. 1641

The nurse knows when the cardiovascular system becomes ineffective in maintaining an adequate mean arterial pressure (MAP). Select the reading below that indicates tissue hypoperfusion.

60 mm Hg

The nurse knows when the cardiovascular system becomes ineffective in maintaining an adequate mean arterial pressure (MAP). Select the reading below that indicates tissue hypoperfusion.

60 mm Hg p. 286, 303.

The nurse is using continuous central venous oximetry (ScvO2) to monitor the blood oxygen saturation of a patient in shock. What value would the nurse document as normal for the patient?

70%

The nurse is using continuous central venous oximetry (ScvO2) to monitor the blood oxygen saturation of a patient in shock. What value would the nurse document as normal for the patient?

70% pg. 289

The nurse is calculating a patient's mean arterial pressure (MAP). What is the patient's MAP, if the blood pressure is 110/70 mm Hg?

83

In progressive stage shock, clinical hypotension is present if the systematic bp i?

85 mmHg

Physiological Integrity 19. Which information obtained by the nurse when assessing a patient with acute respiratory distress syndrome (ARDS) who is being treated with mechanical ventilation and high levels of positive end-expiratory pressure (PEEP) indicates a complication of ventilator therapy is occurring? a. The patient has subcutaneous emphysema. b. The patient has a sinus bradycardia, rate 52. c. The patient's PaO2 is 50 mm Hg and the SaO2 is 88%. d. The patient has bronchial breath sounds in both the lung fields.

A Rationale: Complications of positive-pressure ventilation (PPV) and PEEP include subcutaneous emphysema. Bradycardia, hypoxemia, and bronchial breath sounds are all concerns, but they are not caused by PPV and PEEP. Cognitive Level: Application Text Reference: p. 1816 Nursing Process: Assessment

Physiological Integrity 2. The nurse will monitor for clinical manifestations of hypercapnia when a patient in the emergency department has a. chest trauma and multiple rib fractures. b. carbon monoxide poisoning after a house fire. c. left-sided ventricular failure and acute pulmonary edema. d. tachypnea and acute respiratory distress syndrome (ARDS).

A Rationale: Hypercapnia is caused by poor ventilatory effort, which occurs in chest trauma when rib fractures (or flail chest) decrease lung ventilation. Carbon monoxide poisoning, acute pulmonary edema, and ARDS are more commonly associated with hypoxemia. Cognitive Level: Application Text Reference: p. 1800 Nursing Process: Assessment

Physiological Integrity 14. When admitting a patient in possible respiratory failure with a high PaCO2, which assessment information will be of most concern to the nurse? a. The patient is somnolent. b. The patient's SpO2 is 90%. c. The patient complains of weakness. d. The patient's blood pressure is 162/94.

A Rationale: Increasing somnolence will decrease the patient's respiratory rate and further increase the PaCO2 and respiratory failure. Rapid action is needed to prevent respiratory arrest. An SpO2 of 90%, weakness, and elevated blood pressure all require ongoing monitoring but are not indicators of possible impending respiratory arrest. Cognitive Level: Application Text Reference: p. 1804 Nursing Process: Assessment

Physiological Integrity 22. A patient with acute respiratory distress syndrome (ARDS) has progressed to the fibrotic phase. The patient's family members are anxious about the patient's condition and are continuously present at the hospital. In addressing the family's concerns, it is important for the nurse to a. support the family and help them understand the realistic expectation that the patient's chance for survival is poor. b. inform the family that home health nurses will be able to help them maintain the mechanical ventilation at home after patient discharge. c. refer the family to social support services and case management to plan for transfer of the patient to a long-term care facility. d. provide hope and encouragement to the family because the patient's disease process has started to resolve.

A Rationale: The chance for survival is poor when the patient progresses to the fibrotic stage because permanent damage to the alveoli has occurred. Because of continued severe hypoxemia, the patient is not a candidate for home health or long-term care. The fibrotic stage indicates a poor patient prognosis, not the resolution of the ARDS process. Cognitive Level: Application Text Reference: p. 1814 Nursing Process: Implementation

Physiological Integrity 5. A patient is brought to the emergency department unconscious following a barbiturate overdose. Which potential complication will the nurse include when developing the plan of care? a. Hypercapnic respiratory failure related to decreased ventilatory effort b. Hypoxemic respiratory failure related to diffusion limitations c. Hypoxemic respiratory failure related to shunting of blood d. Hypercapnic respiratory failure related to increased airway resistance

A Rationale: The patient with an opioid overdose develops hypercapnic respiratory failure as a result of the decrease in respiratory rate and depth. Diffusion limitations, blood shunting, and increased airway resistance are not the primary pathophysiology causing the respiratory failure. Cognitive Level: Application Text Reference: p. 1800 Nursing Process: Diagnosis

1 Prone positioning refers to the repositioning of a patient from a supine or lateral position to a prone position. This repositioning improves lung reexpansion through various mechanisms. Firstly, the gravity reverses the effects of fluid in the dependent parts of the lungs as the patient is moved from supine to prone. Secondly, in the prone position, the heart rests on the sternum, away from the lungs, contributing to an overall uniformity of pleural pressures. These two mechanisms help in better ventilation in the patient with respiratory failure. The prone position is a relatively safe supportive therapy used for critically ill patients with acute lung injury or ARDS and is used for improved oxygenation. Resting in other positions such as sitting, supine, or lateral may not help in oxygenation. Text Reference - p. 1622

A nurse is attending a patient with acute respiratory distress syndrome (ARDS). Which position is best for this patient? 1 Prone 2 Sitting 3 Supine 4 Lateral

1, 2, 3 Enteral nutrition preserves the structure and function of the gut mucosa and stops the movement of gut bacteria across the intestinal wall and into the bloodstream. In addition to this, early enteral nutrition is associated with fewer complications. Enteral feedings cannot be administered to all patients; in patients with paralytic ileus, intestinal obstruction, and GI ischemia, enteral feeding is contraindicated. In these patients, parenteral feeding is the best option. Text Reference - p. 1600

A nurse is starting enteral feeding through a nasogastric tube for a patient in ICU. What advantages of enteral feeding over parenteral feeding does the nurse identify in the patient? Select all that apply. 1 Preserves the structure and function of gut mucosa 2 Stops the movement of gut bacteria across the intestinal wall 3 Results in fewer complications 4 Prevents and corrects nutritional deficiencies 5 Can be administered to all patients

4 Central venous pressure (CVP) is a measure of the filling pressure of the right ventricle and is indicative of how the right side of the heart accommodates fluid load. A series of CVP measurements of 12 mm Hg or higher indicates failure of the right ventricle to handle venous return. A normal CVP measurement is 2 to 8 mm Hg. Cardiogenic shock and circulatory failure are late manifestations of heart failure in general and would likely show a decreased CVP and cardiac output. CVP may be increased with left ventricular failure; however, this is a late sign. It is possible to have both right and left failure at the same time. Text Reference - p. 1608

A nurse measures a patient's central venous pressure and recognizes a series of increased readings as directly indicative of: 1 Cardiogenic shock 2 Circulatory failure 3 Left ventricular failure 4 Right ventricular failure

2 Mechanical ventilation can cause pneumothorax as a result of excessive pressure applied to lung tissue. Hypertension is not a direct complication; however, a patient undergoing mechanical ventilation may be anxious and fearful, resulting in high blood pressure; sedation should be considered in this event. Electrolyte imbalance is not a related complication. Mechanical ventilation does increase intrathoracic pressure, which may then increase cardiac output, causing a beneficial secondary effect. Text Reference - p. 1615

A nurse should monitor a patient undergoing mechanical ventilation for which common complication? 1 Hypertension 2 Pneumothorax 3 Electrolyte imbalance 4 Increased cardiac output

3 Presence of condensate or water in tubing triggers a high-pressure ventilation alarm. Power failure triggers ventilator inoperative or low battery alarm. Insufficient gas flow and tracheotomy cuff leak triggers low tidal volume or minute ventilation alarm. Text Reference - p. 1620

A patient is being mechanically ventilated. A high-pressure ventilation alarm sounds. The nurse should assess for what cause of this type of alarm? 1 Power failure 2 Insufficient gas flow 3 Condensate in tubing 4 Tracheotomy cuff leak

3 Before inserting a line into the radial artery, an Allen test should be performed to confirm that ulnar circulation to the hand is adequate. In this test, pressure is applied to the radial and ulnar arteries simultaneously. The patient opens and closes the hand repeatedly until the hand blanches. When the pressure on the ulnar artery is released, the hand should return to a pink color within six seconds. If pinkness does not return within six seconds the ulnar artery is inadequate to maintain blood flow to the extremity and the radial artery should not be used for arterial line insertion. The phlebostatic axis is used to zero the arterial line, which would be done much later. Because of the risk of heparin-induced thrombocytopenia (HIT), heparinized saline should not be routinely used for the flush solution. The flush bag should be set to deliver 3 to 6 mL/hr. Text Reference - p. 1606

A patient is being prepared for insertion of an arterial measuring device. What should be done before the catheter is inserted into the patient's radial artery? 1 Locate the phlebostatic axis 2 Prepare a heparinized flush bag 3 Occlude the radial and ulnar arteries 4 Set an intravenous pump to deliver 15 mL/h

4 Indications for intraaortic balloon pump (IABP) therapy include acute myocardial infarction and cardiogenic shock. The use of the pump with this health problem allows time for emergent angiography. The pump is not used to reduce pressure in the pulmonary artery, improve coronary artery vessel perfusion, or to enhance the effectiveness of cardiac medications. Text Reference - p. 1610

A patient is experiencing cardiogenic shock after an acute myocardial infarction. Why would an intraaortic balloon pump (IABP) be beneficial for this patient? 1 Reduces pressure in the pulmonary artery 2 Improves coronary artery vessel perfusion 3 Enhances effectiveness of cardiac medications 4 Provides time for an emergency angiogram to be performed

2 Benzodiazepines, such as lorazepam, have anxioloytic activity and help to alleviate symptoms of anxiety in patients. Propofol is an anesthetic, and fentanyl is an analgesic agent. These two medications do not help to treat anxiety but are used to relax intubated, ventilated patients. Esomeprazole is a proton pump inhibitor that helps to reduce symptoms of peptic ulcer. Text Reference - p. 1600

A patient is experiencing symptoms of anxiety. The nurse anticipates that which medication will be prescribed? 1 Propofol 2 Lorazepam 3 Fentanyl 4 Esomeprazole

3 An SBT is recommended in patients who demonstrate weaning readiness. An SBT should be at least 30 minutes but no more than 120 minutes. At least 15 minutes but no more than 30 minutes, at least 30 minutes but no more than 60 minutes, and at least 60 minutes but no more than 120 minutes are not recommended time frames to determine weaning readiness. Text Reference - p. 1626

A patient is intubated. The nurse has to perform a spontaneous breathing trial (SBT) on this patient. For how long should this trial be done? 1 At least 15 minutes but not more than 30 minutes 2 At least 30 minutes but not more than 60 minutes 3 At least 30 minutes but not more than 120 minutes 4 At least 60 minutes but not more than 120 minutes

2 The patient with a ventricular assist device (VAD) may be mobile and require an activity plan such as progressive ambulation. Complete bedrest is not required. The patient will be permitted to do more than move from the bed to a chair twice a day. Activity will be greater than bed rest with bathroom privileges. Text Reference - p. 1613

A patient is recovering from the implantation of a ventricular assist device (VAD). What should the nurse anticipate being prescribed for this patient's activity status? 1 Complete bedrest 2 Progressive ambulation 3 Moving out of bed to a chair twice a day 4 Bedrest with bathroom privileges

1 The patient has a urinary tract infection, which can lead to sepsis. In sepsis, oxygen is not extracted properly at the tissue level, resulting in increased central venous oxygen saturation (ScvO2) mixed venous oxygen saturation (SvO2) measurements. Central venous oxygen saturation (ScvO2) mixed venous oxygen saturation (SvO2) measurements would be low if the patient was experiencing decreased cardiac output or increased oxygen demand. Central venous oxygen saturation (ScvO2) mixed venous oxygen saturation (SvO2) measurements would be between 60% and 80% if the oxygen supply and demand was balanced. Text Reference - p. 1609

A patient with a pulmonary arterial catheter for systolic heart failure is diagnosed with a urinary tract infection (UTI). The last central venous oxygen saturation (ScvO2) mixed venous oxygen saturation (SvO2) measurement was 89%. What should the nurse suspect is occurring with this patient? 1 Sepsis 2 Decreased cardiac output 3 Increased oxygen demand 4 Balanced oxygen supply and deman

3 A central venous oxygen saturation (ScvO2) mixed venous oxygen saturation (SvO2) measurement of 48% is low. Metabolic demand exceeds oxygen supply in conditions that increase muscle movement and metabolic rate, including physiologic states such as seizures. Central venous oxygen saturation (ScvO2) mixed venous oxygen saturation (SvO2) measurements greater than 80% are caused by increased oxygen supply. Even though a low central venous oxygen saturation (ScvO2) mixed venous oxygen saturation (SvO2) measurement is associated with a decreased cardiac output, the patient is not experiencing a health problem such as cardiogenic shock caused by left ventricular pump failure that supports decreased cardiac output as the reason for the low measurement. Central venous oxygen saturation (ScvO2) mixed venous oxygen saturation (SvO2) measurements greater than 80% are caused by decreased oxygen demand. Text Reference - p. 1609

A patient with meningitis and seizures has a pulmonary arterial catheter inserted. The most recent central venous oxygen saturation (ScvO2) mixed venous oxygen saturation (SvO2) measurement is 48%. What should the nurse realize is the reason for this patient's measurement? 1 Increased oxygen supply 2 Decreased cardiac output 3 Increased oxygen demand 4 Decreased oxygen demand

120 MAP is calculated by adding the systolic blood pressure to two times the diastolic blood pressure and dividing by three. For this patient that calculation would be 172 + 2(94)/3 = 120 mm Hg. Normal MAP is between 70 and 105 mm Hg. Text Reference - p. 1603

A patient's blood pressure is 172/94 mm Hg. What would the nurse calculate as being this patient's mean arterial pressure (MAP)? Record your answer using a whole number. ____________ mm Hg

4 SIMV stands for synchronized intermittent mandatory ventilation, a mode of ventilation in which the ventilator delivers a preset tidal volume at a preset frequency in synchrony with the patient's spontaneous breathing. Between ventilator-delivered breaths the patient is able to breathe spontaneously, receiving the preset FIO2, but self-regulates the rate and depth of those breaths. Pressure support ventilation (PSV) applies positive pressure only during inspiration. PSV is not used as a sole ventilator support during acute respiratory failure because of the risk of hypoventilation, but it does decrease the work of breathing. Pressure-control inverse ratio ventilation (PC-IRV) sets the ventilation pressure and the ratio of inspiration to expiration to control the patient's breathing. Assist-control ventilation (ACV) or assisted mandatory ventilation (AMV) delivers a preset rate of breaths, but allows the patient to breathe spontaneously, with a preset tidal volume. Text Reference - p. 1620

A patient's family member asks the nurse what SIMV means on the settings of the mechanical ventilator attached to the patient. Which statement best describes this mode of ventilation? 1 "SIMV provides additional inspiratory pressure so that your father does not have to work as hard to breathe, thus enabling better oxygenation and a quicker recovery with fewer complications." 2 "SIMV is a mode that allows the ventilator to totally control your father's breathing. It will prevent him from hyperventilating or hypoventilating, thus ensuring the best oxygenation." 3 "SIMV is a mode that allows your father to breathe on his own, but the ventilator will control how deep a breath he will receive. The ventilator can sense when he wants a breath and it will deliver it." 4 "SIMV is a mode that allows your father to breathe on his own while receiving a preset number of breaths from the ventilator. He can breathe as much or as little as he wants beyond what the ventilator will breathe for him."

Appropriate treatment modalities for the mgmt. of carcinogenic shock include (select all) A) dobutamine to increase myocardial contractility B) Vasopressors to increase systemic vascular resistance C) circulatory assist devices such as an intraaortic balloon pump D) corticosteroids to stabilize the cell wall in the infarcted myocardium E) Trendelenburg positioning to facilitate venous return and increase preload

A) dobutamine to increase myocardial contractility C) circulatory assist devices such as an intraaortic balloon pump

Which set of clinical manifestations is highly characteristic of a septic shock state? A. Tachycardia, hypotension, and warm skin B. Confusion, bradycardia, and truncal rash C. Severe respiratory distress, jugular venous distention, and chest pain D. Decreased cardiac output, hypertension, and poor skin turgor

A. Tachycardia, hypotension, and warm skin

Which of the following conditions presents the most significant risk factor for developing septic shock? A. Use of immunosuppressant medications B. History of severe allergies C. Diagnosis of chronic congestive heart failure D. Genetic predisposition to disorders of hemostasis

A. Use of immunosuppressant medications

Why would a third-degree, circumferential burn of the thigh require prompt medical attention? A. Wound contraction and edema can severely impair limb circulation. B. It is difficult to perform skin grafts of the limbs. C. Burn shock is inevitable. D. Third-degree burns are very painful.

A. Wound contraction and edema can severely impair limb circulation.

1. A 198-lb patient is to receive a dobutamine infusion at 5 mcg/kg/min. The label on the infusion bag states: dobutamine 250 mg in 250 mL of normal saline. When setting the infusion pump, the nurse will set the infusion rate at how many milliliters per hour?

ANS: 27 To administer the dobutamine at the prescribed rate of 5 mcg/kg/min from a concentration of 250 mg in 250 mL, the nurse will need to infuse 27 mL/hr.

1. A patient's vital signs are pulse 90, respirations 24, and BP 128/64 mm Hg, and cardiac output is 4.7 L/min. The patient's stroke volume is _____ mL. (Round to the nearest whole number.)

ANS: 52 Stroke volume = Cardiac output/heart rate 52 mL = (4.7 L x 1000 mL/L)/90

2. The nurse is caring for a patient who has an intraaortic balloon pump (IABP) after a massive heart attack. When assessing the patient, the nurse notices blood backing up into the IABP catheter. In which order should the nurse take the following actions? (Put a comma and a space between each answer choice [A, B, C, D].) a. Confirm that the IABP console has turned off. b. Assess the patient's vital signs and orientation. c. Obtain supplies for insertion of a new IABP catheter. d. Notify the health care provider of the IABP malfunction.

ANS: A, B, D, C Blood in the IABP catheter indicates a possible tear in the balloon. The console should shut off automatically to prevent complications such as air embolism. Next, the nurse will assess the patient and communicate with the health care provider about the patient's assessment and the IABP problem. Finally, supplies for insertion of a new IABP catheter may be needed based on the patient assessment and the decision of the health care provider.

1. When assisting with oral intubation of a patient who is having respiratory distress, in which order will the nurse take these actions? (Put a comma and a space between each answer choice [A, B, C, D, E].) a. Obtain a portable chest-x-ray. b. Position the patient in the supine position. c. Inflate the cuff of the endotracheal tube after insertion. d. Attach an end-tidal CO2 detector to the endotracheal tube. e. Oxygenate the patient with a bag-valve-mask device for several minutes.

ANS: E, B, C, D, A The patient is pre-oxygenated with a bag-valve-mask system for 3 to 5 minutes before intubation and then placed in a supine position. After the intubation, the cuff on the endotracheal tube is inflated to occlude and protect the airway. Tube placement is assessed first with an end-tidal CO2 sensor and then with chest x-ray examination.

1. The health care provider prescribes these actions for a patient who has possible septic shock with a BP of 70/42 mm Hg and oxygen saturation of 90%. In which order will the nurse implement the actions? Put a comma and space between each answer choice (a, b, c, d, etc.) ____________________ a. Obtain blood and urine cultures. b. Give vancomycin (Vancocin) 1 g IV. c. Infuse vasopressin (Pitressin) 0.01 units/min. d. Administer normal saline 1000 mL over 30 minutes. e. Titrate oxygen administration to keep O2 saturation >95%.

ANS: E, D, C, A, B The initial action for this hypotensive and hypoxemic patient should be to improve the oxygen saturation, followed by infusion of IV fluids and vasopressors to improve perfusion. Cultures should be obtained before administration of antibiotics.

The health care provider prescribes these actions for a patient who has possible septic shock with a BP of 70/42 mm Hg and oxygen saturation of 90%. In which order will the nurse implement the actions? Put a comma and space between each answer choice (a, b, c, d, etc.) ____________________ a. Obtain blood and urine cultures. b. Give vancomycin (Vancocin) 1 g IV. c. Infuse vasopressin (Pitressin) 0.01 units/min. d. Administer normal saline 1000 mL over 30 minutes. e. Titrate oxygen administration to keep O2 saturation >95%.

ANS: E, D, C, A, B The initial action for this hypotensive and hypoxemic patient should be to improve the oxygen saturation, followed by infusion of IV fluids and vasopressors to improve perfusion. Cultures should be obtained before administration of antibiotics.

1. The health care provider orders the following interventions for a 67-kg patient who has septic shock with a blood pressure of 70/42 mm Hg and O2 saturation of 90% on room air. In which order will the nurse implement the actions? (Put a comma and a space between each answer choice [A, B, C, D, E].) a. Give vancomycin 1 g IV. b. Obtain blood and urine cultures c. Start norepinephrine 0.5 mcg/min. d. Infuse normal saline 2000 mL over 30 minutes. e. Titrate oxygen administration to keep O2 saturation above 95%.

ANS: E, D, C, B, A The initial action for this hypotensive and hypoxemic patient should be to improve the O2 saturation, followed by infusion of IV fluids and vasopressors to improve perfusion. Cultures should be obtained before giving antibiotics.

1. A patient's vital signs are pulse 87, respirations 24, and BP of 128/64 mm Hg and cardiac output is 4.7 L/min. The patient's stroke volume is _____ mL. (Round to the nearest whole number.)

ANS: 54 Stroke volume = cardiac output/heart rate DIF: Cognitive Level: Understand (comprehension) REF: 1603 TOP: Nursing Process: Assessment MSC: NCLEX: Physiological Integrity

2. The nurse is caring for a patient who has an intraortic balloon pump (IABP) following a massive heart attack. When assessing the patient, the nurse notices blood backing up into the IABP catheter. In which order should the nurse take the following actions? (Put a comma and a space between each answer choice [A, B, C, D].) a. Ensure that the IABP console has turned off. b. Assess the patient's vital signs and orientation. c. Obtain supplies for insertion of a new IABP catheter. d. Notify the health care provider of the IABP malfunction.

ANS: A, B, D, C Blood in the IABP catheter indicates a possible tear in the balloon. The console will shut off automatically to prevent complications such as air embolism. Next, the nurse will assess the patient and communicate with the health care provider about the patient's assessment and the IABP problem. Finally, supplies for insertion of a new IABP catheter may be needed, based on the patient assessment and the decision of the health care provider. DIF: Cognitive Level: Analyze (analysis) REF: 1612 OBJ: Special Questions: Prioritization TOP: Nursing Process: Planning MSC: NCLEX: Physiological Integrity

1. When assisting with oral intubation of a patient who is having respiratory distress, in which order will the nurse take these actions? (Put a comma and a space between each answer choice [A, B, C, D, E].) a. Obtain a portable chest-x-ray. b. Position the patient in the supine position. c. Inflate the cuff of the endotracheal tube after insertion. d. Attach an end-tidal CO2 detector to the endotracheal tube. e. Oxygenate the patient with a bag-valve-mask device for several minutes.

ANS: E, B, C, D, A The patient is pre-oxygenated with a bag-valve-mask system for 3 to 5 minutes before intubation and then placed in a supine position. Following the intubation, the cuff on the endotracheal tube is inflated to occlude and protect the airway. Tube placement is assessed first with an end-tidal CO2 sensor, then with a chest x-ray. DIF: Cognitive Level: Analyze (analysis) REF: 1614 OBJ: Special Questions: Prioritization TOP: Nursing Process: Implementation MSC: NCLEX: Physiological Integrity

22. A patient with respiratory failure has arterial pressure-based cardiac output (APCO) monitoring and is receiving mechanical ventilation with peak end-expiratory pressure (PEEP) of 12 cm H2O. Which information indicates that a change in the ventilator settings may be required? a. The arterial pressure is 90/46. b. The stroke volume is increased. c. The heart rate is 58 beats/minute. d. The stroke volume variation is 12%.

ANS: A The hypotension suggests that the high intrathoracic pressure caused by the PEEP may be decreasing venous return and (potentially) cardiac output. The other assessment data would not be a direct result of PEEP and mechanical ventilation.

Norepinephrine (Levophed) has been prescribed for a patient who was admitted with dehydration and hypotension. Which patient data indicate that the nurse should consult with the health care provider before starting the norepinephrine? a. The patient's central venous pressure is 3 mm Hg. b. The patient is in sinus tachycardia at 120 beats/min. c. The patient is receiving low dose dopamine (Intropin). d. The patient has had no urine output since being admitted.

ANS: A Adequate fluid administration is essential before administration of vasopressors to patients with hypovolemic shock. The patient's low central venous pressure indicates a need for more volume replacement. The other patient data are not contraindications to norepinephrine administration.

A patient with septic shock has a BP of 70/46 mm Hg, pulse 136, respirations 32, temperature 104° F, and blood glucose 246 mg/dL. Which intervention ordered by the health care provider should the nurse implement first? a. Give normal saline IV at 500 mL/hr. b. Give acetaminophen (Tylenol) 650 mg rectally. c. Start insulin drip to maintain blood glucose at 110 to 150 mg/dL. d. Start norepinephrine (Levophed) to keep systolic blood pressure >90 mm Hg.

ANS: A Because of the low systemic vascular resistance (SVR) associated with septic shock, fluid resuscitation is the initial therapy. The other actions also are appropriate, and should be initiated quickly as well.

17. The nurse is caring for a patient who has septic shock. Which assessment finding is most important for the nurse to report to the health care provider? a. Skin cool and clammy c. Blood pressure of 92/56 mm Hg b. Heart rate of 118 beats/min d. O2 saturation of 93% on room air

ANS: A Because patients in the early stage of septic shock have warm and dry skin, the patient's cool and clammy skin indicates that shock is progressing. The other information will also be reported, but does not indicate deterioration of the patient's status.

22. The following interventions are ordered by the health care provider for a patient who has respiratory distress and syncope after eating strawberries. Which will the nurse complete first? a. Give epinephrine. b. Administer diphenhydramine. c. Start continuous ECG monitoring. d. Draw blood for complete blood count (CBC)

ANS: A Epinephrine rapidly causes peripheral vasoconstriction, dilates the bronchi, and blocks the effects of histamine and reverses the vasodilation, bronchoconstriction, and histamine release that cause the symptoms of anaphylaxis. The other interventions are also appropriate but would not be the first ones completed.

1. A patient with septic shock has a urine output of 20 mL/hr for the past 3 hours. The pulse rate is 120 and the central venous pressure and pulmonary artery wedge pressure are low. Which of these orders by the health care provider will the nurse question? a. Give furosemide (Lasix) 40 mg IV. b. Increase normal saline infusion to 150 mL/hr. c. Administer hydrocortisone (SoluCortef) 100 mg IV. d. Prepare to give drotrecogin alpha (Xigris) 24 mcg/kg/hr.

ANS: A Furosemide will lower the filling pressures and renal perfusion further for the patient with septic shock. The other orders are appropriate. DIF: Cognitive Level: Application REF: 1724-1726 | 1731 | 1733 TOP: Nursing Process: Implementation MSC: NCLEX: Physiological Integrity

A patient with septic shock has a urine output of 20 mL/hr for the past 3 hours. The pulse rate is 120 and the central venous pressure and pulmonary artery wedge pressure are low. Which of these orders by the health care provider will the nurse question? a. Give furosemide (Lasix) 40 mg IV. b. Increase normal saline infusion to 150 mL/hr. c. Administer hydrocortisone (SoluCortef) 100 mg IV. d. Prepare to give drotrecogin alpha (Xigris) 24 mcg/kg/hr.

ANS: A Furosemide will lower the filling pressures and renal perfusion further for the patient with septic shock. The other orders are appropriate. DIF: Cognitive Level: Application REF: 1724-1726 | 1731 | 1733 TOP: Nursing Process: Implementation MSC: NCLEX: Physiological Integrity

11. Which assessment finding obtained by the nurse when caring for a patient with a right radial arterial line indicates a need for the nurse to take action? a. The right hand feels cooler than the left hand. b. The mean arterial pressure (MAP) is 77 mm Hg. c. The system is delivering 3 mL of flush solution per hour. d. The flush bag and tubing were last changed 2 days previously.

ANS: A The change in temperature of the right hand suggests that blood flow to the right hand is impaired. The flush system needs to be changed every 96 hours. A mean arterial pressure (MAP) of 75 mm Hg is normal. Flush systems for hemodynamic monitoring are set up to deliver 3 to 6 mL/hr of flush solution.

1. A 78-kg patient with septic shock has a pulse rate of 120 beats/min with low central venous pressure and pulmonary artery wedge pressure. Urine output has been 30 mL/hr for the past 3 hours. Which order by the health care provider should the nurse question? a. Administer furosemide (Lasix) 40 mg IV. b. Increase normal saline infusion to 250 mL/hr. c. Give hydrocortisone (Solu-Cortef) 100 mg IV. d. Titrate norepinephrine to keep systolic blood pressure (BP) above 90 mm Hg.

ANS: A Furosemide will lower the filling pressures and renal perfusion further for the patient with septic shock. Patients in septic shock require large amounts of fluid replacement. If the patient remains hypotensive after initial volume resuscitation with minimally 30 mL/kg, vasopressors such as norepinephrine may be added. IV corticosteroids may be considered for patients in septic shock who cannot maintain an adequate BP with vasopressor therapy despite fluid resuscitation.

A 78-kg patient with septic shock has a urine output of 30 mL/hr for the past 3 hours. The pulse rate is 120/minute and the central venous pressure and pulmonary artery wedge pressure are low. Which order by the health care provider will the nurse question? a. Give PRN furosemide (Lasix) 40 mg IV. b. Increase normal saline infusion to 250 mL/hr. c. Administer hydrocortisone (Solu-Cortef) 100 mg IV. d. Titrate norepinephrine (Levophed) to keep systolic BP >90 mm Hg.

ANS: A Furosemide will lower the filling pressures and renal perfusion further for the patient with septic shock. The other orders are appropriate.

36. A patient who is receiving positive pressure ventilation is scheduled for a spontaneous breathing trial (SBT). Which finding by the nurse is most important to discuss with the health care provider before starting the SBT? a. New ST segment elevation is noted on the cardiac monitor. b. Enteral feedings are being given through an orogastric tube. c. Scattered rhonchi are heard when auscultating breath sounds. d. HYDROmorphone (Dilaudid) is being used to treat postoperative pain.

ANS: A Myocardial ischemia is a contraindication for ventilator weaning. The ST segment elevation is an indication that weaning should be postponed until further investigation and/or treatment for myocardial ischemia can be done. The other information will also be shared with the health care provider, but ventilator weaning can proceed when opioids are used for pain management, abnormal lung sounds are present, or enteral feedings are being used. DIF: Cognitive Level: Apply (application) REF: 1626 OBJ: Special Questions: Prioritization TOP: Nursing Process: Assessment MSC: NCLEX: Physiological Integrity

11. Which assessment finding obtained by the nurse when caring for a patient with a right radial arterial line indicates a need for the nurse to take immediate action? a. The right hand is cooler than the left hand. b. The mean arterial pressure (MAP) is 77 mm Hg. c. The system is delivering 3 mL of flush solution per hour. d. The flush bag and tubing were last changed 3 days previously.

ANS: A The change in temperature of the left hand suggests that blood flow to the left hand is impaired. The flush system needs to be changed every 96 hours. A mean arterial pressure (MAP) of 75 mm Hg is normal. Flush systems for hemodynamic monitoring are set up to deliver 3 to 6 mL/hour of flush solution. DIF: Cognitive Level: Apply (application) REF: 1606 TOP: Nursing Process: Assessment MSC: NCLEX: Physiological Integrity

19. During change-of-shift report, the nurse is told that a patient has been admitted with dehydration and hypotension after having vomiting and diarrhea for 4 days. Which finding is most important for the nurse to report to the health care provider? a. New onset of confusion c. Heart rate 112 beats/min b. Decreased bowel sounds d. Pale, cool, and dry extremities

ANS: A The changes in mental status are indicative that the patient is in the progressive stage of shock and that rapid intervention is needed to prevent further deterioration. The other information is consistent with compensatory shock.

21. The patient with neurogenic shock is receiving a phenylephrine (Neo-Synephrine) infusion through a left forearm IV. Which assessment information obtained by the nurse indicates a need for immediate action? a. The patient's IV infusion site is cool and pale. b. The patient has warm, dry skin on the extremities. c. The patient has an apical pulse rate of 58 beats/min. d. The patient's urine output has been 28 mL over the last hour.

ANS: A The coldness and pallor at the infusion site suggest extravasation of the phenylephrine. The nurse should discontinue the IV and, if possible, infuse the medication into a central line. An apical pulse of 58 is typical for neurogenic shock but does not indicate an immediate need for nursing intervention. A 28 mL urinary output over 1 hour would require the nurse to monitor the output over the next hour, but an immediate change in therapy is not indicated. Warm, dry skin is consistent with early neurogenic shock, but it does not indicate a need for a change in therapy or immediate action. DIF: Cognitive Level: Application REF: 1733-1734 OBJ: Special Questions: Prioritization TOP: Nursing Process: Assessment MSC: NCLEX: Physiological Integrity

The patient with neurogenic shock is receiving a phenylephrine (Neo-Synephrine) infusion through a left forearm IV. Which assessment information obtained by the nurse indicates a need for immediate action? a. The patient's IV infusion site is cool and pale. b. The patient has warm, dry skin on the extremities. c. The patient has an apical pulse rate of 58 beats/min. d. The patient's urine output has been 28 mL over the last hour.

ANS: A The coldness and pallor at the infusion site suggest extravasation of the phenylephrine. The nurse should discontinue the IV and, if possible, infuse the medication into a central line. An apical pulse of 58 is typical for neurogenic shock but does not indicate an immediate need for nursing intervention. A 28 mL urinary output over 1 hour would require the nurse to monitor the output over the next hour, but an immediate change in therapy is not indicated. Warm, dry skin is consistent with early neurogenic shock, but it does not indicate a need for a change in therapy or immediate action. DIF: Cognitive Level: Application REF: 1733-1734 OBJ: Special Questions: Prioritization TOP: Nursing Process: Assessment MSC: NCLEX: Physiological Integrity

Which information obtained by the nurse when caring for a patient who has cardiogenic shock indicates that the patient may be developing multiple organ dysfunction syndrome (MODS)? a. The patient's serum creatinine level is elevated. b. The patient complains of intermittent chest pressure. c. The patient has crackles throughout both lung fields. d. The patient's extremities are cool and pulses are weak.

ANS: A The elevated serum creatinine level indicates that the patient has renal failure as well as heart failure. The crackles, chest pressure, and cool extremities are all consistent with the patient's diagnosis of cardiogenic shock. DIF: Cognitive Level: Application REF: 1740-1741 TOP: Nursing Process: Assessment MSC: NCLEX: Physiological Integrity

14. Which data collected by the nurse caring for a patient who has cardiogenic shock indicate that the patient may be developing multiple organ dysfunction syndrome (MODS)? a. The patient's serum creatinine level is elevated. b. The patient complains of intermittent chest pressure. c. The patient's extremities are cool and pulses are weak. d. The patient has bilateral crackles throughout lung fields.

ANS: A The elevated serum creatinine level indicates that the patient has renal failure as well as heart failure. The crackles, chest pressure, and cool extremities are all symptoms consistent with the patient's diagnosis of cardiogenic shock.

Which data collected by the nurse caring for a patient who has cardiogenic shock indicate that the patient may be developing multiple organ dysfunction syndrome (MODS)? a. The patient's serum creatinine level is elevated. b. The patient complains of intermittent chest pressure. c. The patient's extremities are cool and pulses are weak. d. The patient has bilateral crackles throughout lung fields.

ANS: A The elevated serum creatinine level indicates that the patient has renal failure as well as heart failure. The crackles, chest pressure, and cool extremities are all consistent with the patient's diagnosis of cardiogenic shock.

22. A patient with respiratory failure has arterial pressure-based cardiac output (APCO) monitoring and is receiving mechanical ventilation with peak end-expiratory pressure (PEEP) of 12 cm H2O. Which information indicates that a change in the ventilator settings may be required? a. The arterial pressure is 90/46. b. The heart rate is 58 beats/minute. c. The stroke volume is increased. d. The stroke volume variation is 12%.

ANS: A The hypotension suggests that the high intrathoracic pressure caused by the PEEP may be decreasing venous return and (potentially) cardiac output. The other assessment data would not be a direct result of PEEP and mechanical ventilation. DIF: Cognitive Level: Apply (application) REF: 1622-1624 TOP: Nursing Process: Evaluation MSC: NCLEX: Physiological Integrity

25. When evaluating a patient with a central venous catheter, the nurse observes that the insertion site is red and tender to touch and the patient's temperature is 101.8° F. What should the nurse plan to do? a. Discontinue the catheter and culture the tip. b. Use the catheter only for fluid administration. c. Change the flush system and monitor the site. d. Check the site more frequently for any swelling.

ANS: A The information indicates that the patient has a local and systemic infection caused by the catheter, and the catheter should be discontinued to avoid further complications such as endocarditis. Changing the flush system, continued monitoring, or using the line for fluids will not help prevent or treat the infection.

18. A patient is treated in the emergency department (ED) for shock of unknown etiology. The first action by the nurse should be to a. administer oxygen. b. attach a cardiac monitor. c. obtain the blood pressure. d. check the level of consciousness.

ANS: A The initial actions of the nurse are focused on the ABCs—airway, breathing, circulation—and administration of oxygen should be done first. The other actions should be accomplished as rapidly as possible after oxygen administration. DIF: Cognitive Level: Application REF: 1729-1731 | 1732 | 1733 | 1736-1737 OBJ: Special Questions: Prioritization TOP: Nursing Process: Implementation MSC: NCLEX: Physiological Integrity

A patient is treated in the emergency department (ED) for shock of unknown etiology. The first action by the nurse should be to a. administer oxygen. b. attach a cardiac monitor. c. obtain the blood pressure. d. check the level of consciousness.

ANS: A The initial actions of the nurse are focused on the ABCs—airway, breathing, circulation—and administration of oxygen should be done first. The other actions should be accomplished as rapidly as possible after oxygen administration. DIF: Cognitive Level: Application REF: 1729-1731 | 1732 | 1733 | 1736-1737 OBJ: Special Questions: Prioritization TOP: Nursing Process: Implementation MSC: NCLEX: Physiological Integrity

A patient is admitted to the emergency department (ED) for shock of unknown etiology. The first action by the nurse should be to a. administer oxygen. b. obtain a 12-lead electrocardiogram (ECG). c. obtain the blood pressure. d. check the level of consciousness.

ANS: A The initial actions of the nurse are focused on the ABCs—airway, breathing, and circulation—and administration of oxygen should be done first. The other actions should be accomplished as rapidly as possible after oxygen administration.

33. A patient who is orally intubated and receiving mechanical ventilation is anxious and is "fighting" the ventilator. Which action should the nurse take next? a. Verbally coach the patient to breathe with the ventilator. b. Sedate the patient with the ordered PRN lorazepam (Ativan). c. Manually ventilate the patient with a bag-valve-mask device. d. Increase the rate for the ordered propofol (Diprivan) infusion.

ANS: A The initial response by the nurse should be to try to decrease the patient's anxiety by coaching the patient about how to coordinate respirations with the ventilator. The other actions may also be helpful if the verbal coaching is ineffective in reducing the patient's anxiety. DIF: Cognitive Level: Apply (application) REF: 1623 OBJ: Special Questions: Prioritization TOP: Nursing Process: Implementation MSC: NCLEX: Physiological Integrity

2. A patient with shock of unknown etiology whose hemodynamic monitoring indicates BP 92/54, pulse 64, and an elevated pulmonary artery wedge pressure has the following collaborative interventions prescribed. Which intervention will the nurse question? a. Infuse normal saline at 250 mL/hr. b. Keep head of bed elevated to 30 degrees. c. Give nitroprusside (Nipride) unless systolic BP <90 mm Hg. d. Administer dobutamine (Dobutrex) to keep systolic BP >90 mm Hg.

ANS: A The patient's elevated pulmonary artery wedge pressure indicates volume excess. A normal saline infusion at 250 mL/hr will exacerbate this. The other actions are appropriate for the patient. DIF: Cognitive Level: Application REF: 1719 | 1721-1722 | 1735 TOP: Nursing Process: Planning MSC: NCLEX: Physiological Integrity

A patient with shock of unknown etiology whose hemodynamic monitoring indicates BP 92/54, pulse 64, and an elevated pulmonary artery wedge pressure has the following collaborative interventions prescribed. Which intervention will the nurse question? a. Infuse normal saline at 250 mL/hr. b. Keep head of bed elevated to 30 degrees. c. Give nitroprusside (Nipride) unless systolic BP <90 mm Hg. d. Administer dobutamine (Dobutrex) to keep systolic BP >90 mm Hg.

ANS: A The patient's elevated pulmonary artery wedge pressure indicates volume excess. A normal saline infusion at 250 mL/hr will exacerbate this. The other actions are appropriate for the patient. DIF: Cognitive Level: Application REF: 1719 | 1721-1722 | 1735 TOP: Nursing Process: Planning MSC: NCLEX: Physiological Integrity

The following interventions are ordered by the health care provider for a patient who has respiratory distress and syncope after eating strawberries. Which will the nurse complete first? a. Start a normal saline infusion. b. Give epinephrine (Adrenalin). c. Start continuous ECG monitoring. d. Give diphenhydramine (Benadryl).

ANS: B Epinephrine rapidly causes peripheral vasoconstriction, dilates the bronchi, and blocks the effects of histamine and reverses the vasodilation, bronchoconstriction, and histamine release that cause the symptoms of anaphylaxis. The other interventions are also appropriate but would not be the first ones completed.

A nurse is caring for a patient with shock of unknown etiology whose hemodynamic monitoring indicates BP 92/54, pulse 64, and an elevated pulmonary artery wedge pressure. Which collaborative intervention ordered by the health care provider should the nurse question? a. Infuse normal saline at 250 mL/hr. b. Keep head of bed elevated to 30 degrees. c. Hold nitroprusside (Nipride) if systolic BP <90 mm Hg. d. Titrate dobutamine (Dobutrex) to keep systolic BP >90 mm Hg.

ANS: A The patient's elevated pulmonary artery wedge pressure indicates volume excess. A saline infusion at 250 mL/hr will exacerbate the volume excess. The other actions are appropriate for the patient.

2. Which preventive actions by the nurse will help limit the development of systemic inflammatory response syndrome (SIRS) in patients admitted to the hospital (select all that apply)? a. Ambulate postoperative patients as soon as possible after surgery. b. Use aseptic technique when manipulating invasive lines or devices. c. Remove indwelling urinary catheters as soon as possible after surgery. d. Administer prescribed antibiotics within 1 hour for patients with possible sepsis. e. Advocate for parenteral nutrition for patients who cannot take in adequate calories.

ANS: A, B, C, D Because sepsis is the most frequent etiology for SIRS, measures to avoid infection such as removing indwelling urinary catheters as soon as possible, use of aseptic technique, and early ambulation should be included in the plan of care. Adequate nutrition is important in preventing SIRS. Enteral, rather than parenteral, nutrition is preferred when patients are unable to take oral feedings because enteral nutrition helps maintain the integrity of the intestine, thus decreasing infection risk. Antibiotics should be given within 1 hour after being prescribed to decrease the risk of sepsis progressing to SIRS.

A patient with neurogenic shock has just arrived in the emergency department after a diving accident. He has a cervical collar in place. Which of the following actions should the nurse take (select all that apply)? a. Prepare to administer atropine IV. b. Obtain baseline body temperature. c. Prepare for intubation and mechanical ventilation. d. Administer large volumes of lactated Ringer's solution. e. Administer high-flow oxygen (100%) by non-rebreather mask.

ANS: A, B, C, E All of the actions are appropriate except to give large volumes of lactated Ringer's solution. The patient with neurogenic shock usually has a normal blood volume, and it is important not to volume overload the patient. In addition, lactated Ringer's solution is used cautiously in all shock situations because the failing liver cannot convert lactate to bicarbonate. DIF: Cognitive Level: Application REF: 1736-1737 OBJ: Special Questions: Alternate Item Format TOP: Nursing Process: Implementation MSC: NCLEX: Physiological Integrity

Which preventive actions by the nurse will help limit the development of systemic inflammatory response syndrome (SIRS) in patients admitted to the hospital (select all that apply)? a. Use aseptic technique when caring for invasive lines or devices. b. Ambulate postoperative patients as soon as possible after surgery. c. Remove indwelling urinary catheters as soon as possible after surgery. d. Advocate for parenteral nutrition for patients who cannot take oral feedings. e. Administer prescribed antibiotics within 1 hour for patients with possible sepsis.

ANS: A, B, C, E Because sepsis is the most frequent etiology for SIRS, measures to avoid infection such as removing indwelling urinary catheters as soon as possible, use of aseptic technique, and early ambulation should be included in the plan of care. Adequate nutrition is important in preventing SIRS. Enteral, rather than parenteral, nutrition is preferred when patients are unable to take oral feedings because enteral nutrition helps maintain the integrity of the intestine, thus decreasing infection risk. Antibiotics should be administered within 1 hour after being prescribed to decrease the risk of sepsis progressing to SIRS.

1. A patient with suspected neurogenic shock after a diving accident has arrived in the emergency department. A cervical collar is in place. Which actions should the nurse take (select all that apply)? a. Prepare to administer atropine IV. b. Obtain baseline body temperature. c. Infuse large volumes of lactated Ringer's solution. d. Provide high-flow O2 (100%) by nonrebreather mask. e. Prepare for emergent intubation and mechanical ventilation.

ANS: A, B, D, E All of the actions are appropriate except to give large volumes of lactated Ringer's solution. The patient with neurogenic shock usually has a normal blood volume, and it is important not to volume overload the patient. In addition, lactated Ringer's solution is used cautiously in all shock situations because an ischemic liver cannot convert lactate to bicarbonate.

A patient with suspected neurogenic shock after a diving accident has arrived in the emergency department. A cervical collar is in place. Which actions should the nurse take (select all that apply)? a. Prepare to administer atropine IV. b. Obtain baseline body temperature. c. Infuse large volumes of lactated Ringer's solution. d. Provide high-flow oxygen (100%) by non-rebreather mask. e. Prepare for emergent intubation and mechanical ventilation.

ANS: A, B, D, E All of the actions are appropriate except to give large volumes of lactated Ringer's solution. The patient with neurogenic shock usually has a normal blood volume, and it is important not to volume overload the patient. In addition, lactated Ringer's solution is used cautiously in all shock situations because the failing liver cannot convert lactate to bicarbonate.

4. After surgery for an abdominal aortic aneurysm, a patient's central venous pressure (CVP) monitor indicates low pressures. Which action should the nurse take? a. Administer IV diuretic medications. b. Increase the IV fluid infusion per protocol. c. Increase the infusion rate of IV vasodilators. d. Elevate the head of the patient's bed to 45 degrees.

ANS: B A low CVP indicates hypovolemia and a need for an increase in the infusion rate. Diuretic administration will contribute to hypovolemia and elevation of the head or increasing vasodilators may decrease cerebral perfusion.

4. Following surgery for an abdominal aortic aneurysm, a patient's central venous pressure (CVP) monitor indicates low pressures. Which action is a priority for the nurse to take? a. Administer IV diuretic medications. b. Increase the IV fluid infusion per protocol. c. Document the CVP and continue to monitor. d. Elevate the head of the patient's bed to 45 degrees.

ANS: B A low CVP indicates hypovolemia and a need for an increase in the infusion rate. Diuretic administration will contribute to hypovolemia and elevation of the head may decrease cerebral perfusion. Documentation and continued monitoring is an inadequate response to the low CVP. DIF: Cognitive Level: Apply (application) REF: 1609 TOP: Nursing Process: Planning MSC: NCLEX: Physiological Integrity

11. Norepinephrine has been prescribed for a patient who was admitted with dehydration and hypotension. Which patient data indicate that the nurse should consult with the health care provider before starting the norepinephrine? a. The patient is receiving low dose dopamine. b. The patient's central venous pressure is 3 mm Hg. c. The patient is in sinus tachycardia at 120 beats/min. d. The patient has had no urine output since being admitted.

ANS: B Adequate fluid administration is essential before giving vasopressors to patients with hypovolemic shock. The patient's low central venous pressure indicates a need for more volume replacement. The other patient data are not contraindications to norepinephrine administration.

24. After change-of-shift report in the progressive care unit, who should the nurse care for first? a. Patient who had an inferior myocardial infarction 2 days ago and has crackles in the lung bases b. Patient with suspected urosepsis who has new orders for urine and blood cultures and antibiotics c. Patient who had a T5 spinal cord injury 1 week ago and currently has a heart rate of 54 beats/minute d. Patient admitted with anaphylaxis 3 hours ago who now has clear lung sounds and a blood pressure of 108/58 mm Hg

ANS: B Antibiotics should be given within the first hour for patients who have sepsis or suspected sepsis in order to prevent progression to systemic inflammatory response syndrome and septic shock. The data on the other patients indicate that they are more stable. Crackles heard only at the lung bases do not require immediate intervention in a patient who has had a myocardial infarction. Mild bradycardia does not usually require atropine in patients who have a spinal cord injury. The findings for the patient admitted with anaphylaxis indicate resolution of bronchospasm and hypotension.

Which of these findings is the best indicator that the fluid resuscitation for a patient with hypovolemic shock has been successful? a. Hemoglobin is within normal limits. b. Urine output is 60 mL over the last hour. c. Pulmonary artery wedge pressure (PAWP) is normal. d. Mean arterial pressure (MAP) is 65 mm Hg.

ANS: B Assessment of end organ perfusion, such as an adequate urine output, is the best indicator that fluid resuscitation has been successful. The hemoglobin level, PAWP, and MAP are useful in determining the effects of fluid administration, but they are not as useful as data indicating good organ perfusion. DIF: Cognitive Level: Application REF: 1733-1735 TOP: Nursing Process: Evaluation MSC: NCLEX: Physiological Integrity

Which finding is the best indicator that the fluid resuscitation for a patient with hypovolemic shock has been effective? a. Hemoglobin is within normal limits. b. Urine output is 60 mL over the last hour. c. Central venous pressure (CVP) is normal. d. Mean arterial pressure (MAP) is 72 mm Hg.

ANS: B Assessment of end organ perfusion, such as an adequate urine output, is the best indicator that fluid resuscitation has been successful. The hemoglobin level, CVP, and MAP are useful in determining the effects of fluid administration, but they are not as useful as data indicating good organ perfusion.

Which information about a patient who is receiving vasopressin (Pitressin) to treat septic shock is most important for the nurse to communicate to the heath care provider? a. The patient's heart rate is 108 beats/min. b. The patient is complaining of chest pain. c. The patient's peripheral pulses are weak. d. The patient's urine output is 15 mL/hr.

ANS: B Because vasopressin is a potent vasoconstrictor, it may decrease coronary artery perfusion. The other information is consistent with the patient's diagnosis and should be reported to the health care provider but does not indicate a need for a change in therapy. DIF: Cognitive Level: Application REF: 1735-1736 OBJ: Special Questions: Prioritization TOP: Nursing Process: Assessment MSC: NCLEX: Physiological Integrity

1. A patient who has been in the intensive care unit for 4 days has disturbed sensory perception from sleep deprivation. Which action should the nurse include in the plan of care? a. Administer prescribed sedatives or opioids at bedtime to promote sleep. b. Cluster nursing activities so that the patient has uninterrupted rest periods. c. Silence the alarms on the cardiac monitors to allow 30- to 40-minute naps. d. Eliminate assessments between 2200 and 0600 to allow uninterrupted sleep.

ANS: B Clustering nursing activities and providing uninterrupted rest periods will minimize sleep-cycle disruption. Sedative and opioid medications tend to decrease the amount of rapid eye movement (REM) sleep and can contribute to sleep disturbance and disturbed sensory perception. Silencing the alarms on the cardiac monitors would be unsafe in a critically ill patient, as would discontinuing all assessments during the night.

1. A 68-year-old patient has been in the intensive care unit for 4 days and has a nursing diagnosis of disturbed sensory perception related to sleep deprivation. Which action should the nurse include in the plan of care? a. Administer prescribed sedatives or opioids at bedtime to promote sleep. b. Cluster nursing activities so that the patient has uninterrupted rest periods. c. Silence the alarms on the cardiac monitors to allow 30- to 40-minute naps. d. Eliminate assessments between 0100 and 0600 to allow uninterrupted sleep.

ANS: B Clustering nursing activities and providing uninterrupted rest periods will minimize sleep-cycle disruption. Sedative and opioid medications tend to decrease the amount of rapid eye movement (REM) sleep and can contribute to sleep disturbance and disturbed sensory perception. Silencing the alarms on the cardiac monitors would be unsafe in a critically ill patient, as would discontinuing assessments during the night. DIF: Cognitive Level: Apply (application) REF: 1601 TOP: Nursing Process: Planning MSC: NCLEX: Psychosocial Integrity

12. The central venous oxygen saturation (ScvO2) is decreasing in a patient who has severe pancreatitis. To determine the possible cause of the decreased ScvO2 , the nurse assesses the patient's a. lipase level. c. urinary output. b. temperature. d. body mass index.

ANS: B Elevated temperature increases metabolic demands and O2 use by tissues, resulting in a drop in O2 saturation of central venous blood. Information about the patient's body mass index, urinary output, and lipase will not help in determining the cause of the patient's drop in ScvO2

12. The central venous oxygen saturation (ScvO2) is decreasing in a patient who has severe pancreatitis. To determine the possible cause of the decreased ScvO2, the nurse assesses the patient's a. lipase. b. temperature. c. urinary output. d. body mass index.

ANS: B Elevated temperature increases metabolic demands and oxygen use by tissues, resulting in a drop in oxygen saturation of central venous blood. Information about the patient's body mass index, urinary output, and lipase will not help in determining the cause of the patient's drop in ScvO2. DIF: Cognitive Level: Apply (application) REF: 1609 TOP: Nursing Process: Assessment MSC: NCLEX: Physiological Integrity

22. The following therapies are prescribed by the health care provider for a patient who has respiratory distress and syncope after a bee sting. Which will the nurse administer first? a. normal saline infusion b. epinephrine (Adrenalin) c. dexamethasone (Decadron) d. diphenhydramine (Benadryl)

ANS: B Epinephrine rapidly causes peripheral vasoconstriction, dilates the bronchi, and blocks the effects of histamine and reverses the vasodilation, bronchoconstriction, and histamine release that cause the symptoms of anaphylaxis. The other interventions also are appropriate but would not be the first ones administered. DIF: Cognitive Level: Application REF: 1736-1737 OBJ: Special Questions: Prioritization TOP: Nursing Process: Implementation MSC: NCLEX: Physiological Integrity

The following therapies are prescribed by the health care provider for a patient who has respiratory distress and syncope after a bee sting. Which will the nurse administer first? a. normal saline infusion b. epinephrine (Adrenalin) c. dexamethasone (Decadron) d. diphenhydramine (Benadryl)

ANS: B Epinephrine rapidly causes peripheral vasoconstriction, dilates the bronchi, and blocks the effects of histamine and reverses the vasodilation, bronchoconstriction, and histamine release that cause the symptoms of anaphylaxis. The other interventions also are appropriate but would not be the first ones administered. DIF: Cognitive Level: Application REF: 1736-1737 OBJ: Special Questions: Prioritization TOP: Nursing Process: Implementation MSC: NCLEX: Physiological Integrity

3. While close family members are visiting, a patient has a respiratory arrest, and resuscitation is started. Which action by the nurse is best? a. Tell the family members that watching the resuscitation will be very stressful. b. Ask family members if they wish to remain in the room during the resuscitation. c. Take the family members quickly out of the patient room and remain with them. d. Assign a staff member to wait with family members just outside the patient room.

ANS: B Evidence indicates that many family members want the option of remaining in the room during procedures such as cardiopulmonary resuscitation (CPR) and that this decreases anxiety and facilitates grieving. The other options may be appropriate if the family decides not to remain with the patient.

6. The intensive care unit (ICU) nurse educator determines that teaching a new staff nurse about arterial pressure monitoring has been effective when the nurse a. balances and calibrates the monitoring equipment every 2 hours. b. positions the zero-reference stopcock line level with the phlebostatic axis. c. ensures that the patient is supine with the head of the bed flat for all readings. d. rechecks the location of the phlebostatic axis with changes in the patient's position.

ANS: B For accurate measurement of pressures, the zero-reference level should be at the phlebostatic axis. There is no need to rebalance and recalibrate monitoring equipment every 2 hours. Accurate hemodynamic readings are possible with the patient's head raised to 45 degrees or in the prone position. The anatomic position of the phlebostatic axis does not change when patients are repositioned.

6. The intensive care unit (ICU) nurse educator will determine that teaching about arterial pressure monitoring for a new staff nurse has been effective when the nurse a. balances and calibrates the monitoring equipment every 2 hours. b. positions the zero-reference stopcock line level with the phlebostatic axis. c. ensures that the patient is supine with the head of the bed flat for all readings. d. rechecks the location of the phlebostatic axis when changing the patient's position.

ANS: B For accurate measurement of pressures, the zero-reference level should be at the phlebostatic axis. There is no need to rebalance and recalibrate monitoring equipment hourly. Accurate hemodynamic readings are possible with the patient's head raised to 45 degrees or in the prone position. The anatomic position of the phlebostatic axis does not change when patients are repositioned. DIF: Cognitive Level: Apply (application) REF: 1605 TOP: Nursing Process: Evaluation MSC: NCLEX: Safe and Effective Care Environment

3. A patient with massive trauma and possible spinal cord injury is admitted to the emergency department (ED). Which assessment finding by the nurse will help confirm a diagnosis of neurogenic shock? a. Inspiratory crackles c. Cool, clammy extremities b. Heart rate 45 beats/min d. Temperature 101.2°F (38.4°C)

ANS: B Neurogenic shock is characterized by hypotension and bradycardia. The other findings would be more consistent with other types of shock.

3. While family members are visiting, a patient has a respiratory arrest and is being resuscitated. Which action by the nurse is best? a. Tell the family members that watching the resuscitation will be very stressful. b. Ask family members if they wish to remain in the room during the resuscitation. c. Take the family members quickly out of the patient room and remain with them. d. Assign a staff member to wait with family members just outside the patient room.

ANS: B Research indicates that family members want the option of remaining in the room during procedures such as cardiopulmonary resuscitation (CPR) and that this decreases anxiety and facilitates grieving. The other options may be appropriate if the family decides not to remain with the patient. DIF: Cognitive Level: Apply (application) REF: 1602 TOP: Nursing Process: Implementation MSC: NCLEX: Psychosocial Integrity

17. When caring for a patient who has septic shock, which assessment finding is most important for the nurse to report to the health care provider? a. BP 92/56 mm Hg b. Skin cool and clammy c. Apical pulse 118 beats/min d. Arterial oxygen saturation 91%

ANS: B Since patients in the early stage of septic shock have warm and dry skin, the patient's cool and clammy skin indicates that shock is progressing. The other information also will be reported, but does not indicate deterioration of the patient's status. DIF: Cognitive Level: Application REF: 1723 OBJ: Special Questions: Prioritization TOP: Nursing Process: Assessment MSC: NCLEX: Physiological Integrity

When caring for a patient who has septic shock, which assessment finding is most important for the nurse to report to the health care provider? a. BP 92/56 mm Hg b. Skin cool and clammy c. Apical pulse 118 beats/min d. Arterial oxygen saturation 91%

ANS: B Since patients in the early stage of septic shock have warm and dry skin, the patient's cool and clammy skin indicates that shock is progressing. The other information also will be reported, but does not indicate deterioration of the patient's status. DIF: Cognitive Level: Application REF: 1723 OBJ: Special Questions: Prioritization TOP: Nursing Process: Assessment MSC: NCLEX: Physiological Integrity

Which intervention will the nurse include in the plan of care for a patient who has cardiogenic shock? a. Check temperature every 2 hours. b. Monitor breath sounds frequently. c. Maintain patient in supine position. d. Assess skin for flushing and itching.

ANS: B Since pulmonary congestion and dyspnea are characteristics of cardiogenic shock, the nurse should assess the breath sounds frequently. The head of the bed is usually elevated to decrease dyspnea in patients with cardiogenic shock. Elevated temperature and flushing or itching of the skin are not typical of cardiogenic shock.

A patient with cardiogenic shock has the following vital signs: BP 86/50, pulse 126, respirations 30. The PAWP is increased and cardiac output is low. The nurse will anticipate a. infusion of 5% human albumin. b. administration of furosemide (Lasix) IV. c. titration of an epinephrine (Adrenalin) drip. d. administration of hydrocortisone (SoluCortef).

ANS: B The PAWP indicates that the patient's preload is elevated and furosemide is indicated to reduce the preload and improve cardiac output. Epinephrine would further increase heart rate and myocardial oxygen demand. Normal saline infusion would increase the PAWP further. Hydrocortisone might be used for septic or anaphylactic shock. DIF: Cognitive Level: Application REF: 1735 | 1736 TOP: Nursing Process: Planning MSC: NCLEX: Physiological Integrity

7. A patient with cardiogenic shock has the following vital signs: BP 102/50, pulse 128, respirations 28. The pulmonary artery wedge pressure (PAWP) is increased, and cardiac output is low. The nurse will anticipate an order for which medication? a. 5% albumin infusion c. epinephrine (Adrenalin) drip b. furosemide (Lasix) IV d. hydrocortisone (Solu-Cortef)

ANS: B The PAWP indicates that the patient's preload is elevated, and furosemide is indicated to reduce the preload and improve cardiac output. Epinephrine would further increase the heart rate and myocardial oxygen demand. 5% albumin would also increase the PAWP. Hydrocortisone might be considered for septic or anaphylactic shock.

A patient with cardiogenic shock has the following vital signs: BP 102/50, pulse 128, respirations 28. The pulmonary artery wedge pressure (PAWP) is increased and cardiac output is low. The nurse will anticipate an order for which medication? a. 5% human albumin b. Furosemide (Lasix) IV c. Epinephrine (Adrenalin) drip d. Hydrocortisone (Solu-Cortef)

ANS: B The PAWP indicates that the patient's preload is elevated, and furosemide is indicated to reduce the preload and improve cardiac output. Epinephrine would further increase heart rate and myocardial oxygen demand. 5% human albumin would also increase the PAWP. Hydrocortisone might be considered for septic or anaphylactic shock.

33. The nurse educator is evaluating the performance of a new registered nurse (RN) who is providing care to a patient who is receiving mechanical ventilation with 15 cm H2O of peak end- expiratory pressure (PEEP). Which action indicates that the new RN is safe? a. The RN plans to suction the patient every 1 to 2 hours. b. The RN uses a closed-suction technique to suction the patient. c. The RN tapes the connection between the ventilator tubing and the ET. d. The RN changes the ventilator circuit tubing routinely every 48 hours.

ANS: B The closed-suction technique is used when patients require high levels of PEEP (>10 cm H2O) to prevent the loss of PEEP that occurs when disconnecting the patient from the ventilator. Suctioning should not be scheduled routinely, but it should be done only when patient assessment data indicate the need for suctioning. Taping connections between the ET and ventilator tubing would restrict the ability of the tubing to swivel in response to patient repositioning. Ventilator tubing changes increase the risk for ventilator-associated pneumonia and are not indicated routinely.

34. The nurse educator is evaluating the performance of a new registered nurse (RN) who is providing care to a patient who is receiving mechanical ventilation with 15 cm H2O of peak end-expiratory pressure (PEEP). Which action indicates that the new RN is safe? a. The RN plans to suction the patient every 1 to 2 hours. b. The RN uses a closed-suction technique to suction the patient. c. The RN tapes connection between the ventilator tubing and the ET. d. The RN changes the ventilator circuit tubing routinely every 48 hours.

ANS: B The closed-suction technique is used when patients require high levels of PEEP (>10 cm H2O) to prevent the loss of PEEP that occurs when disconnecting the patient from the ventilator. Suctioning should not be scheduled routinely, but it should be done only when patient assessment data indicate the need for suctioning. Taping connections between the ET and the ventilator tubing would restrict the ability of the tubing to swivel in response to patient repositioning. Ventilator tubing changes increase the risk for ventilator-associated pneumonia (VAP) and are not indicated routinely. DIF: Cognitive Level: Apply (application) REF: 1616 OBJ: Special Questions: Delegation TOP: Nursing Process: Implementation MSC: NCLEX: Safe and Effective Care Environment

12. When the nurse is assessing a patient who is receiving a nitroprusside (Nipride) infusion to treat cardiogenic shock, which finding indicates that the medication is effective? a. No heart murmur is audible. b. Skin is warm, pink, and dry. c. Troponin level is decreased. d. Blood pressure is 90/40 mm Hg.

ANS: B Warm, pink, and dry skin indicates that perfusion to tissues is improved. Since nitroprusside is a vasodilator, the blood pressure may be low even if the medication is effective. Absence of a heart murmur and a decrease in troponin level are not indicators of improvement in shock. DIF: Cognitive Level: Application REF: 1721 | 1723 | 1733-1735 TOP: Nursing Process: Evaluation MSC: NCLEX: Physiological Integrity

When the nurse is assessing a patient who is receiving a nitroprusside (Nipride) infusion to treat cardiogenic shock, which finding indicates that the medication is effective? a. No heart murmur is audible. b. Skin is warm, pink, and dry. c. Troponin level is decreased. d. Blood pressure is 90/40 mm Hg.

ANS: B Warm, pink, and dry skin indicates that perfusion to tissues is improved. Since nitroprusside is a vasodilator, the blood pressure may be low even if the medication is effective. Absence of a heart murmur and a decrease in troponin level are not indicators of improvement in shock. DIF: Cognitive Level: Application REF: 1721 | 1723 | 1733-1735 TOP: Nursing Process: Evaluation MSC: NCLEX: Physiological Integrity

After receiving 2 L of normal saline, the central venous pressure for a patient who has septic shock is 10 mm Hg, but the blood pressure is still 82/40 mm Hg. The nurse will anticipate an order for a. nitroglycerine (Tridil). b. norepinephrine (Levophed). c. sodium nitroprusside (Nipride). d. methylprednisolone (Solu-Medrol).

ANS: B When fluid resuscitation is unsuccessful, vasopressor drugs are administered to increase the systemic vascular resistance (SVR) and blood pressure, and improve tissue perfusion. Nitroglycerin would decrease the preload and further drop cardiac output and BP. Methylprednisolone (Solu-Medrol) is considered if blood pressure does not respond first to fluids and vasopressors. Nitroprusside is an arterial vasodilator and would further decrease SVR.

13. An intraaortic balloon pump (IABP) is being used for a patient who is in cardiogenic shock. Which assessment data indicate to the nurse that the goals of treatment with the IABP are being met? a. Urine output of 25 mL/hr b. Heart rate of 110 beats/minute c. Cardiac output (CO) of 5 L/min d. Stroke volume (SV) of 40 mL/beat

ANS: C A CO of 5 L/min is normal and indicates that the IABP has been successful in treating the shock. The low SV signifies continued cardiogenic shock. The tachycardia and low urine output also suggest continued cardiogenic shock. DIF: Cognitive Level: Apply (application) REF: 1603 TOP: Nursing Process: Evaluation MSC: NCLEX: Physiological Integrity

8. The emergency department (ED) receives notification that a patient who has just been in an automobile accident is being transported to your facility with anticipated arrival in 1 minute. In preparation for the patient's arrival, the nurse will obtain a. 500 mL of 5% albumin. b. lactated Ringer's solution. c. two 14-gauge IV catheters. d. dopamine (Intropin) infusion.

ANS: C A patient with multiple trauma may require fluid resuscitation to prevent or treat hypovolemic shock, so the nurse will anticipate the need for 2 large bore IV lines to administer normal saline. Lactated Ringer's solution should be used cautiously and will not be ordered until the patient has been assessed for possible liver abnormalities. Although colloids may sometimes be used for volume expansion, crystalloids should be used as the initial therapy for fluid resuscitation. Vasopressor infusion is not used as the initial therapy for hypovolemic shock. DIF: Cognitive Level: Application REF: 1731 | 1732 | 1733 TOP: Nursing Process: Planning MSC: NCLEX: Physiological Integrity

The emergency department (ED) receives notification that a patient who has just been in an automobile accident is being transported to your facility with anticipated arrival in 1 minute. In preparation for the patient's arrival, the nurse will obtain a. 500 mL of 5% albumin. b. lactated Ringer's solution. c. two 14-gauge IV catheters. d. dopamine (Intropin) infusion.

ANS: C A patient with multiple trauma may require fluid resuscitation to prevent or treat hypovolemic shock, so the nurse will anticipate the need for 2 large bore IV lines to administer normal saline. Lactated Ringer's solution should be used cautiously and will not be ordered until the patient has been assessed for possible liver abnormalities. Although colloids may sometimes be used for volume expansion, crystalloids should be used as the initial therapy for fluid resuscitation. Vasopressor infusion is not used as the initial therapy for hypovolemic shock. DIF: Cognitive Level: Application REF: 1731 | 1732 | 1733 TOP: Nursing Process: Planning MSC: NCLEX: Physiological Integrity

20. The nurse notes thick, white secretions in the endotracheal tube (ET) of a patient who is receiving mechanical ventilation. Which intervention will most directly treat this finding? a. Reposition the patient every 1 to 2 hours. b. Increase suctioning frequency to every hour. c. Add additional water to the patient's enteral feedings. d. Instill 5 mL of sterile saline into the ET before suctioning.

ANS: C Because the patient's secretions are thick, better hydration is indicated. Suctioning every hour without any specific evidence for the need will increase the incidence of mucosal trauma and would not address the etiology of the ineffective airway clearance. Instillation of saline does not liquefy secretions and may decrease the SpO2 . Repositioning the patient is appropriate but will not decrease the thickness of secretions.

20. The nurse notes thick, white secretions in the endotracheal tube (ET) of a patient who is receiving mechanical ventilation. Which intervention will be most effective in addressing this problem? a. Increase suctioning to every hour. b. Reposition the patient every 1 to 2 hours. c. Add additional water to the patient's enteral feedings. d. Instill 5 mL of sterile saline into the ET before suctioning.

ANS: C Because the patient's secretions are thick, better hydration is indicated. Suctioning every hour without any specific evidence for the need will increase the incidence of mucosal trauma and would not address the etiology of the ineffective airway clearance. Instillation of saline does not liquefy secretions and may decrease the SpO2. Repositioning the patient is appropriate but will not decrease the thickness of secretions. DIF: Cognitive Level: Apply (application) REF: 1617 TOP: Nursing Process: Implementation MSC: NCLEX: Physiological Integrity

Which finding about a patient who is receiving vasopressin (Pitressin) to treat septic shock is most important for the nurse to communicate to the health care provider? a. The patient's urine output is 18 mL/hr. b. The patient's heart rate is 110 beats/minute. c. The patient is complaining of chest pain. d. The patient's peripheral pulses are weak.

ANS: C Because vasopressin is a potent vasoconstrictor, it may decrease coronary artery perfusion. The other information is consistent with the patient's diagnosis and should be reported to the health care provider but does not indicate a need for a change in therapy.

18. The nurse notes premature ventricular contractions (PVCs) while suctioning a patient's endotracheal tube. Which action by the nurse is a priority? a. Decrease the suction pressure to 80 mm Hg. b. Document the dysrhythmia in the patient's chart. c. Stop and ventilate the patient with 100% oxygen. d. Give antidysrhythmic medications per protocol.

ANS: C Dysrhythmias during suctioning may indicate hypoxemia or sympathetic nervous system stimulation. The nurse should stop suctioning and ventilate the patient with 100% oxygen. Lowering the suction pressure will decrease the effectiveness of suctioning without improving the hypoxemia. Because the PVCs occurred during suctioning, there is no need for antidysrhythmic medications (which may have adverse effects) unless they recur when the suctioning is stopped and patient is well oxygenated. DIF: Cognitive Level: Apply (application) REF: 1616 OBJ: Special Questions: Prioritization TOP: Nursing Process: Implementation MSC: NCLEX: Physiological Integrity

14. The nurse is caring for a patient who has an intraaortic balloon pump in place. Which action should be included in the plan of care? a. Position the patient supine at all times. b. Avoid the use of anticoagulant medications. c. Measure the patient's urinary output every hour. d. Provide passive range of motion for all extremities.

ANS: C Monitoring urine output will help determine whether the patient's cardiac output has improved and also help monitor for balloon displacement. The head of the bed can be elevated up to 30 degrees. Heparin is used to prevent thrombus formation. Limited movement is allowed for the extremity with the balloon insertion site to prevent displacement of the balloon. DIF: Cognitive Level: Apply (application) REF: 1613 TOP: Nursing Process: Planning MSC: NCLEX: Physiological Integrity

3. A patient with massive trauma and possible spinal cord injury is admitted to the emergency department (ED). Which finding by the nurse will help confirm a diagnosis of neurogenic shock? a. Cool, clammy skin b. Inspiratory crackles c. Apical heart rate 48 beats/min d. Temperature 101.2° F (38.4° C)

ANS: C Neurogenic shock is characterized by hypotension and bradycardia. The other findings would be more consistent with other types of shock. DIF: Cognitive Level: Comprehension REF: 1721-1722 | 1723 TOP: Nursing Process: Assessment MSC: NCLEX: Physiological Integrity

A patient with massive trauma and possible spinal cord injury is admitted to the emergency department (ED). Which finding by the nurse will help confirm a diagnosis of neurogenic shock? a. Cool, clammy skin b. Inspiratory crackles c. Apical heart rate 48 beats/min d. Temperature 101.2° F (38.4° C)

ANS: C Neurogenic shock is characterized by hypotension and bradycardia. The other findings would be more consistent with other types of shock. DIF: Cognitive Level: Comprehension REF: 1721-1722 | 1723 TOP: Nursing Process: Assessment MSC: NCLEX: Physiological Integrity

A 19-year-old patient with massive trauma and possible spinal cord injury is admitted to the emergency department (ED). Which assessment finding by the nurse will help confirm a diagnosis of neurogenic shock? a. Inspiratory crackles. b. Cool, clammy extremities. c. Apical heart rate 45 beats/min. d. Temperature 101.2° F (38.4° C).

ANS: C Neurogenic shock is characterized by hypotension and bradycardia. The other findings would be more consistent with other types of shock

An older patient with cardiogenic shock is cool and clammy and hemodynamic monitoring indicates a high systemic vascular resistance (SVR). Which intervention should the nurse anticipate doing next? a. Increase the rate for the dopamine (Intropin) infusion. b. Decrease the rate for the nitroglycerin (Tridil) infusion. c. Increase the rate for the sodium nitroprusside (Nipride) infusion. d. Decrease the rate for the 5% dextrose in normal saline (D5/.9 NS) infusion.

ANS: C Nitroprusside is an arterial vasodilator and will decrease the SVR and afterload, which will improve cardiac output. Changes in the D5/.9 NS and nitroglycerin infusions will not directly decrease SVR. Increasing the dopamine will tend to increase SVR.

5. When caring for a patient with pulmonary hypertension, which parameter is most appropriate for the nurse to monitor to evaluate the effectiveness of the treatment? a. Central venous pressure (CVP) b. Systemic vascular resistance (SVR) c. Pulmonary vascular resistance (PVR) d. Pulmonary artery wedge pressure (PAWP)

ANS: C PVR is a major contributor to pulmonary hypertension, and a decrease would indicate that pulmonary hypertension was improving. The other parameters also may be monitored but do not directly assess for pulmonary hypertension. DIF: Cognitive Level: Apply (application) REF: 1603-1604 TOP: Nursing Process: Evaluation MSC: NCLEX: Physiological Integrity

16. When the nurse educator is evaluating the skills of a new registered nurse (RN) caring for patients experiencing shock, which action by the new RN indicates a need for more education? a. Placing the pulse oximeter on the ear for a patient with septic shock b. Keeping the head of the bed flat for a patient with hypovolemic shock c. Maintaining a cool room temperature for a patient with neurogenic shock d. Increasing the nitroprusside infusion rate for a patient with a very high SVR

ANS: C Patients with neurogenic shock have poikilothermia. The room temperature should be kept warm to avoid hypothermia. The other actions by the new RN are appropriate.

16. When the charge nurse is evaluating the skills of a new RN, which action by the new RN indicates a need for more education in the care of patients with shock? a. Placing the pulse oximeter on the ear for a patient with septic shock b. Keeping the head of the bed flat for a patient with hypovolemic shock c. Decreasing the room temperature to 68° F for a patient with neurogenic shock d. Increasing the nitroprusside (Nipride) infusion rate for a patient with a high SVR

ANS: C Patients with neurogenic shock may have poikilothermia. The room temperature should be kept warm to avoid hypothermia. The other actions by the new RN are appropriate. DIF: Cognitive Level: Application REF: 1721-1722 | 1724 OBJ: Special Questions: Delegation TOP: Nursing Process: Evaluation MSC: NCLEX: Safe and Effective Care Environment

When the charge nurse is evaluating the skills of a new RN, which action by the new RN indicates a need for more education in the care of patients with shock? a. Placing the pulse oximeter on the ear for a patient with septic shock b. Keeping the head of the bed flat for a patient with hypovolemic shock c. Decreasing the room temperature to 68° F for a patient with neurogenic shock d. Increasing the nitroprusside (Nipride) infusion rate for a patient with a high SVR

ANS: C Patients with neurogenic shock may have poikilothermia. The room temperature should be kept warm to avoid hypothermia. The other actions by the new RN are appropriate. DIF: Cognitive Level: Application REF: 1721-1722 | 1724 OBJ: Special Questions: Delegation TOP: Nursing Process: Evaluation MSC: NCLEX: Safe and Effective Care Environment

6. To evaluate the effectiveness of the omeprazole (Prilosec) being administered to a patient with systemic inflammatory response syndrome (SIRS), which assessment will the nurse make? a. Auscultate bowel sounds. b. Ask the patient about nausea. c. Monitor stools for occult blood. d. Check for abdominal distention.

ANS: C Proton pump inhibitors are given to decrease the risk for stress ulcers in critically ill patients. The other assessments also will be done, but these will not help in determining the effectiveness of the omeprazole administration. DIF: Cognitive Level: Application REF: 1735-1737 | 1742-1743 TOP: Nursing Process: Evaluation MSC: NCLEX: Physiological Integrity

To evaluate the effectiveness of the omeprazole (Prilosec) being administered to a patient with systemic inflammatory response syndrome (SIRS), which assessment will the nurse make? a. Auscultate bowel sounds. b. Ask the patient about nausea. c. Monitor stools for occult blood. d. Check for abdominal distention.

ANS: C Proton pump inhibitors are given to decrease the risk for stress ulcers in critically ill patients. The other assessments also will be done, but these will not help in determining the effectiveness of the omeprazole administration. DIF: Cognitive Level: Application REF: 1735-1737 | 1742-1743 TOP: Nursing Process: Evaluation MSC: NCLEX: Physiological Integrity

6. To evaluate the effectiveness of the pantoprazole (Protonix) ordered for a patient with systemic inflammatory response syndrome (SIRS), which assessment will the nurse perform? a. Auscultate bowel sounds. c. Check stools for occult blood. b. Ask the patient about nausea. d. Palpate for abdominal tenderness.

ANS: C Proton pump inhibitors are given to decrease the risk for stress ulcers in critically ill patients. The other assessments will also be done, but these will not help in determining the effectiveness of the pantoprazole administration.

23. A nurse is weaning a 68-kg patient who has chronic obstructive pulmonary disease (COPD) from mechanical ventilation. Which patient assessment finding indicates that the weaning protocol should be stopped? a. The patient's heart rate is 97 beats/min. b. The patient's oxygen saturation is 93%. c. The patient respiratory rate is 32 breaths/min. d. The patient's spontaneous tidal volume is 450 mL.

ANS: C Tachypnea is a sign that the patient's work of breathing is too high to allow weaning to proceed. The patient's heart rate is within normal limits, but the nurse should continue to monitor it. An O2 saturation of 93% is acceptable for a patient with COPD. A spontaneous tidal volume of 450 mL is within the acceptable range.

18. A patient is admitted to the emergency department (ED) for shock of unknown etiology. The first action by the nurse should be to a. obtain the blood pressure. b. check the level of orientation. c. administer supplemental oxygen. d. obtain a 12-lead electrocardiogram.

ANS: C The initial actions of the nurse are focused on the ABCs—airway, breathing, and circulation—and administration of O2 should be done first. The other actions should be accomplished as rapidly as possible after providing O2.

The patient with neurogenic shock is receiving a phenylephrine (Neo-Synephrine) infusion through a right forearm IV. Which assessment finding obtained by the nurse indicates a need for immediate action? a. The patient's heart rate is 58 beats/minute. b. The patient's extremities are warm and dry. c. The patient's IV infusion site is cool and pale. d. The patient's urine output is 28 mL over the last hour.

ANS: C The coldness and pallor at the infusion site suggest extravasation of the phenylephrine. The nurse should discontinue the IV and, if possible, infuse the medication into a central line. An apical pulse of 58 is typical for neurogenic shock but does not indicate an immediate need for nursing intervention. A 28-mL urinary output over 1 hour would require the nurse to monitor the output over the next hour, but an immediate change in therapy is not indicated. Warm, dry skin is consistent with early neurogenic shock, but it does not indicate a need for a change in therapy or immediate action.

36. After change-of-shift report on a ventilator weaning unit, which patient should the nurse assess first? a. Patient who failed a spontaneous breathing trial and has been placed in a rest mode on the ventilator b. Patient who is intubated and has continuous partial pressure end-tidal CO2 (PETCO2) monitoring c. Patient who was successfully weaned and extubated 4 hours ago and has no urine output for the last 6 hours d. Patient with a central venous O2 saturation (ScvO2) of 69% while on bilevel positive airway pressure (BiPAP)

ANS: C The decreased urine output may indicate acute kidney injury or that the patient's cardiac output and perfusion of vital organs have decreased. Any of these causes would require rapid action. The data about the other patients indicate that their conditions are stable and do not require immediate assessment or changes in their care. Continuous PETCO2 monitoring is frequently used when patients are intubated. The rest mode should be used to allow patient recovery after a failed SBT, and an ScvO2 of 69% is within normal limits.

A patient who has been involved in a motor vehicle crash arrives in the emergency department (ED) with cool, clammy skin; tachycardia; and hypotension. Which intervention ordered by the health care provider should the nurse implement first? a. Insert two large-bore IV catheters. b. Initiate continuous electrocardiogram (ECG) monitoring. c. Provide oxygen at 100% per non-rebreather mask. d. Draw blood to type and crossmatch for transfusions.

ANS: C The first priority in the initial management of shock is maintenance of the airway and ventilation. ECG monitoring, insertion of IV catheters, and obtaining blood for transfusions should also be rapidly accomplished but only after actions to maximize oxygen delivery have been implemented.

19. Which assessment finding obtained by the nurse when caring for a patient receiving mechanical ventilation indicates the need for suctioning? a. The patient was last suctioned 6 hours ago. b. The patient's oxygen saturation drops to 93%. c. The patient's respiratory rate is 32 breaths/min. d. The patient has occasional audible expiratory wheezes.

ANS: C The increase in respiratory rate indicates that the patient may have decreased airway clearance and requires suctioning. Suctioning is done when patient assessment data indicate that it is needed and not on a scheduled basis. Occasional expiratory wheezes do not indicate poor airway clearance, and suctioning the patient may induce bronchospasm and increase wheezing. An O2 saturation of 93% is acceptable and does not suggest that immediate suctioning is needed.

19. Which assessment finding obtained by the nurse when caring for a patient receiving mechanical ventilation indicates the need for suctioning? a. The patient's oxygen saturation is 93%. b. The patient was last suctioned 6 hours ago. c. The patient's respiratory rate is 32 breaths/minute. d. The patient has occasional audible expiratory wheezes.

ANS: C The increase in respiratory rate indicates that the patient may have decreased airway clearance and requires suctioning. Suctioning is done when patient assessment data indicate that it is needed, not on a scheduled basis. Occasional expiratory wheezes do not indicate poor airway clearance, and suctioning the patient may induce bronchospasm and increase wheezing. An oxygen saturation of 93% is acceptable and does not suggest that immediate suctioning is needed. DIF: Cognitive Level: Apply (application) REF: 1616 TOP: Nursing Process: Assessment MSC: NCLEX: Physiological Integrity

8. Which action should the nurse take when the low pressure alarm sounds for a patient who has an arterial line in the left radial artery? a. Fast flush the arterial line. b. Check the left hand for pallor. c. Assess for cardiac dysrhythmias. d. Re-zero the monitoring equipment.

ANS: C The low pressure alarm indicates a drop in the patient's blood pressure, which may be caused by cardiac dysrhythmias. There is no indication to re-zero the equipment. Pallor of the left hand would be caused by occlusion of the radial artery by the arterial catheter, not by low pressure. There is no indication of a need for flushing the line.

8. Which action is a priority for the nurse to take when the low pressure alarm sounds for a patient who has an arterial line in the left radial artery? a. Fast flush the arterial line. b. Check the left hand for pallor. c. Assess for cardiac dysrhythmias. d. Rezero the monitoring equipment.

ANS: C The low pressure alarm indicates a drop in the patient's blood pressure, which may be caused by cardiac dysrhythmias. There is no indication to rezero the equipment. Pallor of the left hand would be caused by occlusion of the radial artery by the arterial catheter, not by low pressure. There is no indication of a need for flushing the line. DIF: Cognitive Level: Apply (application) REF: 1606 OBJ: Special Questions: Prioritization TOP: Nursing Process: Implementation MSC: NCLEX: Physiological Integrity

31. The nurse notes that a patient's endotracheal tube (ET), which was at the 22-cm mark, is now at the 25-cm mark and the patient is anxious and restless. Which action should the nurse take next? a. Offer reassurance to the patient. b. Bag the patient at an FIO2 of 100%. c. Listen to the patient's breath sounds. d. Notify the patient's health care provider.

ANS: C The nurse should first determine whether the ET tube has been displaced into the right mainstem bronchus by listening for unilateral breath sounds. If so, assistance will be needed to reposition the tube immediately. The other actions are also appropriate, but detection and correction of tube malposition are the most critical actions. DIF: Cognitive Level: Apply (application) REF: 1614 OBJ: Special Questions: Prioritization TOP: Nursing Process: Implementation MSC: NCLEX: Physiological Integrity

24. The nurse is caring for a patient receiving a continuous norepinephrine (Levophed) IV infusion. Which patient assessment finding indicates that the infusion rate may need to be adjusted? a. Heart rate is 58 beats/minute. b. Mean arterial pressure (MAP) is 56 mm Hg. c. Systemic vascular resistance (SVR) is elevated. d. Pulmonary artery wedge pressure (PAWP) is low.

ANS: C Vasoconstrictors such as norepinephrine (Levophed) will increase SVR, and this will increase the work of the heart and decrease peripheral perfusion. The infusion rate may need to be decreased. Bradycardia, hypotension (MAP of 56 mm Hg), and low PAWP are not associated with norepinephrine infusion. DIF: Cognitive Level: Apply (application) REF: 1604 TOP: Nursing Process: Evaluation MSC: NCLEX: Physiological Integrity

A client is admitted to the hospital with reports of chest pain. The nurse is monitoring the client and notifies the physician when the client exhibits

Adventitious breath sounds pg. 300

24. The nurse is caring for a patient receiving a continuous norepinephrine IV infusion. Which patient assessment finding indicates that the infusion rate may need to be adjusted? a. Heart rate is slow at 58 beats/min. b. Mean arterial pressure (MAP) is 56 mm Hg. c. Systemic vascular resistance (SVR) is elevated. d. Pulmonary artery wedge pressure (PAWP) is low.

ANS: C Vasoconstrictors such as norepinephrine will increase SVR, and this will increase the work of the heart and decrease peripheral perfusion. The infusion rate may need to be decreased. Bradycardia, hypotension (MAP of 56 mm Hg), and low PAWP are not associated with norepinephrine infusion.

12. A nurse is assessing a patient who is receiving a nitroprusside infusion to treat cardiogenic shock. Which finding indicates that the drug is effective? a. No new heart murmurs c. Warm, pink, and dry skin b. Decreased troponin level d. Blood pressure of 92/40 mm Hg

ANS: C Warm, pink, and dry skin indicates that perfusion to tissues is improved. Because nitroprusside is a vasodilator, the blood pressure may be low even if the drug is effective. Absence of a heart murmur and a decrease in troponin level are not indicators of improvement in shock.

A nurse is assessing a patient who is receiving a nitroprusside (Nipride) infusion to treat cardiogenic shock. Which finding indicates that the medication is effective? a. No new heart murmurs b. Decreased troponin level c. Warm, pink, and dry skin d. Blood pressure 92/40 mm Hg

ANS: C Warm, pink, and dry skin indicates that perfusion to tissues is improved. Since nitroprusside is a vasodilator, the blood pressure may be low even if the medication is effective. Absence of a heart murmur and a decrease in troponin level are not indicators of improvement in shock.

5. After receiving 1000 mL of normal saline, the central venous pressure for a patient who has septic shock is 10 mm Hg, but the blood pressure is still 82/40 mm Hg. The nurse will anticipate the administration of a. nitroglycerine (Tridil). b. drotrecogin alpha (Xigris). c. norepinephrine (Levophed). d. sodium nitroprusside (Nipride).

ANS: C When fluid resuscitation is unsuccessful, vasopressor drugs are administered to increase the systemic vascular resistance (SVR) and improve tissue perfusion. Nitroglycerin would decrease the preload and further drop cardiac output and BP. Drotrecogin alpha may decrease inappropriate inflammation and help prevent systemic inflammatory response syndrome, but it will not directly improve blood pressure. Nitroprusside is an arterial vasodilator and would further decrease SVR. DIF: Cognitive Level: Application REF: 1731 | 1733-1735 TOP: Nursing Process: Planning MSC: NCLEX: Physiological Integrity

After receiving 1000 mL of normal saline, the central venous pressure for a patient who has septic shock is 10 mm Hg, but the blood pressure is still 82/40 mm Hg. The nurse will anticipate the administration of a. nitroglycerine (Tridil). b. drotrecogin alpha (Xigris). c. norepinephrine (Levophed). d. sodium nitroprusside (Nipride).

ANS: C When fluid resuscitation is unsuccessful, vasopressor drugs are administered to increase the systemic vascular resistance (SVR) and improve tissue perfusion. Nitroglycerin would decrease the preload and further drop cardiac output and BP. Drotrecogin alpha may decrease inappropriate inflammation and help prevent systemic inflammatory response syndrome, but it will not directly improve blood pressure. Nitroprusside is an arterial vasodilator and would further decrease SVR. DIF: Cognitive Level: Application REF: 1731 | 1733-1735 TOP: Nursing Process: Planning MSC: NCLEX: Physiological Integrity

29. The nurse responds to a ventilator alarm and finds the patient lying in bed gasping and holding the endotracheal tube (ET) in her hand. Which action should the nurse take next? a. Activate the rapid response team. b. Provide reassurance to the patient. c. Call the health care provider to reinsert the tube. d. Manually ventilate the patient with 100% oxygen.

ANS: D The nurse should ensure maximal patient oxygenation by manually ventilating with a bag-valve-mask system. Offering reassurance to the patient, notifying the health care provider about the need to reinsert the tube, and activating the rapid response team are also appropriate after the nurse has stabilized the patient's oxygenation.

18. The nurse notes premature ventricular contractions (PVCs) while suctioning a patient's endotracheal tube. Which next action by the nurse is indicated? a. Plan to suction the patient more frequently. b. Decrease the suction pressure to 80 mm Hg. c. Give antidysrhythmic medications per protocol. d. Stop and ventilate the patient with 100% oxygen.

ANS: D Dysrhythmias during suctioning may indicate hypoxemia or sympathetic nervous system stimulation. The nurse should stop suctioning and ventilate the patient with 100% O2 . There is no indication that more frequent suctioning is needed. Lowering the suction pressure will decrease the effectiveness of suctioning without improving the hypoxemia. Because the PVCs occurred during suctioning, there is no need for antidysrhythmic medications (which may have adverse effects) unless they recur when the suctioning is stopped and patient is well oxygenated.

16. To verify the correct placement of an oral endotracheal tube (ET) after insertion, the best initial action by the nurse is to a. auscultate for the presence of bilateral breath sounds. b. obtain a portable chest x-ray to check tube placement. c. observe the chest for symmetric chest movement with ventilation. d. use an end-tidal CO2 monitor to check for placement in the trachea.

ANS: D End-tidal CO2 monitors are currently recommended for rapid verification of ET placement. Auscultation for bilateral breath sounds and checking chest expansion are also used, but they are not as accurate as end-tidal CO2 monitoring. A chest x-ray confirms the placement but is done after the tube is secured. DIF: Cognitive Level: Apply (application) REF: 1614-1615 TOP: Nursing Process: Evaluation MSC: NCLEX: Physiological Integrity

35. The nurse is caring for a patient with a subarachnoid hemorrhage who is intubated and placed on a mechanical ventilator with 10 cm H2O of peak end-expiratory pressure (PEEP). When monitoring the patient, the nurse will need to notify the health care provider immediately if the patient develops a. oxygen saturation of 93%. b. respirations of 20 breaths/minute. c. green nasogastric tube drainage. d. increased jugular venous distention.

ANS: D Increases in jugular venous distention in a patient with a subarachnoid hemorrhage may indicate an increase in intracranial pressure (ICP) and that the PEEP setting is too high for this patient. A respiratory rate of 20, O2 saturation of 93%, and green nasogastric tube drainage are within normal limits. DIF: Cognitive Level: Apply (application) REF: 1623-1624 TOP: Nursing Process: Assessment MSC: NCLEX: Physiological Integrity

A patient with cardiogenic shock is cool and clammy and hemodynamic monitoring indicates a high systemic vascular resistance (SVR). Which action will the nurse anticipate taking? a. Increase the rate for the prescribed dopamine (Intropin) infusion. b. Decrease the rate for the prescribed nitroglycerin (Tridil) infusion. c. Decrease the rate for the prescribed 5% dextrose in water (D5W) infusion. d. Increase the rate for the prescribed sodium nitroprusside (Nipride) infusion.

ANS: D Nitroprusside is an arterial vasodilator and will decrease the SVR and afterload, which will improve cardiac output. Changes in the D5W and nitroglycerin infusions will not directly increase SVR. Increasing the dopamine will tend to increase SVR. DIF: Cognitive Level: Application REF: 1733-1734 TOP: Nursing Process: Planning MSC: NCLEX: Physiological Integrity

7. When monitoring the effectiveness of treatment for a patient with a large anterior wall myocardial infarction, the most pertinent measurement for the nurse to obtain is a. central venous pressure (CVP). b. systemic vascular resistance (SVR). c. pulmonary vascular resistance (PVR). d. pulmonary artery wedge pressure (PAWP).

ANS: D PAWP reflects left ventricular end diastolic pressure (or left ventricular preload) and is a sensitive indicator of cardiac function. Because the patient is high risk for left ventricular failure, the PAWP must be monitored. An increase will indicate left ventricular failure. The other values would also provide useful information, but the most definitive measurement of changes in cardiac function is the PAWP.

7. When monitoring for the effectiveness of treatment for a patient with a large anterior wall myocardial infarction, the most important information for the nurse to obtain is a. central venous pressure (CVP). b. systemic vascular resistance (SVR). c. pulmonary vascular resistance (PVR). d. pulmonary artery wedge pressure (PAWP).

ANS: D PAWP reflects left ventricular end diastolic pressure (or left ventricular preload) and is a sensitive indicator of cardiac function. Because the patient is high risk for left ventricular failure, the PAWP must be monitored. An increase will indicate left ventricular failure. The other values would also provide useful information, but the most definitive measurement of changes in cardiac function is the PAWP. DIF: Cognitive Level: Apply (application) REF: 1607 TOP: Nursing Process: Evaluation MSC: NCLEX: Physiological Integrity

To evaluate the effectiveness of the pantoprazole (Protonix) ordered for a patient with systemic inflammatory response syndrome (SIRS), which assessment will the nurse perform? a. Auscultate bowel sounds. b. Palpate for abdominal pain. c. Ask the patient about nausea. d. Check stools for occult blood.

ANS: D Proton pump inhibitors are given to decrease the risk for stress ulcers in critically ill patients. The other assessments also will be done, but these will not help in determining the effectiveness of the pantoprazole administration.

19. During change-of-shift report, the nurse learns that a patient has been admitted with dehydration and hypotension after having vomiting and diarrhea for 3 days. Which finding is most important for the nurse to report to the health care provider? a. Decreased bowel sounds b. Apical pulse 110 beats/min c. Pale, cool, and dry extremities d. New onset of confusion and agitation

ANS: D The changes in mental status are indicative that the patient is in the progressive stage of shock and that rapid intervention is needed to prevent further deterioration. The other information is consistent with compensatory shock. DIF: Cognitive Level: Application REF: 1728-1729 OBJ: Special Questions: Prioritization TOP: Nursing Process: Assessment MSC: NCLEX: Physiological Integrity

37. After change-of-shift report on a ventilator weaning unit, which patient should the nurse assess first? a. Patient who failed a spontaneous breathing trial and has been placed in a rest mode on the ventilator b. Patient who is intubated and has continuous partial pressure end-tidal CO2 (PETCO2) monitoring c. Patient with a central venous oxygen saturation (ScvO2) of 69% while on bilevel positive airway pressure (BiPAP) d. Patient who was successfully weaned and extubated 4 hours ago and now has no urine output for the last 6 hours

ANS: D The decreased urine output may indicate acute kidney injury or that the patient's cardiac output and perfusion of vital organs have decreased. Any of these causes would require rapid action. The data about the other patients indicate that their conditions are stable and do not require immediate assessment or changes in their care. Continuous PETCO2 monitoring is frequently used when patients are intubated. The rest mode should be used to allow patient recovery after a failed SBT, and an ScvO2 of 69% is within normal limits. DIF: Cognitive Level: Analyze (analysis) REF: 1625 | 1627 OBJ: Special Questions: Prioritization; Multiple Patients TOP: Nursing Process: Planning MSC: NCLEX: Safe and Effective Care Environment

37. After change-of-shift report, which patient should the progressive care nurse assess first? a. Patient who was extubated this morning and has a temperature of 101.4°F (38.6°C) b. Patient with bilevel positive airway pressure (BiPAP) for obstructive sleep apnea and a respiratory rate of 16 c. Patient with arterial pressure monitoring who is 2 hours post-percutaneous coronary intervention and needs to void d. Patient who is receiving IV heparin for a venous thromboembolism and has a partial thromboplastin time (PTT) of 101 sec

ANS: D The findings for this patient indicate high risk for bleeding from an elevated (nontherapeutic) PTT. The nurse needs to adjust the rate of the infusion (dose) per the health care provider's parameters. The patient with BiPAP for sleep apnea has a normal respiratory rate. The patient recovering from the percutaneous coronary intervention will need to be assisted with voiding and this task could be delegated to unlicensed assistive personnel. The patient with a fever may be developing ventilator-associated pneumonia, but addressing the bleeding risk is a higher priority.

38. After change-of-shift report, which patient should the progressive care nurse assess first? a. Patient who was extubated in the morning and has a temperature of 101.4° F (38.6° C) b. Patient with bilevel positive airway pressure (BiPAP) for sleep apnea whose respiratory rate is 16 c. Patient with arterial pressure monitoring who is 2 hours post-percutaneous coronary intervention who needs to void d. Patient who is receiving IV heparin for a venous thromboembolism and has a partial thromboplastin time (PTT) of 98 sec

ANS: D The findings for this patient indicate high risk for bleeding from an elevated (nontherapeutic) PTT. The nurse needs to adjust the rate of the infusion (dose) per the health care provider's parameters. The patient with BiPAP for sleep apnea has a normal respiratory rate. The patient recovering from the percutaneous coronary intervention will need to be assisted with voiding and this task could be delegated to unlicensed assistive personnel. The patient with a fever may be developing ventilator-associated pneumonia, but addressing the bleeding risk is a higher priority. DIF: Cognitive Level: Analyze (analysis) REF: 1600 OBJ: Special Questions: Prioritization; Multiple Patients TOP: Nursing Process: Planning MSC: NCLEX: Safe and Effective Care Environment

20. A patient who has been involved in a motor vehicle crash is admitted to the emergency department (ED) with cool, clammy skin; tachycardia; and hypotension. Which of these prescribed interventions should the nurse implement first? a. Place the patient on continuous cardiac monitor. b. Draw blood to type and crossmatch for transfusions. c. Insert two 14-gauge IV catheters in antecubital space. d. Administer oxygen at 100% per non-rebreather mask

ANS: D The first priority in the initial management of shock is maintenance of the airway and ventilation. Cardiac monitoring, insertion of IV catheters, and obtaining blood for transfusions also should be rapidly accomplished, but only after actions to maximize oxygen delivery have been implemented. DIF: Cognitive Level: Application REF: 1732 OBJ: Special Questions: Prioritization TOP: Nursing Process: Implementation

A patient who has been involved in a motor vehicle crash is admitted to the emergency department (ED) with cool, clammy skin; tachycardia; and hypotension. Which of these prescribed interventions should the nurse implement first? a. Place the patient on continuous cardiac monitor. b. Draw blood to type and crossmatch for transfusions. c. Insert two 14-gauge IV catheters in antecubital space. d. Administer oxygen at 100% per non-rebreather mask

ANS: D The first priority in the initial management of shock is maintenance of the airway and ventilation. Cardiac monitoring, insertion of IV catheters, and obtaining blood for transfusions also should be rapidly accomplished, but only after actions to maximize oxygen delivery have been implemented. DIF: Cognitive Level: Application REF: 1732 OBJ: Special Questions: Prioritization TOP: Nursing Process: Implementation

32. The nurse educator is evaluating the care that a new registered nurse (RN) provides to a patient receiving mechanical ventilation. Which action by the new RN indicates the need for more education? a. The RN increases the FIO2 to 100% before suctioning. b. The RN secures a bite block in place using adhesive tape. c. The RN asks for assistance to reposition the endotracheal tube. d. The RN positions the patient with the head of bed at 10 degrees.

ANS: D The head of the patient's bed should be positioned at 30 to 45 degrees to prevent ventilator-associated pneumonia. The other actions by the new RN are appropriate. DIF: Cognitive Level: Apply (application) REF: 1623 OBJ: Special Questions: Delegation TOP: Nursing Process: Evaluation MSC: NCLEX: Safe and Effective Care Environment

15. While waiting for cardiac transplantation, a patient with severe cardiomyopathy has a ventricular assist device (VAD) implanted. When planning care for this patient, the nurse should anticipate a. giving immunosuppressive medications. b. preparing the patient for a permanent VAD. c. teaching the patient the reason for complete bed rest. d. monitoring the surgical incision for signs of infection.

ANS: D The insertion site for the VAD provides a source for transmission of infection to the circulatory system and requires frequent monitoring. Patient's with VADs are able to have some mobility and may not be on bed rest. The VAD is a bridge to transplantation, not a permanent device. Immunosuppression is not necessary for nonbiologic devices like the VAD. DIF: Cognitive Level: Apply (application) REF: 1613 TOP: Nursing Process: Planning MSC: NCLEX: Physiological Integrity

17. To maintain proper cuff pressure of an endotracheal tube (ET) when the patient is on mechanical ventilation, the nurse should a. inflate the cuff with a minimum of 10 mL of air. b. inflate the cuff until the pilot balloon is firm on palpation. c. inject air into the cuff until a manometer shows 15 mm Hg pressure. d. inject air into the cuff until a slight leak is heard only at peak inflation.

ANS: D The minimal occluding volume technique involves injecting air into the cuff until an air leak is present only at peak inflation. The volume to inflate the cuff varies with the ET and the patient's size. Cuff pressure should be maintained at 20 to 25 mm Hg. An accurate assessment of cuff pressure cannot be obtained by palpating the pilot balloon.

17. To maintain proper cuff pressure of an endotracheal tube (ET) when the patient is on mechanical ventilation, the nurse should a. inflate the cuff with a minimum of 10 mL of air. b. inflate the cuff until the pilot balloon is firm on palpation. c. inject air into the cuff until a manometer shows 15 mm Hg pressure. d. inject air into the cuff until a slight leak is heard only at peak inflation.

ANS: D The minimal occluding volume technique involves injecting air into the cuff until an air leak is present only at peak inflation. The volume to inflate the cuff varies with the ET and the patient's size. Cuff pressure should be maintained at 20 to 25 mm Hg. An accurate assessment of cuff pressure cannot be obtained by palpating the pilot balloon. DIF: Cognitive Level: Understand (comprehension) REF: 1615 TOP: Nursing Process: Implementation MSC: NCLEX: Physiological Integrity

30. The nurse responds to a ventilator alarm and finds the patient lying in bed holding the endotracheal tube (ET). Which action should the nurse take next? a. Activate the rapid response team. b. Provide reassurance to the patient. c. Call the health care provider to reinsert the tube. d. Manually ventilate the patient with 100% oxygen.

ANS: D The nurse should ensure maximal patient oxygenation by manually ventilating with a bag-valve-mask system. Offering reassurance to the patient, notifying the health care provider about the need to reinsert the tube, and activating the rapid response team are also appropriate after the nurse has stabilized the patient's oxygenation. DIF: Cognitive Level: Apply (application) REF: 1617 OBJ: Special Questions: Prioritization TOP: Nursing Process: Implementation MSC: NCLEX: Physiological Integrity

10. While assisting with the placement of a pulmonary artery (PA) catheter, the nurse notes that the catheter is correctly placed when the balloon is inflated and the monitor shows a a. typical PA pressure waveform. b. tracing of the systemic arterial pressure. c. tracing of the systemic vascular resistance. d. typical PA wedge pressure (PAWP) tracing.

ANS: D The purpose of a PA line is to measure PAWP, so the catheter is floated through the pulmonary artery until the dilated balloon wedges in a distal branch of the pulmonary artery, and the PAWP readings are available. After insertion, the balloon is deflated and the PA waveform will be observed. Systemic arterial pressures are obtained using an arterial line, and the systemic vascular resistance is a calculated value, not a waveform.

25. When caring for the patient with a pulmonary artery (PA) pressure catheter, the nurse observes that the PA waveform indicates that the catheter is in the wedged position. Which action should the nurse take next? a. Zero balance the transducer. b. Activate the fast flush system. c. Notify the health care provider. d. Deflate and reinflate the PA balloon.

ANS: D When the catheter is in the wedge position, blood flow past the catheter is obstructed, placing the patient at risk for pulmonary infarction. A health care provider or advanced practice nurse should be called to reposition the catheter. The other actions will not correct the wedging of the PA catheter. DIF: Cognitive Level: Apply (application) REF: 1608 TOP: Nursing Process: Implementation MSC: NCLEX: Physiological Integrity

Which type of shock causes an absence of bowel sounds? 1 Cardiogenic shock 2 Neurogenic shock 3 Hypovolemic shock 4 Anaphylactic shock

Absence of bowel sounds is associated with hypovolemic shock. Decreased bowel sounds are seen with cardiogenic shock. Bowel dysfunction is associated with neurogenic shock. Abdominal pain, nausea and vomiting are seen with anaphylactic shock. Test-Taking Tip: Identifying content and what is being asked about that content is critical to your choosing the correct response. Be alert for words in the stem of the item that are the same or similar in nature to those in one or two of the options. Text Reference - p. 1635

A client presents to the community health office experiencing rapidly increasing symptoms of anaphylactic shock. Which nursing action would be completed first?

Administer an epinephrine injection.

You are caring for a client in shock who is deteriorating. You are infusing IV fluids and giving medications as ordered. What type of medications are you most likely giving to this client?

Adrenergic drugs

The nurse is caring for a client in shock who is deteriorating. The nurse is infusing IV fluids and giving medications as ordered. What type of medications is the nurse most likely giving to this client?

Adrenergic drugs pg. 294

Which colloid is expensive but rapidly expands plasma volume?

Albumin

2 To ensure adequate tracheal perfusion, the nurse should maintain cuff pressure at 20 to 25 cm H2O. Excess cuff pressure can damage the tracheal mucosa. Lesser cuff pressure may cause the ET tube to become destabilized and extubate. Text Reference - p. 1615

An endotracheal (ET) tube is inserted in a patient. The nurse inflates the cuff to stabilize the tube. How much cuff pressure should be maintained to keep it inflated and ensure adequate tracheal perfusion? 1 10-15 cm H2O 2 20-25 cm H2O 3 30-35 cm H2O 4 40-45 cm H2O

The nurse is providing care for an older adult patient who is experiencing low partial pressure of oxygen in arterial blood (PaO2) as a result of worsening left-sided pneumonia. Which intervention should the nurse use to help the patient mobilize his secretions? A. Augmented coughing or huff coughing B. Positioning the patient side-lying on his left side C. Frequent and aggressive nasopharyngeal suctioning D. Application of noninvasive positive pressure ventilation (NIPPV)

Ans. A Augmented coughing and huff coughing techniques may aid the patient in the mobilization of secretions. If positioned side-lying, the patient should be positioned on his right side (good lung down) for improved perfusion and ventilation. Suctioning may be indicated but should always be performed cautiously because of the risk of hypoxia. NIPPV is inappropriate in the treatment of patients with excessive secretions.

The nurse in the cardiac care unit is caring for a patient who has developed acute respiratory failure. Which medication does the nurse know is being used to decrease this patient's pulmonary congestion and agitation? A. Morphine sulfate B. Albuterol (Ventolin) C. Azithromycin (Zithromax) D. Methylprednisolone (Solu-Medrol)

Ans. A For a patient with acute respiratory failure related to the heart, morphine is used to decrease pulmonary congestion as well as anxiety, agitation, and pain. Albuterol is used to reduce bronchospasm. Azithromycin is used for pulmonary infections. Methylprednisolone is used to reduce airway inflammation and edema

A patient is in acute respiratory distress syndrome (ARDS) as a result of sepsis. Which measure(s) would most likely be implemented to maintain cardiac output? A. Administer crystalloid fluids or colloid solutions. B. Position the patient in the Trendelenburg position. C. Place the patient on fluid restriction and administer diuretics. D. Perform chest physiotherapy and assist with staged coughing

Ans. A Low cardiac output may necessitate crystalloid fluids or colloid solutions in addition to lowering positive end-expiratory pressure (PEEP) or administering inotropes. The Trendelenburg position (not recommended to treat hypotension) and chest physiotherapy are unlikely to relieve decreased cardiac output, and fluid restriction and diuresis would be inappropriate interventions

The patient has pulmonary fibrosis and experiences hypoxemia during exercise but not at rest. To plan for the patient's care, the nurse should know that this patient is experiencing which physiologic mechanism of respiratory failure? A. Diffusion limitation B. Intrapulmonary shunt C. Alveolar hypoventilation D. Ventilation-perfusion mismatch

Ans. A The patient with pulmonary fibrosis has a thickened alveolar-capillary interface that slows gas transport, and hypoxemia is more likely during exercise than at rest. Intrapulmonary shunt occurs when alveoli fill with fluid (e.g., ARDS, pneumonia). Alveolar hypoventilation occurs when there is a generalized decrease in ventilation (e.g., restrictive lung disease, CNS diseases, neuromuscular diseases). Ventilation-perfusion mismatch occurs when the amount of air does not match the amount of blood that the lung receives (e.g., COPD, pulmonary embolus).

A 70-year-old patient in the ICU has become agitated and inattentive since his heart surgery. The nurse knows that this ICU psychosis frequently occurs in individuals with pre-existing dementia, history of alcohol abuse, and severe disease. What interventions should the nurse provide this patient to improve the patient's cognition (select all that apply)? A. Improve oxygenation. B. Provide a small amount of beer. C. Have the family stay with the patient. D. Enable the patient to sleep on a schedule with dim lights. E. Decrease sensory overload by conversing away from patient's room.

Ans. A, D, E ICU psychosis is from delirium in most ICU patients. Improving oxygenation, enabling the patient to sleep, and decreasing sensory overload along with orientation is all helpful in improving the patient's cognition. The beer may or may not be allowed for this patient, and the nurse should not assume that it will help. Having a family member stay with the patient to reorient the patient is helpful, but the family group may increase sensory overload with conversations not involving the patient.

When caring for the patient with ARDS, the critical care nurse knows that therapy is appropriate for the patient when which goal is being met? A. pH is 7.32. B. PaO2 is greater than or equal to 60 mm Hg. C. PEEP increased to 20 cm H2O caused BP to fall to 80/40. D. No change in PaO2 when patient is turned from supine to prone position

Ans. B The overall goal in caring for the patient with ARDS is for the PaO2 to be greater than or equal to 60mm Hg with adequate lung ventilation to maintain a normal pH of 7.35 to 7.45. PEEP is usually increased for ARDS patients, but a dramatic reduction in BP indicates a complication of decreased cardiac output. A positive occurrence is a marked improvement in PaO2 from perfusion better matching ventilation when the anterior air-filled, nonatelectatic alveoli become dependent in the prone position

Which factor indicates that tracheotomy would be preferable to endotracheal intubation? A. The patient is unable to clear secretions. B. The patient is at high risk for aspiration. C. A long-term airway is probably necessary. D. An upper airway obstruction is impairing the patient's ventilation.

Ans. C A tracheotomy is indicated when the need for an artificial airway is expected to be long term. Aspiration risk, an inability to clear secretions, and upper airway obstruction are indications for an artificial airway, but these are not specific indications for tracheotomy.

The nurse is aware of the value of using a mini-tracheostomy to facilitate suctioning when patients are unable to independently mobilize their secretions. For which patient is the use of a mini-trach indicated? A. A patient whose recent ischemic stroke has resulted in the loss of his gag reflex B. A patient who requires long-term mechanical ventilation as the result of a spinal cord injury C. A patient whose increased secretions are the result of community-acquired pneumonia D. A patient with a head injury who has developed aspiration pneumonia after his family insisted on spoon-feeding him

Ans. C It is appropriate to suction a patient with pneumonia using a mini-trach if blind suctioning is ineffective or difficult. An absent or compromised gag reflex, long-term mechanical ventilation, and a history of aspiration contraindicates the use of a mini-trach

When planning care for a patient on a mechanical ventilator, the nurse understands that the application of positive end-expiratory pressure (PEEP) to the ventilator settings has which therapeutic effect? A. Increased inflation of the lungs B. Prevention of barotrauma to the lung tissue C. Prevention of alveolar collapse during expiration D. Increased fraction of inspired oxygen concentration (FIO2) administration

Ans. C PEEP is positive pressure that is applied to the airway during exhalation. This positive pressure prevents the alveoli from collapsing, improving oxygenation and enabling a reduced FIO2 requirement. PEEP does not cause increased inflation of the lungs or prevent barotrauma. Actually auto-PEEP resulting from inadequate exhalation time may contribute to barotrauma.

The patient has developed cardiogenic shock after a left anterior descending myocardial infection. Which circulatory-assist device should the nurse expect to use for this patient? A. Cardiopulmonary bypass B. Impedance cardiography (ICG) C. Intraaortic balloon pump (IABP) D. Central venous pressure (CVP) measurement

Ans. C The most commonly used mechanical circulatory-assist device is the intraaortic balloon pump (IABP), and it is used to decrease ventricular workload, increase myocardial perfusion, and augment circulation. Cardiopulmonary bypass provides circulation during open heart surgery. It is not used as an assist device after surgery. ICG is a noninvasive method to obtain cardiac output and assess thoracic fluid status. CVP measurement is an invasive measurement of right ventricular preload and reflects fluid volume problems

The post-anesthesia care unit (PACU) has several patients with endotracheal tubes. Which patient should receive the least amount of endotracheal suctioning? A. Transplantation of a kidney B. Replacement of aortic valve C. Cerebral aneurysm resection D. Formation of an ileal conduit

Ans. C The nurse should avoid suctioning the patient after a craniotomy until it is necessary because suctioning will increase this patient's intracranial pressure. The patients with a kidney transplantation, aortic valve replacement, or formation of an ileal conduit will not be negatively affected by suctioning, although it should only be done when needed, not routinely.

The nurse is caring for a patient who is admitted with a barbiturate overdose. The patient is comatose with BP 90/60, apical pulse 110, and respiratory rate 8. Based upon the initial assessment findings, the nurse recognizes that the patient is at risk for which type of respiratory failure? A. Hypoxemic respiratory failure related to shunting of blood B. Hypoxemic respiratory failure related to diffusion limitation C. Hypercapnic respiratory failure related to alveolar hypoventilation D. Hypercapnic respiratory failure related to increased airway resistance

Ans. C The patient's respiratory rate is decreased as a result of barbiturate overdose, which caused respiratory depression. The patient is at risk for hypercapnic respiratory failure resulting from the decreased respiratory rate and thus decreased CO2 elimination. Barbiturate overdose does not lead to shunting of blood, diffusion limitations, or increased airway resistance.

The nurse is admitting a 45-year-old asthmatic patient in acute respiratory distress. The nurse auscultates the patient's lungs and notes cessation of the inspiratory wheezing. The patient has not yet received any medication. What should this finding most likely suggest to the nurse? A. Spontaneous resolution of the acute asthma attack B. An acute development of bilateral pleural effusions C. Airway constriction requiring intensive interventions D. Overworked intercostal muscles resulting in poor air exchange

Ans. C When the patient in respiratory distress has inspiratory wheezing, and then it ceases, it is an indication of airway obstruction. This finding requires emergency action to restore the airway. Cessation of inspiratory wheezing does not indicate spontaneous resolution of the acute asthma attack, bilateral pleural effusion development, or overworked intercostal muscles in this asthmatic patient that is in acute respiratory distress

Which interventions should the nurse perform prior to suctioning a patient who has an endotracheal (ET) tube using open-suction technique (select all that apply)? A. Put on clean gloves. B. Administer a bronchodilator. C. Perform a cardiopulmonary assessment. D. Hyperoxygenate the patient for 30 seconds. E. Insert a few drops of normal saline into the ET to break up secretions

Ans. C, D Suctioning is preceded by a thorough assessment and hyperoxygenation for 30 seconds. Sterile, not clean, gloves are necessary, and it is not necessary to administer a bronchodilator. Instillation of normal saline into the ET tube is not an accepted standard practice

When the nurse is explaining treatment to the families, for which patient would NIPPV be an appropriate intervention to promote oxygenation? A. A patient whose cardiac output and blood pressure are unstable B. A patient whose respiratory failure is due to a head injury with loss of consciousness C. A patient with a diagnosis of cystic fibrosis and who is currently producing copious secretions D. A patient who is experiencing respiratory failure as a result of the progression of myasthenia gravis

Ans. D NIPPV is most effective in treating patients with respiratory failure resulting from chest wall and neuromuscular disease. It is not recommended in patients who are experiencing hemodynamic instability, decreased level of consciousness, or excessive secretions.

A patient's daughter asks the nurse what SIMV means on the settings of the mechanical ventilator attached to her father. Which statement best describes this mode of ventilation? A. "SIMV provides additional inspiratory pressure so that your father does not have to work as hard to breathe, thus enabling better oxygenation and a quicker recovery with fewer complications." B. "SIMV is a mode that allows the ventilator to totally control your father's breathing. It will prevent him from hyperventilating or hypoventilating, thus ensuring the best oxygenation." C. "SIMV is a mode that allows your father to breathe on his own, but the ventilator will control how deep a breath he will receive. The ventilator can sense when he wants a breath, and it will deliver it." D. "SIMV is a mode that allows your father to breathe on his own while receiving a preset number of breaths from the ventilator. He can breathe as much or as little as he wants beyond what the ventilator will breathe for him."

Ans. D SIMV stands for synchronized intermittent mandatory ventilation, a mode of ventilation in which the ventilator delivers a preset tidal volume at a preset frequency in synchrony with the patient's spontaneous breathing. Between ventilator-delivered breaths the patient is able to breathe spontaneously, receiving the preset FIO2 but self-regulates the rate and depth of those breaths. Pressure support ventilation (PSV) applies positive pressure only during inspiration. PSV is not used as a sole ventilator support during acute respiratory failure because of the risk of hypoventilation, but it does decrease the work of breathing. Pressure-control inverse ratio ventilation (PC-IRV) sets the ventilation pressure and the ratio of inspiration to expiration to control the patient's breathing. Assist-control ventilation (ACV) or assisted mandatory ventilation (AMV) delivers a preset rate of breaths but allows the patient to breathe spontaneously, with a preset tidal volume

The nurse is caring for a patient who has been on a mechanical ventilator for several days. Which weaning parameter would tell the nurse whether or not the patient has enough muscle strength to breathe without assistance? A. Tidal volume B. Minute ventilation C. Forced vital capacity D. Negative inspiratory force

Ans. D The negative inspiratory force (NIF) measures inspiratory muscle strength. Tidal volume and minute ventilation assess the patient's respiratory endurance. Forced vital capacity is not used as a measure to determine weaning from a ventilator

A patient is scheduled for a myelogram and the nurse explains to the patient that this is an invasive procedure, which assesses for any lesions in the spinal cord. The nurse should explain that the preparation is similar to which of the following neurologic tests? A) Lumbar puncture B) MRI C) Cerebral angiography D) EEG

Ans: A Feedback: A myelogram is an x-ray of the spinal subarachnoid space taken after the injection of a contrast agent into the spinal subarachnoid space through a lumbar puncture. Patient preparation for a myelogram would be similar to that for lumbar puncture. The other listed diagnostic tests do not involve lumbar puncture.

A patient with lower back pain is scheduled for myelography using metrizamide (a water-soluble contrast dye). After the test, the nurse should prioritize what action? A) Positioning the patient with the head of the bed elevated 45 degrees B) Administering IV morphine sulfate to prevent headache C) Limiting fluids for the next 12 hours D) Helping the patient perform deep breathing and coughing exercises

Ans: A Feedback: After myelography, the patient lies in bed with the head of the bed elevated 30 to 45 degrees. The patient is advised to remain in bed in the recommended position for 3 hours or as prescribed. Drinking liberal amounts of fluid for rehydration and replacement of CSF may decrease the incidence of post-lumbar puncture headache. Deep breathing and coughing exercises are not normally necessary since there is no consequent risk of atelectasis.

A patient is being given a medication that stimulates her parasympathetic system. Following administration of this medication, the nurse should anticipate what effect? A) Constricted pupils B) Dilated bronchioles C) Decreased peristaltic movement D) Relaxed muscular walls of the urinary bladder

Ans: A Feedback: Parasympathetic stimulation results in constricted pupils, constricted bronchioles, increased peristaltic movement, and contracted muscular walls of the urinary bladder.

A patient exhibiting an uncoordinated gait has presented at the clinic. Which of the following is the most plausible cause of this patient's health problem? A) Cerebellar dysfunction B) A lesion in the pons C) Dysfunction of the medulla D) A hemorrhage in the midbrain

Ans: A Feedback: The cerebellum controls fine movement, balance, position sense, and integration of sensory input. Portions of the pons control the heart, respiration, and blood pressure. Cranial nerves IX through XII connect to the brain in the medulla. Cranial nerves III and IV originate in the midbrain.

A patient is admitted to the medical unit with an exacerbation of multiple sclerosis. When assessing this patient, the nurse has the patient stick out her tongue and move it back and forth. What is the nurse assessing? A) Function of the hypoglossal nerve B) Function of the vagus nerve C) Function of the spinal nerve D) Function of the trochlear nerve

Ans: A Feedback: The hypoglossal nerve is the 12th cranial nerve. It is responsible for movement of the tongue. None of the other listed nerves affects motor function in the tongue.

The patient in the ED has just had a diagnostic lumbar puncture. To reduce the incidence of a post-lumbar puncture headache, what is the nurse's most appropriate action? A) Position the patient prone. B) Position the patient supine with the head of bed flat. C) Position the patient left side-lying. D) Administer acetaminophen as ordered.

Ans: A The lumbar puncture headache may be avoided if a small-gauge needle is used and if the patient remains prone after the procedure. Acetaminophen is not administered as a preventative measure for post-lumbar puncture headaches.

The nurse is doing an initial assessment on a patient newly admitted to the unit with a diagnosis of cerebrovascular accident (CVA). The patient has difficulty copying a figure that the nurse has drawn and is diagnosed with visual-receptive aphasia. What brain region is primarily involved in this deficit? A) Temporal lobe B) Parietal-occipital area C) Inferior posterior frontal areas D) Posterior frontal area

Ans: B Difficulty copying a figure that the nurse has drawn would be considered visual-receptive aphasia, which involves the parietal-occipital area. Expressive aphasia, the inability to express oneself, is often associated with damage to the frontal area. Receptive aphasia, the inability to understand what someone else is saying, is often associated with damage to the temporal lobe area.

The nurse is caring for a patient who exhibits abnormal results of the Weber test and Rinne test. The nurse should suspect dysfunction involving what cranial nerve? A) Trigeminal B) Acoustic C) Hypoglossal D) Trochlear

Ans: B Feedback: Abnormal hearing can correlate with damage to cranial nerve VIII (acoustic). The acoustic nerve functions in hearing and equilibrium. The trigeminal nerve functions in facial sensation, corneal reflex, and chewing. The hypoglossal nerve moves the tongue. The trochlear nerve controls muscles that move the eye.

The nurse is performing a neurologic assessment of a patient whose injuries have rendered her unable to follow verbal commands. How should the nurse proceed with assessing the patient's level of consciousness (LOC)? A) Assess the patient's vital signs and correlate these with the patient's baselines. B) Assess the patient's eye opening and response to stimuli. C) Document that the patient currently lacks a level of consciousness. D) Facilitate diagnostic testing in an effort to obtain objective data.

Ans: B Feedback: If the patient is not alert or able to follow commands, the examiner observes for eye opening; verbal response and motor response to stimuli, if any; and the type of stimuli needed to obtain a response. Vital signs and diagnostic testing are appropriate, but neither will allow the nurse to gauge the patient's LOC. Inability to follow commands does not necessarily denote an absolute lack of consciousness.

In the course of a focused neurologic assessment, the nurse is palpating the patient's major muscle groups at rest and during passive movement. Data gleaned from this assessment will allow the nurse to describe which of the following aspects of neurologic function? A) Muscle dexterity B) Muscle tone C) Motor symmetry D) Deep tendon reflexes

Ans: B Feedback: Muscle tone (the tension present in a muscle at rest) is evaluated by palpating various muscle groups at rest and during passive movement. Data from this assessment do not allow the nurse to ascertain the patient's dexterity, reflexes, or motor symmetry

The nurse is planning the care of a patient with Parkinson's disease. The nurse should be aware that treatment will focus on what pathophysiological phenomenon? A) Premature degradation of acetylcholine B) Decreased availability of dopamine C) Insufficient synthesis of epinephrine D) Delayed reuptake of serotonin

Ans: B Feedback: Parkinson's disease develops from decreased availability of dopamine, not acetylcholine, epinephrine, or serotonin.

The physician has ordered a somatosensory evoked responses (SERs) test for a patient for whom the nurse is caring. The nurse is justified in suspecting that this patient may have a history of what type of neurologic disorder? A) Hypothalamic disorder B) Demyelinating disease C) Brainstem deficit D) Diabetic neuropathy

Ans: B Feedback: SERs are used to detect deficits in the spinal cord or peripheral nerve conduction and to monitor spinal cord function during surgical procedures. The test is also useful in the diagnosis of demyelinating diseases, such as multiple sclerosis and polyneuropathies, where nerve conduction is slowed. The test is not done to diagnose hypothalamic disorders, brainstem deficits, or diabetic neuropathies.

A trauma patient in the ICU has been declared brain dead. What diagnostic test is used in making the determination of brain death? A) Magnetic resonance imaging (MRI) B) Electroencephalography (EEG) C) Electromyelography (EMG) D) Computed tomography (CT

Ans: B Feedback: The EEG can be used in determining brain death. MRI, CT, and EMG are not normally used in determining brain death.

Assessment is crucial to the care of patients with neurologic dysfunction. What does accurate and appropriate assessment require? Select all that apply. A) The ability to select mediations for the neurologic dysfunction B) Understanding of the tests used to diagnose neurologic disorders C) Knowledge of nursing interventions related to assessment and diagnostic testing D) Knowledge of the anatomy of the nervous system E) The ability to interpret the results of diagnostic tests

Ans: B, C, D Feedback: Assessment requires knowledge of the anatomy and physiology of the nervous system and an understanding of the array of tests and procedures used to diagnose neurologic disorders. Knowledge about the nursing implications and interventions related to assessment and diagnostic testing is also essential. Selecting medications and interpreting diagnostic tests are beyond the normal scope of the nurse.

The nurse educator is reviewing the assessment of cranial nerves. What should the educator identify as the specific instances when cranial nerves should be assessed? Select all that apply. A) When a neurogenic bladder develops B) When level of consciousness is decreased C) With brain stem pathology D) In the presence of peripheral nervous system disease E) When a spinal reflex is interrupted

Ans: B, C, D Feedback: Cranial nerves are assessed when level of consciousness is decreased, with brain stem pathology, or in the presence of peripheral nervous system disease. Abnormalities in muscle tone and involuntary movements are less likely to prompt the assessment of cranial nerves, since these nerves do not directly mediate most aspects of muscle tone and movement.

A patient had a lumbar puncture performed at the outpatient clinic and the nurse has phoned the patient and family that evening. What does this phone call enable the nurse to determine? A) What are the patient's and family's expectations of the test B) Whether the patient's family had any questions about why the test was necessary C) Whether the patient has had any complications of the test D) Whether the patient understood accurately why the test was done

Ans: C Feedback: Contacting the patient and family after diagnostic testing enables the nurse to determine whether they have any questions about the procedure or whether the patient had any untoward results. The other listed information should have been elicited from the patient and family prior to the test.

When caring for a patient with an altered level of consciousness, the nurse is preparing to test cranial nerve VII. What assessment technique would the nurse use to elicit a response from cranial nerve VII? A) Palpate trapezius muscle while patient shrugs should against resistance. B) Administer the whisper or watch-tick test. C) Observe for facial movement symmetry, such as a smile. D) Note any hoarseness in the patient's voice.

Ans: C Feedback: Cranial nerve VII is the facial nerve. An appropriate assessment technique for this cranial nerve would include observing for symmetry while the patient performs facial movements: smiles, whistles, elevates eyebrows, and frowns. Palpating and noting strength of the trapezius muscle while the patient shrugs shoulders against resistance would be completed to assess cranial nerve XI (spinal accessory). Assessing cranial nerve VIII (acoustic) would involve using the whisper or watch-tick test to evaluate hearing. Noting any hoarseness in the patient's voice would involve assessment of cranial nerve X (vagus)

A patient is scheduled for CT scanning of the head because of a recent onset of neurologic deficits. What should the nurse tell the patient in preparation for this test? A) "No metal objects can enter the procedure room." B) "You need to fast for 8 hours prior to the test." C) "You will need to lie still throughout the procedure." D) "There will be a lot of noise during the test."

Ans: C Feedback: Preparation for CT scanning includes teaching the patient about the need to lie quietly throughout the procedure. If the patient were having an MRI, metal and noise would be appropriate teaching topics. There is no need to fast prior to a CT scan of the brain.

During the performance of the Romberg test, the nurse observes that the patient sways slightly. What is the nurse's most appropriate action? A) Facilitate a referral to a neurologist. B) Reposition the patient supine to ensure safety. C) Document successful completion of the assessment. D) Follow up by having the patient perform the Rinne test.

Ans: C Feedback: Slight swaying during the Romberg test is normal, but a loss of balance is abnormal and is considered a positive Romberg test. Slight swaying is not a significant threat to the patient's safety. The Rinne test assesses hearing, not balance.

A patient is having a "fight or flight response" after receiving bad news about his prognosis. What affect will this have on the patient's sympathetic nervous system? A) Constriction of blood vessels in the heart muscle B) Constriction of bronchioles C) Increase in the secretion of sweat D) Constriction of pupils

Ans: C Feedback: Sympathetic nervous system stimulation results in dilated blood vessels in the heart and skeletal muscle, dilated bronchioles, increased secretion of sweat, and dilated pupils.

A trauma patient was admitted to the ICU with a brain injury. The patient had a change in level of consciousness, increased vital signs, and became diaphoretic and agitated. The nurse should recognize which of the following syndromes as the most plausible cause of these symptoms? A) Adrenal crisis B) Hypothalamic collapse C) Sympathetic storm D) Cranial nerve deficit

Ans: C Feedback: Sympathetic storm is a syndrome associated with changes in level of consciousness, altered vital signs, diaphoresis, and agitation that may result from hypothalamic stimulation of the sympathetic nervous system following traumatic brain injury. Alterations in cranial nerve or adrenal function would not have this result.

The nurse caring for an 80 year-old patient knows that she has a pre-existing history of dulled tactile sensation. The nurse should first consider what possible cause for this patient's diminished tactile sensation? A) Damage to cranial nerve VIII B) Adverse medication effects C) Age-related neurologic changes D) An undiagnosed cerebrovascular accident in early adulthood

Ans: C Feedback: Tactile sensation is dulled in the elderly person due to a decrease in the number of sensory receptors. While thorough assessment is necessary, it is possible that this change is unrelated to pathophysiological processes.

A patient in the OR goes into malignant hyperthermia due to an abnormal reaction to the anesthetic. The nurse knows that the area of the brain that regulates body temperature is which of the following? A) Cerebellum B) Thalamus C) Hypothalamus D) Midbrain

Ans: C Feedback: The hypothalamus plays an important role in the endocrine system because it regulates the pituitary secretion of hormones that influence metabolism, reproduction, stress response, and urine production. It works with the pituitary to maintain fluid balance through hormonal release and maintains temperature regulation by promoting vasoconstriction or vasodilatation. The cerebellum, thalamus, and midbrain and not directly involved in temperature regulation.

The nurse is caring for a patient with an upper motor neuron lesion. What clinical manifestations should the nurse anticipate when planning the patient's neurologic assessment? A) Decreased muscle tone B) Flaccid paralysis C) Loss of voluntary control of movement D) Slow reflexes

Ans: C Upper motor neuron lesions do not cause muscle atrophy, flaccid paralysis, or slow reflexes. However, upper motor neuron lesions normally cause loss of voluntary control.

A 72-year-old man has been brought to his primary care provider by his daughter, who claims that he has been experiencing uncharacteristic lapses in memory. What principle should underlie the nurse's assessment and management of this patient? A) Loss of short-term memory is normal in older adults, but loss of long-term memory is pathologic. B) Lapses in memory in older adults are considered benign unless they have negative consequences. C) Gradual increases in confusion accompany the aging process. D) Thorough assessment is necessary because changes in cognition are always considered to be pathologic

Ans: D Feedback: Although mental processing time decreases with age, memory, language, and judgment capacities remain intact. Change in mental status should never be assumed to be a normal part of aging.

The neurologic nurse is testing the function of a patient's cerebellum and basal ganglia. What action will most accurately test these structures? A) Have the patient identify the location of a cotton swab on his or her skin with the eyes closed. B) Elicit the patient's response to a hypothetical problem. C) Ask the patient to close his or her eyes and discern between hot and cold stimuli. D) Guide the patient through the performance of rapid, alternating movements.

Ans: D Feedback: Cerebellar and basal ganglia influence on the motor system is reflected in balance control and coordination. Coordination in the hands and upper extremities is tested by having the patient perform rapid, alternating movements and point-to-point testing. The cerebellum and basal ganglia do not mediate cutaneous sensation or judgment.

A nurse is caring for a patient diagnosed with Ménière's disease. While completing a neurologic examination on the patient, the nurse assesses cranial nerve VIII. The nurse would be correct in identifying the function of this nerve as what? A) Movement of the tongue B) Visual acuity C) Sense of smell D) Hearing and equilibrium

Ans: D Feedback: Cranial nerve VIII (acoustic) is responsible for hearing and equilibrium. Cranial nerve XII (hypoglossal) is responsible for movement of the tongue. Cranial nerve II (optic) is responsible for visual acuity and visual fields. Cranial nerve I (olfactory) functions in sense of smell.

The nursing students are learning how to assess function of cranial nerve VIII. To assess the function of cranial nerve VIII the students would be correct in completing which of the following assessment techniques? A) Have the patient identify familiar odors with the eyes closed. B) Assess papillary reflex. C) Utilize the Snellen chart. D) Test for air and bone conduction (Rinne test).

Ans: D Feedback: Cranial nerve VIII is the acoustic nerve. It functions in hearing and equilibrium. When assessing this nerve, the nurse would test for air and bone conduction (Rinne) with a tuning fork. Assessment of papillary reflex would be completed for cranial nerves III (oculomotor), IV (trochlear), and VI (abducens). The Snellen chart would be used to assess cranial nerve II (optic).

A patient for whom the nurse is caring has positron emission tomography (PET) scheduled. In preparation, what should the nurse explain to the patient? A) The test will temporarily limit blood flow through the brain. B) An allergy to iodine precludes getting the radio-opaque dye. C) The patient will need to endure loud noises during the test. D) The test may result in dizziness or lightheadedness.

Ans: D Feedback: Key nursing interventions for PET scan include explaining the test and teaching the patient about inhalation techniques and the sensations (e.g., dizziness, light-headedness, and headache) that may occur. A PET scan does not impede blood flow through the brain. An allergy to iodine precludes the dye for an MRI, and loud noise is heard in an MRI.

The nurse is conducting a focused neurologic assessment. When assessing the patient's cranial nerve function, the nurse would include which of the following assessments? A) Assessment of hand grip B) Assessment of orientation to person, time, and place C) Assessment of arm drift D) Assessment of gag reflex

Ans: D Feedback: The gag reflex is governed by the glossopharyngeal nerve, one of the cranial nerves. Hand grip and arm drifting are part of motor function assessment. Orientation is an assessment parameter related to a mental status examination.

A patient is currently being stimulated by the parasympathetic nervous system. What effect will this nervous stimulation have on the patient's bladder? A) The parasympathetic nervous system causes urinary retention. B) The parasympathetic nervous system causes bladder spasms. C) The parasympathetic nervous system causes urge incontinence. D) The parasympathetic nervous system makes the bladder contract.

Ans: D Feedback: The parasympathetic division of the nervous system causes contraction (stimulation) of the urinary bladder muscles and a decrease (inhibition) in heart rate, whereas the sympathetic division produces relaxation (inhibition) of the urinary bladder and an increase (stimulation) in the rate and force of the heartbeat.

The nurse is admitting a patient to the unit who is diagnosed with a lower motor neuron lesion. What entry in the patient's electronic record is most consistent with this diagnosis? A) "Patient exhibits increased muscle tone." B) "Patient demonstrates normal muscle structure with no evidence of atrophy." C) "Patient demonstrates hyperactive deep tendon reflexes." D) "Patient demonstrates an absence of deep tendon reflexes."

Ans: D Lower motor neuron lesions cause flaccid muscle paralysis, muscle atrophy, decreased muscle tone, and loss of voluntary control.

An elderly patient is being discharged home. The patient lives alone and has atrophy of his olfactory organs. The nurse tells the patient's family that it is essential that the patient have what installed in the home? A) Grab bars B) Nonslip mats C) Baseboard heaters D) A smoke detector

Ans: D The sense of smell deteriorates with age. The olfactory organs are responsible for smell. This may present a safety hazard for the patient because he or she may not smell smoke or gas leaks. Smoke detectors are universally necessary, but especially for this patient.

What term is used to describe the fibrous connective tissue that hugs the brain closely and extends into every fold of the brain's surface? A) Dura mater B) Arachnoid C) Fascia D) Pia mater

Ans: D The term "meninges" describes the fibrous connective tissue that covers the brain and spinal cord. The meninges have three layers, the dura mater, arachnoid, and pia mater. The pia mater is the innermost membrane that hugs the brain closely and extends into every fold of the brain's surface. The dura mater, the outermost layer, covers the brain and spinal cord. The arachnoid, the middle membrane, is responsible for the production of cerebrospinal fluid.

3 Auto-positive end-expiratory pressure (auto-PEEP) is caused by inadequate exhalation time. Barotrauma, hemodynamic instability and increased work of breathing are the results and not the causes of the auto-PEEP. Text Reference - p. 1621

Auto-positive end-expiratory pressure (PEEP) is the additional PEEP over what is set by the health care provider. What causes auto-PEEP during mechanical ventilation? 1 Barotrauma 2 Hemodynamic instability 3 Inadequate exhalation time 4 Increased work of breathing

A patient is brought to the ER following a motor vehicle accident in which he sustained head trauma. Preliminary assessment reveals a vision deficit in the patient's left eye. The nurse should associate this abnormal finding with trauma to which of the following cerebral lobes? A)Temporal B)Occipital C)Parietal D)Frontal

B The posterior lobe of the cerebral hemisphere is responsible for visual interpretation. The temporal lobe contains the auditory receptive areas. The parietal lobe contains the primary sensory cortex, and is essential to an individual's awareness of the body in space, as well as orientation in space and spatial relations. The frontal lobe functions in concentration, abstract thought, information storage or memory, and motor function.

Physiological Integrity 13. When the nurse is caring for an obese patient with left lower-lobe pneumonia, gas exchange will be best when the patient is positioned a. on the left side. b. on the right side. c. in the high-Fowler's position. d. in the tripod position.

B Rationale: The patient should be positioned with the "good" lung in the dependent position to improve the match between ventilation and perfusion. The obese patient's abdomen will limit respiratory excursion when sitting in the high-Fowler's or tripod positions. Cognitive Level: Comprehension Text Reference: pp. 1809-1810 Nursing Process: Implementation

Psychosocial Integrity 23. The nurse obtains the vital signs for a patient admitted 2 days ago with gram-negative sepsis: temperature 101.2° F, blood pressure 90/56 mm Hg, pulse 92, respirations 34. Which action should the nurse take next? a. Notify the health care provider of the patient's vital signs. b. Obtain oxygen saturation using pulse oximetry. c. Document the vital signs and continue to monitor. d. Administer PRN acetaminophen (Tylenol) 650 mg.

B Rationale: The patient's increased respiratory rate in combination with the admission diagnosis of gram-negative sepsis indicates that acute respiratory distress syndrome (ARDS) may be developing; the nurse should check for hypoxemia, a hallmark of ARDS. The health care provider should be notified after further assessment of the patient. Documentation and continued monitoring of the vital signs are needed but do not constitute an adequate response to the patient situation. Tylenol administration is appropriate but not the highest priority for this patient. Cognitive Level: Application Text Reference: pp. 1813-1814 Nursing Process: Implementation

A gerontologic nurse planning the neurologic assessment of an older adult is considering normal, age-related changes. Of what phenomenon should the nurse be aware? A) Hyperactive deep tendon reflexes B) Reduction in cerebral blood flow C) Increased cerebral metabolism D) Hypersensitivity to painful stimuli

B Reduction in cerebral blood flow (CBF) is a change that occurs in the normal aging process. Deep tendon reflexes can be decreased or, in some cases, absent. Cerebral metabolism decreases as the patient advances in age. Reaction to painful stimuli may be decreased with age. Because pain is an important warning signal, caution must be used when hot or cold packs are used.

A 78 yo man has confusion and Temp of 104 F. He is a diabetic with purulent drainage from his right great toe. His assessment findings are BP 84/80, HR 110, RR 42 and shallow, CO 8L/min, and PAWP 4 mmHg. The pts sxs are most likely indicative of A) sepsis B) Septic shock C) Multiple organ dysfunction syndrome D) Systemic inflammatory response syndrome

B) Septic shock

Shock occurs when tissue perfusion is inadequate to deliver oxygen and nutrients to support cellular function. When caring for patients who may develop indicators of shock, the nurse is aware that the most important measurement of shock is:

Blood pressure pg. 286-287

Shock occurs when tissue perfusion is inadequate to deliver oxygen and nutrients to support cellular function. When caring for patients who may develop indicators of shock, the nurse is aware that the most important measurement of shock is:

Blood pressure.

Which of the following shock states is (are) characterized by acute, severe bronchoconstriction? A. Cardiogenic B. Anaphylactic C. Hypovolemic D. All of the above

B. Anaphylactic

A nurse is assessing reflexes in a patient with hyperactive reflexes. When the patient's foot is abruptly dorsiflexed, it continues to "beat" two to three times before settling into a resting position. How would the nurse document this finding? A) Rigidity B) Flaccidity C) Clonus D) Ataxia

C When reflexes are very hyperactive, a phenomenon called clonus may be elicited. If the foot is abruptly dorsiflexed, it may continue to "beat" two to three times before it settles into a position of rest. Rigidity is an increase in muscle tone at rest characterized by increased resistance to passive stretch. Flaccidity is lack of muscle tone. Ataxia is the inability to coordinate muscle movements, resulting in difficulty walking, talking, and performing self-care activities.

Physiological Integrity 24. Which of these nursing actions included in the care of a mechanically ventilated patient with acute respiratory distress syndrome (ARDS) is most appropriate for the RN to delegate to an experienced LPN/LVN working in the intensive care unit? a. Placing the patient in the prone position b. Assessment of patient breath sounds c. Administration of enteral tube feedings d. Obtaining the pulmonary artery pressures

C Rationale: Administration of tube feedings is included in LPN/LVN education and scope of practice and can be safely delegated to an LPN/LVN who is experienced in caring for critically ill patients. Placing a patient who is on a ventilator in the prone position requires multiple staff and should be supervised by an RN. Assessment of breath sounds and obtaining pulmonary artery pressures require advanced assessment skills and should be done by the RN caring for a critically ill patient. Cognitive Level: Application Text Reference: pp. 1816-1818 Nursing Process: Implementation

1. It will be most important for the nurse to check pulse oximetry for which of these patients? a. A patient with emphysema and a respiratory rate of 16 b. A patient with massive obesity who is refusing to get out of bed c. A patient with pneumonia who has just been admitted to the unit d. A patient who has just received morphine sulfate for postoperative pain

C Rationale: Hypoxemia and hypoxemic respiratory failure are caused by disorders that interfere with the transfer of oxygen into the blood, such as pneumonia. The other listed disorders are more likely to cause problems with hypercapnia because of ventilatory failure. Cognitive Level: Application Text Reference: pp. 1799-1800 Nursing Process: Assessment

Physiological Integrity 3. When a patient is diagnosed with pulmonary fibrosis, the nurse will teach the patient about the risk for poor oxygenation because of a. too-rapid movement of blood flow through the pulmonary blood vessels. b. incomplete filling of the alveoli with air because of reduced respiratory ability. c. decreased transfer of oxygen into the blood because of thickening of the alveoli. d. mismatch between lung ventilation and blood flow through the blood vessels of the lung.

C Rationale: Pulmonary fibrosis causes the alveolar-capillary interface to become thicker, which increases the amount of time it takes for gas to diffuse across the membrane. Too-rapid pulmonary blood flow is another cause of shunt but does not describe the pathology of pulmonary fibrosis. Decrease in alveolar ventilation will cause hypercapnia. Ventilation and perfusion are matched in pulmonary fibrosis; the problem is with diffusion. Cognitive Level: Application Text Reference: p. 1802 Nursing Process: Implementation

Physiological Integrity 16. When caring for a patient who developed acute respiratory distress syndrome (ARDS) as a result of a urinary tract infection (UTI), the nurse is asked by the patient's family how a urinary tract infection could cause lung damage. Which response by the nurse is appropriate? a. "The infection spread through the circulation from the urinary tract to the lungs." b. "The urinary tract infection produced toxins that damaged the lungs." c. "The infection caused generalized inflammation that damaged the lungs." d. "The fever associated with the infection led to scar tissue formation in the lungs."

C Rationale: The pathophysiologic changes that occur in ARDS are thought to be caused by inflammatory and immune reactions that lead to changes at the alveolar-capillary membrane. ARDS is not directly caused by infection, toxins, or fever. Cognitive Level: Application Text Reference: p. 1813 Nursing Process: Implementation

Physiological Integrity 21. When prone positioning is used in the care of a patient with acute respiratory distress syndrome (ARDS), which information obtained by the nurse indicates that the positioning is effective? a. The skin on the patient's back is intact and without redness. b. Sputum and blood cultures show no growth after 24 hours. c. The patient's PaO2 is 90 mm Hg, and the SaO2 is 92%. d. Endotracheal suctioning results in minimal mucous return.

C Rationale: The purpose of prone positioning is to improve the patient's oxygenation as indicated by the PaO2 and SaO2. The other information will be collected but does not indicate whether prone positioning has been effective. Cognitive Level: Application Text Reference: pp. 1817-1818 Nursing Process: Evaluation

Low levels of which molecule contribute to the pathophysiology of septic shock? A. Nitric oxide B. Interleukin 1 C. Activated protein C D. Epinephrine

C. Activated protein C

Why do individuals with severe burns have difficulty breathing and often require intubation, even if there was no smoke inhalation causing acute lung injury? A. Severe pulmonary edema develops immediately following all severe burn injuries. B. Pulmonary emboli typically form after severe burn injuries. C. Airway edema is a common occurrence with severe burn injuries. D. Pneumothorax and pleural effusions occur with severe burn injuries.

C. Airway edema is a common occurrence with severe burn injuries.

Which of the following pathophysiological events causes the severe hypotension observed in neurogenic shock? A. Increased capillary permeability B. Diuresis C. Decreased peripheral vascular resistance D. All of the above

C. Decreased peripheral vascular resistance

Which burn injury involves most of the dermis and leaves only a few epidermal appendages intact? A. First degree B. Superficial partial thickness (second degree) C. Deep partial thickness (second degree) D. Third degree

C. Deep partial thickness (second degree)

Which of the following shock states is (are) characterized by vasodilation of the systemic arteries? A. Hypovolemic B. Cardiogenic C. Distributive D. All of the above

C. Distributive

A consequence of switching from aerobic to anaerobic cellular metabolism during shock states is: A. increased ATP production. B. cellular dehydration. C. lactic acidosis. D. free radical formation.

C. lactic acidosis.

Which of the following type of shock are older adults more likely to develop?

Cardiogenic shock

Older adults are more likely to develop which type of shock?

Cardiogenic shock pg. 299

What is the correct order of the pathophysiologic steps involved in systemic inflammatory response syndrome (SIRS)? 1. Activation of coagulation cascade 2. Release of mediators 3. Increase in vascular permeability 4. Leakage of mediators and proteins into interstitial space 5. Digestion of foreign debris by white blood cells

Correct 1. Release of mediators Correct 2. Increase in vascular permeability Correct 3. Leakage of mediators and proteins into interstitial space Correct 4. Digestion of foreign debris by white blood cells Correct 5. Activation of coagulation cascade Release of mediators occurs in a patient with systemic inflammatory response syndrome (SIRS) when the inflammatory response is activated. This release increases the vascular permeability. As a result, mediators and proteins leak out of the blood vessels into the interstitial space. White blood cells digest the foreign debris and the coagulation cascade is activated. Text Reference - p. 1649

5. When assessing the hemodynamic information for a newly admitted patient in shock of unknown etiology, the nurse will anticipate administration of large volumes of crystalloids when the a. cardiac output is increased and the central venous pressure (CVP) is low. b. pulmonary artery wedge pressure (PAWP) is increased, and the urine output is low. c. heart rate is decreased, and the systemic vascular resistance is low. d. cardiac output is decreased and the PAWP is high.

Correct Answer: A Rationale: A high cardiac output and low CVP suggest septic shock, and massive fluid replacement is indicated. Increased PAWP indicates that the patient has excessive fluid volume (and suggests cardiogenic shock), and diuresis is indicated. Bradycardia and a low systemic vascular resistance (SVR) suggest neurogenic shock, and fluids should be infused cautiously.

19. While assessing a patient in shock who has an arterial line in place, the nurse notes a drop in the systolic BP from 92 mm Hg to 76 mm Hg when the head of the patient's bed is elevated to 75 degrees. This finding indicates a need for a. additional fluid replacement. b. antibiotic administration. c. infusion of a sympathomimetic drug. d. administration of increased oxygen.

Correct Answer: A Rationale: A postural drop in BP is an indication of volume depletion and suggests the need for additional fluid infusions. There are no data to suggest that antibiotics, sympathomimetics, or additional oxygen are needed.

21. A patient outcome that is appropriate for the patient in shock who has a nursing diagnosis of decreased cardiac output related to relative hypovolemia is a. urine output of 0.5 ml/kg/hr. b. decreased peripheral edema. c. decreased CVP. d. oxygen saturation 90% or more.

Correct Answer: A Rationale: A urine output of 0.5 ml/kg/hr indicates adequate renal perfusion, which is a good indicator of cardiac output. The patient may continue to have peripheral edema because fluid infusions may be needed despite third-spacing of fluids in relative hypovolemia. Decreased central venous pressure (CVP) for a patient with relative hypovolemia indicates that additional fluid infusion is necessary. An oxygen saturation of 90% will not necessarily indicate that cardiac output has improved.

12. All of these collaborative interventions are ordered by the health care provider for a patient stung by a bee who develops severe respiratory distress and faintness. Which one will the nurse administer first? a. Epinephrine (Adrenalin) b. Normal saline infusion c. Dexamethasone (Decadron) d. Diphenhydramine (Benadryl)

Correct Answer: A Rationale: Epinephrine rapidly causes peripheral vasoconstriction, dilates the bronchi, and blocks the effects of histamine and reverses the vasodilation, bronchoconstriction, and histamine release that cause the symptoms of anaphylaxis. The other interventions are also appropriate but would not be the first ones administered.

13. A patient with a myocardial infarction (MI) and cardiogenic shock has the following vital signs: BP 86/50, pulse 126, respirations 30. Hemodynamic monitoring reveals an elevated PAWP and decreased cardiac output. The nurse will anticipate a. administration of furosemide (Lasix) IV. b. titration of an epinephrine (Adrenalin) drip. c. administration of a normal saline bolus. d. assisting with endotracheal intubation.

Correct Answer: A Rationale: The PAWP indicates that the patient's preload is elevated and furosemide is indicated to reduce the preload and improve cardiac output. Epinephrine would further increase myocardial oxygen demand and might extend the MI. The PAWP is already elevated, so normal saline boluses would be contraindicated. There is no indication that the patient requires endotracheal intubation.

4. While caring for a seriously ill patient, the nurse determines that the patient may be in the compensatory stage of shock on finding a. cold, mottled extremities. b. restlessness and apprehension. c. a heart rate of 120 and cool, clammy skin. d. systolic BP less than 90 mm Hg.

Correct Answer: B Rationale: Restlessness and apprehension are typical during the compensatory stage of shock. Cold, mottled extremities, cool and clammy skin, and a systolic BP less than 90 are associated with the progressive and refractory stages.

6. A patient who has been involved in a motor-vehicle crash is admitted to the ED with cool, clammy skin, tachycardia, and hypotension. All of these orders are written. Which one will the nurse act on first? a. Insert two 14-gauge IV catheters. b. Administer oxygen at 100% per non-rebreather mask. c. Place the patient on continuous cardiac monitor. d. Draw blood to type and crossmatch for transfusions.

Correct Answer: B Rationale: The first priority in the initial management of shock is maintenance of the airway and ventilation. Cardiac monitoring, insertion of IV catheters, and obtaining blood for transfusions should also be rapidly accomplished, but only after actions to maximize oxygen delivery have been implemented.

11. A patient who is receiving chemotherapy is admitted to the hospital with acute dehydration caused by nausea and vomiting. Which action will the nurse include in the plan of care to best prevent the development of shock, systemic inflammatory response syndrome (SIRS), and multiorgan dysfunction syndrome (MODS)? a. Administer all medications through the patient's indwelling central line. b. Place the patient in a private room. c. Restrict the patient to foods that have been well-cooked or processed. d. Insert a nasogastric (NG) tube for enteral feeding.

Correct Answer: B Rationale: The patient who has received chemotherapy is immune compromised, and placing the patient in a private room will decrease the exposure to other patients and reduce infection/sepsis risk. Administration of medications through the central line increases the risk for infection and sepsis. There is no indication that the patient is neutropenic, and restricting the patient to cooked and processed foods is likely to decrease oral intake further and cause further malnutrition, a risk factor for sepsis and shock. Insertion of an NG tube is invasive and will not decrease the patient's nausea and vomiting.

14. The triage nurse receives a call from a community member who is driving an unconscious friend with multiple injuries after a motorcycle accident to the hospital. The caller states that they will be arriving in 1 minute. In preparation for the patient's arrival, the nurse will obtain a. a liter of lactated Ringer's solution. b. 500 ml of 5% albumin. c. two 14-gauge IV catheters. d. a retention catheter.

Correct Answer: C Rationale: A patient with multiple trauma may require fluid resuscitation to prevent or treat hypovolemic shock, so the nurse will anticipate the need for 2 large bore IV lines to administer normal saline. Lactated Ringer's solution should be used cautiously and will not be ordered until the patient had been assessed for possible liver abnormalities. Although colloids may sometimes be used for volume expansion, it is generally accepted that crystalloids should be used as the initial therapy for fluid resuscitation. A catheter would likely be ordered, but in the 1 minute that the nurse has to obtain supplies, the IV catheters would take priority.

27. When caring for a patient who has just been admitted with septic shock, which of these assessment data will be of greatest concern to the nurse? a. BP 88/56 mm Hg b. Apical pulse 110 beats/min c. Urine output 15 ml for 2 hours d. Arterial oxygen saturation 90%

Correct Answer: C Rationale: The best data for assessing the adequacy of cardiac output are those that provide information about end-organ perfusion such as urine output by the kidneys. The low urine output is an indicator that renal tissue perfusion is inadequate and the patient is in the progressive stage of shock. The low BP, increase in pulse, and low-normal O2 saturation are more typical of compensated septic shock.

9. The patient with neurogenic shock is receiving a phenylephrine (Neo-Synephrine) infusion through a left-forearm IV. Which assessment information obtained by the nurse indicates a need for immediate action? a. The patient has an apical pulse rate of 58 beats/min. b. The patient's urine output has been 28 ml over the last hour. c. The patient's IV infusion site is cool and pale. d. The patient has warm, dry skin on the extremities.

Correct Answer: C Rationale: The coldness and pallor at the infusion site suggest extravasation of the Neo-Synephrine. The nurse should discontinue the IV and, if possible, infuse the medication into a central line. An apical pulse of 58 is typical for neurogenic shock but does not indicate an immediate need for nursing intervention. A 28-ml output over 1 hour would require the nurse to monitor the output over the next hour, but an immediate change in therapy is not indicated. Warm, dry skin indicates that the patient is in early neurogenic shock.

7. A patient with massive trauma and possible spinal cord injury is admitted to the ED. The nurse suspects that the patient may be experiencing neurogenic shock in addition to hypovolemic shock, based on the finding of a. cool, clammy skin. b. shortness of breath. c. heart rate of 48 beats/min d. BP of 82/40 mm Hg.

Correct Answer: C Rationale: The normal sympathetic response to shock/hypotension is an increase in heart rate. The presence of bradycardia suggests unopposed parasympathetic function, as occurs in neurogenic shock. The other symptoms are consistent with hypovolemic shock.

3. A patient with hypovolemic shock has a urinary output of 15 ml/hr. The nurse understands that the compensatory physiologic mechanism that leads to altered urinary output is a. activation of the sympathetic nervous system (SNS), causing vasodilation of the renal arteries. b. stimulation of cardiac -adrenergic receptors, leading to increased cardiac output. c. release of aldosterone and antidiuretic hormone (ADH), which cause sodium and water retention. d. movement of interstitial fluid to the intravascular space, increasing renal blood flow.

Correct Answer: C Rationale: The release of aldosterone and ADH lead to the decrease in urine output by increasing the reabsorption of sodium and water in the renal tubules. SNS stimulation leads to renal artery vasoconstriction. -Receptor stimulation does increase cardiac output, but this would improve urine output. During shock, fluid leaks from the intravascular space into the interstitial space.

10. A patient in septic shock has not responded to fluid resuscitation, as evidenced by a decreasing BP and cardiac output. The nurse anticipates the administration of a. nitroglycerine (Tridil). b. dobutamine (Dobutrex). c. norepinephrine (Levophed). d. sodium nitroprusside (Nipride).

Correct Answer: C Rationale: When fluid resuscitation is unsuccessful, administration of vasopressor drugs is used to increase the systemic vascular resistance (SVR) and improve tissue perfusion. Nitroglycerin would decrease the preload and further drop cardiac output and BP. Dobutamine will increase stroke volume, but it would also further decrease SVR. Nitroprusside is an arterial vasodilator and would further decrease SVR.

16. The nurse is caring for a patient admitted with a urinary tract infection and sepsis. Which information obtained in the assessment indicates a need for a change in therapy? a. The patient is restless and anxious. b. The patient has a heart rate of 134. c. The patient has hypotonic bowel sounds. d. The patient has a temperature of 94.1° F.

Correct Answer: D Rationale: Hypothermia is an indication that the patient is in the progressive stage of shock. The other data are consistent with compensated shock.

20. The best nursing intervention for a patient in shock who has a nursing diagnosis of fear related to perceived threat of death is to a. arrange for the hospital pastoral care staff to visit the patient. b. ask the health care provider to prescribe a sedative drug for the patient. c. leave the patient alone with family members whenever possible. d. place the patient's call bell where it can be easily reached.

Correct Answer: D Rationale: The patient who is fearful should feel that the nurse is immediately available if needed. Pastoral care staff should be asked to visit only after checking with the patient to determine whether this is desired. Providing time for family to spend with the patient is appropriate, but patients and family should not feel that the nurse is unavailable. Sedative administration is helpful but does not as directly address the patient's anxiety about dying.

2. A diabetic patient who has had vomiting and diarrhea for the past 3 days is admitted to the hospital with a blood glucose of 748 mg/ml (41.5 mmol/L) and a urinary output of 120 ml in the first hour. The vital signs are blood pressure (BP) 72/62; pulse 128, irregular and thready; respirations 38; and temperature 97° F (36.1° C). The patient is disoriented and lethargic with cold, clammy skin and cyanosis in the hands and feet. The nurse recognizes that the patient is experiencing the a. progressive stage of septic shock. b. compensatory stage of diabetic shock. c. refractory stage of cardiogenic shock. d. progressive stage of hypovolemic shock.

Correct Answer: D Rationale: The patient's history of hyperglycemia (and the associated polyuria), vomiting, and diarrhea is consistent with hypovolemia, and the symptoms are most consistent with the progressive stage of shock. The patient's temperature of 97° F is inconsistent with septic shock. The history is inconsistent with a diagnosis of cardiogenic shock, and the patient's neurologic status is not consistent with refractory shock.

The nurse is caring for a patient brought to the hospital with complaints of vomiting with suspected respiratory failure caused by sepsis. What early clinical manifestation of the condition does the nurse document? Select all that apply. 1 Organ dysfunction 2 White-out chest x-ray 3 Cough and restlessness 4 Dyspnea and tachypnea 5 Normal chest auscultation

Correct3 Cough and restlessness Correct4 Dyspnea and tachypnea Correct5 Normal chest auscultation Early clinical manifestations of acute respiratory distress syndrome (ARDS) usually appear within 24 to 48 hours after a lung injury. The patient may cough and feel restless and exhibit signs of dyspnea and tachypnea. Chest auscultation usually reveals normal crackling or rhonchi breathing sounds. Organ dysfunction may develop later if therapy is not promptly started. In the advanced stage of ARDS, the chest x-ray is white-out and shows empty air spaces.

18. When assessing a patient with chronic obstructive pulmonary disease (COPD), the nurse finds a new onset of agitation and confusion. Which action should the nurse take first? a. Notify the health care provider. b. Check pupils for reaction to light. c. Attempt to calm and reorient the patient. d. Assess oxygenation using pulse oximetry.

D

20. The nurse is caring for a 78-year-old patient who was hospitalized 2 days earlier with community-acquired pneumonia. Which assessment information is most important to communicate to the health care provider? a. Scattered crackles bilaterally in the posterior lung bases. b. Persistent cough that is productive of blood-tinged sputum. c. Temperature of 101.5° F (38.6° C) after 2 days of IV antibiotic therapy. d. Decreased oxygen saturation to 90% with 100% O2 by non-rebreather mask.

D

Physiological Integrity 18. After prolonged cardiopulmonary bypass, a patient develops increasing shortness of breath and hypoxemia. To determine whether the patient has acute respiratory distress syndrome (ARDS) or pulmonary edema caused by left ventricular failure, the nurse will anticipate assisting with a. positioning the patient for a chest radiograph. b. drawing blood for arterial blood gases. c. obtaining a ventilation-perfusion scan. d. inserting a pulmonary artery catheter.

D Rationale: Pulmonary artery wedge pressure will remain at normal levels in the patient with ARDS because the fluid in the alveoli is caused by increased permeability of the alveolar-capillary membrane rather than by the backup of fluid from the lungs (as occurs in cardiogenic pulmonary edema). The other tests will not help in differentiating cardiogenic from noncardiogenic pulmonary edema. Cognitive Level: Application Text Reference: p. 1815 Nursing Process: Implementation

Physiological Integrity 7. A patient with chronic obstructive pulmonary disease (COPD) arrives in the emergency department complaining of acute respiratory distress. When monitoring the patient, which assessment by the nurse will be of most concern? a. The patient is sitting in the tripod position. b. The patient has bibasilar lung crackles. c. The patient's pulse oximetry indicates an O2 saturation of 91%. d. The patient's respiratory rate has decreased from 30 to 10/min.

D Rationale: A decrease in respiratory rate in a patient with respiratory distress suggests the onset of fatigue and a high risk for respiratory arrest; therefore, the nurse will need to take immediate action. Patients who are experiencing respiratory distress frequently sit in the tripod position because it decreases the work of breathing. Crackles in the lung bases may be the baseline for a patient with COPD. An oxygen saturation of 91% is common in patients with COPD and will provide adequate gas exchange and tissue oxygenation. Cognitive Level: Application Text Reference: p. 1804 Nursing Process: Assessment

Physiological Integrity 4. A patient is diagnosed with a large pulmonary embolism. When explaining to the patient what has happened to cause respiratory failure, which information will the nurse include? a. "Oxygen transfer into your blood is slow because of thick membranes between the small air sacs and the lung circulation." b. "Thick secretions in your small airways are blocking air from moving into the small air sacs in your lungs." c. "Large areas of your lungs are getting good blood flow but are not receiving enough air to fill the small air sacs." d. "Blood flow though some areas of your lungs is decreased even though you are taking adequate breaths."

D Rationale: A pulmonary embolus limits blood flow but does not affect ventilation, leading to a ventilation-perfusion mismatch. The response beginning, "Oxygen transfer into your blood is slow because of thick membranes" describes a diffusion problem. The remaining two responses describe ventilation-perfusion mismatch with adequate blood flow but poor ventilation. Cognitive Level: Application Text Reference: p. 1802 Nursing Process: Implementation

Physiological Integrity 20. Which statement by the nurse when explaining the purpose of positive end-expiratory pressure (PEEP) to the family members of a patient with ARDS is correct? a. "PEEP will prevent fibrosis of the lung from occurring." b. "PEEP will push more air into the lungs during inhalation." c. "PEEP allows the ventilator to deliver 100% oxygen to the lungs." d. "PEEP prevents the lung air sacs from collapsing during exhalation."

D Rationale: By preventing alveolar collapse during expiration, PEEP improves gas exchange and oxygenation. PEEP will not prevent the fibrotic changes that occur with ARDS, push more air into the lungs, or change the fraction of inspired oxygen (FIO2) delivered to the patient. Cognitive Level: Comprehension Text Reference: p. 1817 Nursing Process: Planning

Physiological Integrity 10. While caring for a patient who has been admitted with a pulmonary embolism, the nurse notes a change in the patient's arterial oxyhemoglobin saturation (SpO2) from 94% to 88%. The nurse will a. assist the patient to cough and deep-breathe. b. help the patient to sit in a more upright position. c. suction the patient's oropharynx. d. increase the oxygen flow rate.

D Rationale: Increasing oxygen flow rate will usually improve oxygen saturation in patients with ventilation-perfusion mismatch, as occurs with pulmonary embolism. Because the problem is with perfusion, actions that improve ventilation, such as deep-breathing and coughing, sitting upright, and suctioning, are not likely to improve oxygenation. Cognitive Level: Application Text Reference: pp. 1802, 1807 Nursing Process: Implementation

Physiological Integrity 6. When assessing a patient with chronic lung disease, the nurse finds a sudden onset of agitation and confusion. Which action should the nurse take first? a. Monitor the patient every 10 to 15 minutes. b. Notify the patient's health care provider immediately. c. Attempt to calm and reassure the patient. d. Assess vital signs and pulse oximetry.

D Rationale: The nurse needs to collect additional clinical data to share with the health care provider and to start interventions quickly if appropriate (e.g., increased oxygen flow if hypoxic). The change in the patient's neurologic status may indicate deterioration in respiratory function, and the health care provider should be notified immediately but only after some additional information is obtained. Monitoring the patient and attempting to calm the patient are appropriate actions, but they will not prevent further deterioration of the patient's clinical status and may delay care. Cognitive Level: Application Text Reference: pp. 1804-1805 Nursing Process: Assessment

Physiological Integrity 12. A patient in acute respiratory failure as a complication of COPD has a PaCO2 of 65 mm Hg, rhonchi audible in the right lung, and marked fatigue with a weak cough. The nurse will plan to a. allow the patient to rest to help conserve energy. b. arrange for a humidifier to be placed in the patient's room. c. position the patient on the right side with the head of the bed elevated. d. assist the patient with augmented coughing to remove respiratory secretions.

D Rationale: The patient's assessment indicates that assisted coughing is needed to help remove secretions, which will improve PaCO2 and will also help to correct fatigue. If the patient is allowed to rest, the PaCO2 will increase. Humidification may help loosen secretions, but the weak cough effort will prevent the secretions from being cleared. The patient should be positioned with the good lung down to improve gas exchange. Cognitive Level: Application Text Reference: p. 1809 Nursing Process: Planning

Physiological Integrity 15. The nurse is caring for a patient who was hospitalized 2 days earlier with aspiration pneumonia. Which assessment information is most important to communicate to the health care provider? a. The patient has a cough that is productive of blood-tinged sputum. b. The patient has scattered crackles throughout the posterior lung bases. c. The patient's temperature is 101.5° F after 2 days of IV antibiotic therapy. d. The patient's SpO2 has dropped to 90%, although the O2 flow rate has been increased.

D Rationale: The patient's dropping SpO2 despite having an increase in FIO2 indicates the possibility of acute respiratory distress syndrome (ARDS). The patient's blood-tinged sputum and scattered crackles are not unusual in a patient with pneumonia, although they do require continued monitoring. The continued temperature elevation indicates a possible need to change antibiotics, but this is not as urgent a concern as the progression toward hypoxemia despite an increase in O2 flow rate. Cognitive Level: Application Text Reference: p. 1815 Nursing Process: Assessment

Which of the following features is highly characteristic of a septic shock state? A. High peripheral vascular resistance B. Inhibition of the sympathetic nervous system C. Metabolic alkalosis D. Hypermetabolism

D. Hypermetabolism

In the cardiac cycle, the ventricles contract during: A. the refractory period. B. diastole. C. repolarization. D. systole.

D. systole.

A 68-yr-old male patient diagnosed with sepsis is orally intubated on mechanical ventilation. Which nursing action is most important? Use the open-suctioning technique. Administer morphine for discomfort. Limit noise and cluster care activities. Elevate the head of the bed 30 degrees.

Elevate the head of the bed 30 degrees. The two major complications of endotracheal intubation are unplanned extubation and aspiration. To prevent aspiration, all intubated patients and patients receiving enteral feedings must have the head of the bed elevated a minimum of 30 to 45 degrees unless medically contraindicated. Closed-suction technique is preferred over the open-suction technique because oxygenation and ventilation are maintained during suctioning, and exposure to secretions is reduced. The nurse should provide comfort measures such as morphine to relieve anxiety and pain associated with intubation. To promote rest and sleep, the nurse should limit noise and cluster activities.

A patient presents to the emergency department after being stung by a bee, complaining of difficulty breathing. What vasoconstrictive medication should be given at this time?

Epinephrine

The nurse is assessing an acutely ill patient. When prioritizing the patient's care, the nurse should recognize that the patient is at risk for hypovolemic shock when:

Fluid circulating in the blood vessels decreases. pg.

Elevating the patient's legs slightly to improve cerebral circulation is contraindicated in which of the following disease processes?

Head injury pg. 297

You are holding a class on shock for the staff nurses at your institution. What would you tell them about the stages of shock?

In the compensation stage, catecholamines are released. pg. 287

The nurse is administering a medication to the client with a positive inotropic effect. Which action of the medication does the nurse anticipate?

Increase the force of myocardial contraction pg. 299

You are caring for a client in the compensation stage of shock. You know that in this stage of shock adrenaline and noradrenaline are released into the circulation. What positive effect does this have on your client?

Increases myocardial contractility

The nurse is caring for a patient intubated and on a mechanical ventilator for several days. Which weaning parameter would tell the nurse if the patient has enough muscle strength to breathe without assistance? Tidal volume Minute ventilation Forced vital capacity Negative inspiratory force

Negative inspiratory force The negative inspiratory force measures inspiratory muscle strength. Tidal volume and minute ventilation assess the patient's respiratory endurance. Forced vital capacity is not used as a measure to determine weaning from a ventilator

Following a motor vehicle collision, a client is admitted to the emergency department with a blood pressure of 88/46, pulse of 54 beats/min with a regular rhythm, and respirations of 20 breaths/min with clear lung sounds. The client's skin is dry and warm. The nurse assesses the client to be in which type of shock?

Neurogenic

A 17-year-old-male client with a history of depression is brought to the ED after overdosing on Valium. This client is at risk for developing which type of distributive shock?

Neurogenic Shock

A client is in a driving accident creating a spinal cord injury. The nurse caring for a client realizes that the client is at risk for which type of shock?

Neurogenic pg. 301

The client exhibits a blood pressure of 110/68 mm Hg, pulse rate of 112 beats/min, temperature of 102°F with skin warm and flushed. Respirations are 30 breaths/min. The nurse assesses the client may be exhibiting the early stage of which shock?

Septic

You are assessing a 6-year-old little girl in the emergency department (ED) who was brought in by her mother. She was stung by a bee and is allergic to bee venom. The child is now having trouble breathing. She is vasodilated, hypotensive, and has broken out in hives. What do you suspect is wrong with this child?

She is having an allergic reaction and going into anaphylactic shock.

The nurse is obtaining physician orders which include a pulse pressure. The nurse is most correct to report which of the following?

The difference between the systolic and diastolic pressure pg. 288

1 Patients requiring ventilation up to 3 days are said to have received short-term ventilation. More than 3 days (7, 14, and 20 days) denotes long-term ventilation. Text Reference - p. 1625

The health care provider advises short-term ventilation for a patient. How many days would be considered short-term ventilation? 1 Up to 3 days 2 Up to 7 days 3 Up to 14 days 4 Up to 20 days

4 Progressive care units (PCUs), also called intermediate care units, provide a transition between the intensive care unit (ICU) and the general care unit or discharge. Generally, PCU patients are at risk for serious complications, but their risk is lower than that of the ICU patient. Therefore, the nurse anticipates that the patient with a stable blood pressure on a stable dose of a vasoactive drug will be transferred to the PCU. A patient who required a new drip overnight for hypotension and a patient who crashed and required intubation and mechanical ventilation are unstable and the nurse does not anticipate their transfer to the PCU. A preoperative heart transplant patient is often admitted to the PCU. A patient who is 12 hours post transplantation is not expected to be transferred to the PCU at this time. Text Reference - p. 1599

The intensive care unit (ICU) charge nurse is reviewing patient medical records during the overnight shift. Which patient does the nurse anticipate will be transferred to the progressive care unit (PCU)? 1 The patient who required a new drip overnight for hypotension. 2 The patient who was intubated and ventilated after crashing overnight. 3 The patient who is 12 hours postop for a heart and lung transplantation. 4 The patient whose blood pressure is 100/70 mm Hg on a stable dose of a vasoactive drug.

2 Music therapy is a nonpharmacologic relaxation intervention that can be used to treat the patient anxiety that often occurs in the intensive care unit (ICU) environment. Lorazepam is an appropriate pharmacologic, not nonpharmacologic, intervention for anxiety. A sedation holiday is appropriate to conduct a neurologic exam that is often difficult due to the deep sedation required for intubation and mechanical ventilation. Range-of-motion exercises can help treat the physical manifestations of an injury, but are not relaxation interventions. Text Reference - p. 1601

The nurse is providing care to a patient in the intensive care unit (ICU) who is experiencing anxiety. Which nonpharmacologic relaxation intervention is appropriate for this patient? 1 Lorazepam 2 Music therapy 3 Sedation holiday 4 Range-of-motion exercises

2 Frank-Starling's law explains the effects of preload and states that the more a myocardial fiber is stretched during filling, the more it shortens during systole and the greater the force of the contraction. Cardiac index (CI) is the measurement of the cardiac output adjusted for body surface area (BSA). It is a more precise measurement of the efficiency of the heart's pumping action. Systemic vascular resistance (SVR) is opposition encountered by the left ventricle to blood flow by the vessels. Pulmonary vascular resistance (PVR) is opposition encountered by the right ventricle to blood flow by the vessels. Text Reference - p. 1604

The nurse educator is preparing a lecture on hemodynamic monitoring. What should the educator use to explain the effects of preload? 1 Cardiac index 2 Frank-Starling's law 3 Systemic vascular resistance 4 Pulmonary vascular resistance

1 Critical care nursing is a specialty dealing with human responses to life-threatening problems. Medial surgical nursing is a specialty dealing with the care of adult patients in a variety of settings. Mental health nursing is a specialty dealing with people of all ages with mental illness or mental distress. Maternal newborn nursing is a specialty dealing with the care of women throughout their pregnancy and childbirth. Text Reference - p. 1598

The nurse educator is teaching a group of nursing students about critical care nursing. Which statement by a student indicates appropriate understanding of this nursing specialty? 1 "Critical care nursing is a specialty dealing with human responses to life-threatening problems." 2 "Critical care nursing is a specialty dealing with the care of adult patients in a variety of settings." 3 "Critical care nursing is a specialty dealing with people of all ages with mental illness or mental distress." 4 "Critical care nursing is a specialty dealing with the care of women throughout their pregnancy and childbirth."

1 Critical care nurses will face ethical dilemmas related to the care of patients. This can occur over perceived issues of delivering futile or nonbeneficial care, such as attempting to resuscitate a neonate born at 21 weeks of gestation. This situation does not represent a societal, temporal, or sequential dilemma. Text Reference - p. 1599

The nurse in the newborn intensive care unit (NICU) is providing care to a neonate born at 21 weeks gestation who is being resuscitated in the delivery room. The standard gestation compatible with life is 23 to 24 weeks gestation. Which type of dilemma could the nurse be facing? 1 Ethical 2 Societal 3 Temporal 4 Sequential

2 Inadequate pain control is often linked with agitation and anxiety and is known to add to the stress response. Pain is associated with infection, but the patient is at the highest risk for developing anxiety. Dementia and depression are not associated with unrelieved pain. Text Reference - p. 1600

The nurse is assessing a patient in the intensive care unit (ICU). The patient is intubated and exhibiting symptoms of pain. Based on this data what condition is the patient at the highest risk for developing? 1 Anxiety 2 Infection 3 Dementia 4 Depression

2 Patients with preexisting dementia, such as Alzheimer's disease, are at an increased risk for developing delirium when receiving care in the intensive care unit (ICU). Diabetes mellitus, Parkinson's disease, and multiple sclerosis are not known risk factors for developing delirium. Text Reference - p. 1601

The nurse is providing care to a patient in the intensive care unit (ICU). The patient is currently sedated due to intubation and mechanical ventilation. Which finding in the patient's medical record would place this patient at an increased risk for delirium? 1 Diabetes mellitus 2 Alzheimer's disease 3 Parkinson's disease 4 Multiple sclerosi

4 Intraaortic balloon therapy has potential complications such as site infection, thromboembolism, arterial trauma, hematologic complications, and hemorrhage from the insertion site. Maintaining the head of the bed below 45 degrees helps to prevent breathlessness in the patient in the event of arterial trauma. Monitoring coagulation profiles, hematocrit, and platelet count is beneficial when the patient has a hematologic complication. An occlusive dressing prevents risk of surface infection but is not used to treat arterial trauma in the patient. Heparin helps to prevent thromboembolism but does not treat arterial trauma. Text Reference - p. 1612

The nurse is caring for a patient who has suffered arterial trauma during intraaortic balloon pump (IABP) therapy. What is the appropriate nursing action for this patient? 1 Monitor coagulation profiles 2 Apply an occlusive dressing 3 Administer prophylactic heparin 4 Maintain head of the bed below 45 degrees

1 Accurate oxygen saturation of hemoglobin (SpO2) measurements may be difficult to obtain on patients who are hypothermic, are receiving vasopressor therapy, or experiencing shock. A body temperature of 95.4° F is hypothermic and is most likely the reason this measurement is difficult to obtain. A cardiac glycoside is not a vasopressor and would not cause this difficulty. A blood pressure of 118/72 mm Hg and heart rate of 72 with occasional ectopy are not manifestations of shock and would not cause this difficulty. Text Reference - p. 1609

The nurse is having difficulty obtaining an accurate oxygen saturation of hemoglobin (SpO2) measurement on a patient. What should the nurse consider as the reason for this difficulty? 1 Body temperature 95.4° F 2 Receiving a cardiac glycoside 3 Blood pressure of 118/72 mm Hg 4 Heart rate of 72 beats/minute with occasional ectopy

2 Normal central venous oxygen saturation or ScvO2/SvO2 is 60% to 80%. It denotes normal oxygen supply and metabolic demand. Any reading out of the normal range can cause danger to the patient. More than 80% denotes increased oxygen supply and decreased oxygen demand. Less than 60% denotes decreased oxygen supply and increased demand. Text Reference - p. 1609

The nurse is monitoring a patient in ICU. Which ScvO2/SvO2reading is a cause of concern in the patient? 1. 68% 2. 54% 3. 72% 4. 78%

10 The formula to calculate the minute ventilation (VE) is: VE = VTf. Therefore, VE = (20) (0.5) = 10 L/min. Text Reference - p. 1626

The nurse is performing a weaning assessment on a patient receiving mechanical ventilation. What is the value of minute ventilation (VE) if the patient's spontaneous respiratory rate (f) is 20 breaths/minute and spontaneous tidal volume (VT) is 0.5 L? Record your answer using a whole number. ___________________L/min

3 Stroke volume variation (SVV) is the variation of the arterial pulsation caused by heart-lung interaction. It is a sensitive indicator of preload responsiveness when used on select patients. SVV is used only for patients on controlled mechanical ventilation with a fixed respiratory rate and a fixed tidal volume of 8 mL/kg. SVV is not used on patients who have spontaneous respirations even though an arterial line does need to be in place. The patient may have continuous cardiac monitoring but the patient needs to be intubated rather than provided with oxygen via a face mask. The patient needs to be intubated; however, nasal intubation is not identified as a requirement. It is not identified that the patient needs to be receiving positive end expiration pressure through the ventilator. Text Reference - p. 1607

The nurse is preparing a patient for arterial pressure-based cardiac output (APCO) measuring. What patient criteria must be met before this measuring device can be used to determine the patient's stroke volume variation (SVV)? 1 Spontaneous respirations and placement of an arterial line 2 Continuous cardiac monitoring and application of oxygen via face mask 3 Controlled mechanical ventilation and fixed respiratory rate and tidal volume 4 Nasal intubation and positive end expiration pressure setting on the ventilator

4 When an intensive care unit (ICU) patient exhibits manifestations of delirium it is the nurse's priority to address physiologic factors that could be contributing to the patient's symptoms. Correction of oxygenation by increasing the patient's oxygen is the priority intervention for this patient. Placing a clock in the room and updating the calendar in the room are important when providing care to a patient with delirium but these are not the priorities in the current situation. Administering an opioid pain medication is likely to enhance the clinical manifestations of delirium. Text Reference - p. 1601

The nurse is providing care for a patient who is receiving care in the intensive care unit (ICU). The patient is exhibiting symptoms of delirium. Which will the nurse address on priority when providing care to this patient? 1 Placing a clock in the room 2 Updating the calendar in the room 3 Administering opioid analgesics for pain 4 Administering increased oxygen, per order

2 Tachypnea and disorientation are early and subtle signs of deterioration. The rapid response team (RRT) brings rapid and immediate care to unstable patients in non-critical care units. While assessing pain, documenting the data in the medical record, and administering prescribed medications such as antihypertensive medications are all appropriate actions, they are not the priority nursing actions in this situation. Text Reference - p. 1599

The nurse is providing care to a postoperative patient on a medical-surgical unit. The patient is experiencing tachypnea and becomes disoriented at times. Which is the priority action by the nurse in this situation? 1 Assessing the current level of pain 2 Activating the rapid response team 3 Documenting the data in the medical record 4 Administering the prescribed antihypertensive medication

1 Nonverbal communication is important when providing care to an unresponsive patient in the intensive care unit (ICU). High levels of procedure-related touch and lower levels of comfort-related touch often characterize the ICU environment. The nurse would encourage the patient's spouse to touch and talk with her husband even if he is unresponsive. There is no way of knowing how aware the patient is of the surrounding when sedated. While bringing items from home for comfort are important, this is not the most appropriate statement by the nurse. Telling the spouse to sit where she will not be in the way of providing care is not therapeutic. Text Reference - p. 1601

The nurse is providing care to an unresponsive patient in the intensive care unit (ICU). The patient's spouse is at the bedside and states, "I just want him to know that I am here with him." Which statement by the nurse is most appropriate? 1 "You should talk and touch you husband whenever you visit." 2 "Your husband is so sedated he is not aware of his surroundings." 3 "You can bring items from home to make your husband more comfortable." 4 "Please sit where you will not be in the way as we provide care to your husband."

4 The patient with a collapsed lung and multiple injuries sustained in a car accident would be admitted to the trauma intensive care unit (ICU). Patients with minor acute injuries or those who are not expected to recover from an illness is usually not admitted to an ICU. Therefore, the ICU is not used to manage a fractured wrist; to prolong the natural process of death such as the patient with stage IV metastatic breast cancer; or for the patient in a persistent coma or vegetative state. Text Reference - p. 1600

The nurse is providing care to several patients in the emergency department (ED). Which patient does the nurse anticipate will be admitted to the intensive care unit (ICU)? 1 A patient with a fractured wrist 2 A patient with stage IV metastatic breast cancer 3 A patient in a permanent vegetative state who has a urinary tract infection 4 The patient with a collapsed lung and multiple injuries sustained in a car accident

4 The nurse should advise the caregiver to avoid external rotation of the patient's hip; this movement can be avoided by properly positioning the patient and by the use of specialized mattresses and beds. Simple maneuvers such as arm circles, knee bends and quadriceps setting should be performed, because they maintain the muscle tone in the upper and lower extremities of the patient. Text Reference - p. 1624

The nurse is teaching the patient's caregiver about receiving positive pressure ventilation. What movements should the nurse tell the caregiver to avoid doing to the patient? 1 Arm circles 2 Knee bends 3 Quadriceps setting 4 External rotation of the hip

2 Tidal volume is the volume of gas delivered to a patient during each ventilator breath. The number of breaths the ventilator delivers per minute is called the respiratory rate. The positive pressure used to augment the patient's inspiratory pressure is called pressure support. The positive pressure applied at the end of expiration of ventilator breaths is called positive end-expiratory pressure. Text Reference - p. 1619

The nurse working in a critical care unit understands that tidal volume is an important setting in a mechanical ventilator. Which statement appropriately describes tidal volume? 1 Number of breaths the ventilator delivers per minute 2 Volume of gas delivered to patient during each ventilator breath 3 Positive pressure used to augment patient's inspiratory pressure 4 Positive pressure applied at the end of expiration of ventilator breaths

A nurse is weaning a 68-kg patient who has chronic obstructive pulmonary disease (COPD) from mechanical ventilation. Which patient assessment finding indicates that the weaning protocol should be stopped? a. The patient's heart rate is 97 beats/min. b. The patient's oxygen saturation is 93%. c. The patient respiratory rate is 32 breaths/min. d. The patient's spontaneous tidal volume is 450 mL.

The patient respiratory rate is 32 breaths/min. Tachypnea is a sign that the patient's work of breathing is too high to allow weaning to proceed. The patient's heart rate is within normal limits, but the nurse should continue to monitor it. An O2 saturation of 93% is acceptable for a patient with COPD. A spontaneous tidal volume of 450 mL is within the acceptable range.

A client is receiving support through an intra-aortic balloon counterpulsation. The catheter for the balloon is inserted in the right femoral artery. The nurse evaluates the following as a complication of the therapy:

The right foot is cooler than the left foot. pg. 300

4 Pulmonary vascular resistance (PVR) is calculated using the pulmonary artery mean pressure (PAMR) minus the pulmonary artery wedge pressure (PAWP), multiplying by 80 and dividing by the cardiac output (CO). Systolic and diastolic blood pressures are used to determine mean arterial pressure (MAP). Stroke volume (SV) and right ventricular ejection fraction (RVEF) are used to determine right ventricular end-diastolic volume (RVEDV). Mean arterial pressure (MAP), central venous pressure (CVP), and cardiac output (CO) are used to determine systemic vascular resistance (SVR). Text Reference - p. 1603

To determine a patient's peripheral vascular resistance (PVR) what hemodynamic parameters should the nurse use? 1 Systolic and diastolic blood pressures 2 Stroke volume and right ventricular ejection fraction 3 Mean arterial pressure, central venous pressure, and cardiac output 4 Pulmonary artery mean pressure, pulmonary artery wedge pressure, and cardiac output

3 To obtain an accurate central venous pressure (CVP) reading, the nurse should ensure that the patient is supine and in horizontal position so the zero reference point at the level of the right atrium (also known as the phlebostatic axis) is level with the zero mark on the manometer. If a patient is unable to lie flat, the manometer should be positioned so the zero reference is at the level of the right atrium and the degree of head elevation noted so there is consistency across measurements. Right or left side-lying and supine positions with the head of the bed elevated 45 degrees are all incorrect for obtaining a CVP reading. Text Reference - p. 1605

To obtain an accurate central venous pressure reading with a central venous catheter, a nurse should place the patient in what position? 1 Left side-lying 2 Right side-lying 3 Supine and horizontal 4 Supine with head of bed elevated 45 degrees

When planning the care of the patient in cardiogenic shock, what does the nurse understand is the primary treatment goal?

Treat the oxygenation needs of the heart muscle

3 Negative inspiratory force is used to assess the muscle strength in a patient with positive pressure ventilation. Vital capacity, minute ventilation and rapid shallow breathing index are used to assess the muscle endurance. Test-Taking Tip: Multiple-choice questions can be challenging because students think that they will recognize the right answer when they see it or that the right answer will somehow stand out from the other choices. This is a dangerous misconception. The more carefully the question is constructed, the more each of the choices will seem like the correct response. Text Reference - p. 1626

What measurement is used to assess muscle strength in a patient with positive pressure ventilation? 1 Vital capacity 2 Minute ventilation 3 Negative inspiratory force 4 Rapid shallow breathing index

75 Rapid shallow breathing index is equal to respiratory rate/tidal volume; therefore, 30/0.4 = 75. Text Reference - p. 1626

What will be the rapid shallow breathing index for a patient with a tidal volume of 0.4 L and respiratory rate of 30/minute? Record your answer using a whole number. ___ L

2 While caring for a patient requiring mechanical ventilation, the registered nurses (RNs) must administer sedatives by themselves; they should not delegate this task to unlicensed assistive personnel (UAP). UAP can be tasked with obtaining vital signs, measuring urine output, and performing bedside glucose tests. Text Reference - p. 1625

When caring for a critically ill patient on mechanical ventilation, what task must the registered nurses (RNs) perform by themselves and not delegate to unlicensed assistive personnel (UAP)? 1 Obtaining vital signs 2 Administering sedatives 3 Measuring urine output 4 Performing bedside glucose test

1 Prior to the family entering the intensive care unit (ICU) to visit a family member who is critically ill the nurse should provide the family with a description of what to expect regarding the patient's appearance and the equipment that is being used to provide care to their family member. The nurse should accompany the family into the patient's room; it is not appropriate for the nurse to ask the physician to do this, nor is it appropriate for the family to enter the room alone. Although it is important to instruct the family on what to expect, the family should be encouraged to touch and speak to the patient. Text Reference - p. 1602

Which action by the nurse is most appropriate when bringing a family member of a critically ill patient into the intensive care unit (ICU) for the first time? 1 Give a description of what to expect 2 Ask the physician to accompany the family 3 Allow the family member to enter the room alone 4 Instruct the family not to touch and speak to the patient

Patients with acute respiratory failure will have drug therapy to meet their individual needs. Which drugs will meet the goal of reducing pulmonary congestion (select all that apply)? a. Morphine d. Albuterol (Ventolin) b. Furosemide (Lasix) e. Ceftriaxone (Rocephin) c. Nitroglycerin (Tridil) f. Methylprednisolone (Solu-Medrol)

a, b, c. Morphine and nitroglycerin (e.g., Tridil) will decrease pulmonary congestion caused by heart failure; IV diuretics (e.g., furosemide [Lasix]) are also used. Inhaled albuterol (Ventolin) or metaproterenol (Alupent) will relieve bronchospasms. Ceftriaxone (Rocephin) and azithromycin (Zithromax) are used to treat pulmonary infections. Methylprednisolone (Solu-Medrol), an IV corticosteroid, will reduce airway inflammation. Morphine is also used to decrease anxiety, agitation, and pain.

Which changes of aging contribute to the increased risk for respiratory failure in older adults (select all that apply)? a. Alveolar dilation d. Increased infection risk b. Increased delirium e. Decreased respiratory muscle strength c. Changes in vital signs f. Diminished elastic recoil within the airways

a, d, e, f. Changes from aging that increase the older adult's risk for respiratory failure include alveolar dilation, increased risk for infection, decreased respiratory muscle strength, and diminished elastic recoil in the airways. Although delirium can complicate ventilator management, it does not increase the older patient's risk for respiratory failure. The older adult's blood pressure (BP) and heart rate (HR) increase but this does not affect the risk for respiratory failure. The ventilatory capacity is decreased and the larger air spaces decrease the surface area for gas exchange, which increases the risk.

The patient progressed from acute lung injury to acute respiratory distress syndrome (ARDS). He is on the ventilator and receiving propofol (Diprivan) for sedation and fentanyl (Sublimaze) to decrease anxiety, agitation, and pain in order to decrease his work of breathing, O2 consumption, carbon dioxide production, and risk of injury. What intervention is recommended in caring for this patient? a. A sedation holiday c. Keeping his legs still to avoid dislodging the airway b. Monitoring for hypermetabolism d. Repositioning him every 4 hours to decrease agitation

a. A sedation holiday is needed to assess the patient's condition and readiness to extubate. A hypermetabolic state occurs with critical illness. With malnourished patients, enteral or parenteral nutrition is started within 24 hours; with well-nourished patients it is started within 3 days. With these medications, the patient will be assessed for cardiopulmonary depression. Venous thromboembolism prophylaxis will be used but there is no reason to keep the legs still. Repositioning the patient every 2 hours may help to decrease discomfort and agitation

Although ARDS may result from direct lung injury or indirect lung injury as a result of systemic inflammatory response syndrome (SIRS), the nurse is aware that ARDS is most likely to occur in the patient with a host insult resulting from a. sepsis. c. prolonged hypotension. b. oxygen toxicity. d. cardiopulmonary bypass.

a. Although ARDS may occur in the patient who has virtually any severe illness and may be both a cause and a result of systemic inflammatory response syndrome (SIRS), the most common precipitating insults of ARDS are sepsis, gastric aspiration, and severe massive trauma.

The best patient response to treatment of ARDS occurs when initial management includes what? a. Treatment of the underlying condition c. Treatment with diuretics and mild fluid restriction b. Administration of prophylactic antibiotics d. Endotracheal intubation and mechanical ventilation

a. Because ARDS is precipitated by a physiologic insult, a critical factor in its prevention and early management is treatment of the underlying condition. Prophylactic antibiotics, treatment with diuretics and fluid restriction, and mechanical ventilation are also used as ARDS progresses.

Which assessment finding should cause the nurse to suspect the early onset of hypoxemia? a. Restlessness c. Central cyanosis b. Hypotension d. Cardiac dysrhythmias

a. Because the brain is very sensitive to a decrease in oxygen delivery, restlessness, agitation, disorientation, and confusion are early signs of hypoxemia, for which the nurse should be alert. Mild hypertension is also an early sign, accompanied by tachycardia. Central cyanosis is an unreliable, late sign of hypoxemia. Cardiac dysrhythmias also occur later

Priority Decision: After endotracheal intubation and mechanical ventilation have been started, a patient in respiratory failure becomes very agitated and is breathing asynchronously with the ventilator. What is it most important for the nurse to do first? a. Evaluate the patient's pain level, ABGs, and electrolyte values b. Sedate the patient to unconsciousness to eliminate patient awareness c. Administer the PRN vecuronium (Norcuron) to promote synchronous ventilations d. Slow the rate of ventilations provided by the ventilator to allow for spontaneous breathing by the patient

a. It is most important to assess the patient for the cause of the restlessness and agitation (e.g., pain, hypoxemia, electrolyte imbalances) and treat the underlying cause before sedating the patient. Although sedation, analgesia, and neuromuscular blockade are often used to control agitation and pain, these treatments may contribute to prolonged ventilator support and hospital days.

A patient who is receiving positive pressure ventilation is scheduled for a spontaneous breathing trial (SBT). Which finding by the nurse is most likely to result in postponing the SBT? a. New ST segment elevation is noted on the cardiac monitor. b. Enteral feedings are being given through an orogastric tube. c. Scattered rhonchi are heard when auscultating breath sounds. d. hydromorphone (Dilaudid) is being used to treat postoperative pain.

a. New ST segment elevation is noted on the cardiac monitor. Myocardial ischemia is a contraindication for ventilator weaning. The ST segment elevation is an indication that weaning should be postponed until further investigation and/or treatment for myocardial ischemia can be done. Ventilator weaning can proceed when opioids are used for pain management, abnormal lung sounds are present, or enteral feedings are being used.

When mechanical ventilation is used for the patient with ARDS, what is the rationale for applying positive end- expiratory pressure (PEEP)? a. Prevent alveolar collapse and open up collapsed alveoli b. Permit smaller tidal volumes with permissive hypercapnia c. Promote complete emptying of the lungs during exhalation d. Permit extracorporeal oxygenation and carbon dioxide removal outside the body

a. Positive end-expiratory pressure (PEEP) used with mechanical ventilation applies positive pressure to the airway and lungs at the end of exhalation, keeping the lung partially expanded and preventing collapse of the alveoli and helping to open up collapsed alveoli. Permissive hypercapnia is allowed when the patient with ARDS is ventilated with smaller tidal volumes to prevent barotrauma. Extracorporeal membrane oxygenation and extracorporeal CO2 removal involve passing blood across a gas-exchanging membrane outside the body and then returning oxygenated blood to the body.

40. A patient is to be discharged home with mechanical ventilation. Before discharge, it is most important for the nurse to a. teach the caregiver to care for the patient with a home ventilator. b. help the caregiver plan for placement of the patient in a long-term care facility. c. stress the advantages for the patient in being cared for in the home environment. d. have the caregiver arrange for around-the-clock home health nurses for the first several weeks.

a. Rationale: Care of a ventilator-dependent patient in the home requires that the caregiver know how to manage the ventilator and take care of the patient on it. The nurse should ensure that caregivers understand the potential sacrifices they may have to make and the impact that home mechanical ventilation will have over time, before final decisions and arrangements are made. Placement in long- term care facilities is not usually necessary unless the caregiver can no longer manage the care or the patient's condition deteriorates.

In preparing a patient in the ICU for oral endotracheal intubation, the nurse a. places the patient supine with the head extended and neck flexed. b. tells the patient that the tongue must be extruded while the tube is inserted. c. positions the patient supine with the head hanging over the edge of the bed to align the mouth and trachea. d. informs the patient that while it will not be possible to talk during insertion of the tube, speech will be possible after it is correctly placed.

a. Rationale: The patient is positioned with the mouth, pharynx, and trachea in direct alignment, with the head extended in the "sniffing position," but the head must not hang over the edge of the bed. The patient may be asked to extrude the tongue during nasal intubation. Speaking is not possible during intubation or while the tube is in place because the tube splits the vocal cords.

A patient who is orally intubated and receiving mechanical ventilation is anxious and is "fighting" the ventilator. Which action should the nurse take next? a. Verbally coach the patient to breathe with the ventilator. b. Sedate the patient with the ordered PRN lorazepam (Ativan). c. Manually ventilate the patient with a bag-valve-mask device. d. Increase the rate for the ordered propofol (Diprivan) infusion.

a. Verbally coach the patient to breathe with the ventilator. The initial response by the nurse should be to try to decrease the patient's anxiety by coaching the patient about how to coordinate respirations with the ventilator. The other actions may also be helpful if the verbal coaching is ineffective in reducing the patient's anxiety.

A patient with a massive hemothorax and pneumothorax has absent breath sounds in the right lung. To promote improved V/Q matching, how should the nurse position the patient? a. On the left side c. In a reclining chair bed b. On the right side d. Supine with the head of the bed elevated

a. When there is impaired function of one lung, the patient should be positioned with the unaffected lung in the dependent position to promote perfusion to the functioning tissue. If the diseased lung is positioned dependently, more V/Q mismatch would occur. The head of the bed may be elevated or a reclining chair may be used, with the patient positioned on the unaffected side, to maximize thoracic expansion if the patient has increased work of breathing.

14. A patient admitted with acute respiratory failure has a nursing diagnosis of ineffective airway clearance related to thick, secretions. Which action is a priority for the nurse to include in the plan of care? * a. Encourage use of the incentive spirometer. b. Offer the patient fluids at frequent intervals. c. Teach the patient the importance of ambulation. d. Titrate oxygen level to keep O2 saturation >93%.

b

24. The nurse is caring for a patient who is intubated and receiving positive pressure ventilation to treat acute respiratory distress syndrome (ARDS). Which finding is most important to report to the health care provider? * a. Blood urea nitrogen (BUN) level 32 mg/dL b. Red-brown drainage from orogastric tube c. Scattered coarse crackles heard throughout lungs d. Arterial blood gases: pH 7.31, PaCO2 50, PaO2 68

b

A patient with an oral ET tube has a nursing diagnosis of risk for aspiration related to presence of artificial airway. Appropriate nursing interventions for this patient are to (select all that apply) a. assess gag reflex. b. ensure the cuff is properly inflated. c. suction the patient's mouth frequently. d. raise the head of the bed 30 to 45 degrees unless the patient is unstable. e. keep the ventilator tubing cleared of condensed water.

b, c, d. Rationale: Because the patient with an ET tube cannot protect the airway from aspiration and cannot swallow, the cuff should always be inflated and the head of the bed (HOB) elevated while the patient is receiving tube feedings or mouth care is being performed. The HOB elevated 30 to 45 degrees helps reduce risk. The mouth and oropharynx should be suctioned with Yankauer or tonsil suction to remove accumulated secretions that cannot be swallowed. Clearing the ventilatory tubing of condensed water is important to prevent respiratory infection.

Which descriptions are characteristic of hypoxemic respiratory failure (select all that apply)? a. Referred to as ventilatory failure b. Primary problem is inadequate O2 transfer c. Risk of inadequate O2 saturation of hemoglobin exists d. Body is unable to compensate for acidemia of increased PaCO2 e. Most often caused by ventilation-perfusion (V/Q) mismatch and shunt f. Exists when PaO2 is 60 mm Hg or less, even when O2 is administered at 60%

b, c, e, f. Hypoxemic respiratory failure is often caused by ventilation-perfusion (V/Q) mismatch and shunt. It is called oxygenation failure because the primary problem is inadequate oxygen transfer. There is a risk of inadequate oxygen saturation of hemoglobin and it exists when PaO2 is 60 mm Hg or less, even when oxygen is administered at 60%. Ventilatory failure is hypercapnic respiratory failure. Hypercapnic respiratory failure results from an imbalance between ventilatory supply and ventilatory demand and the body is unable to compensate for the acidemia of increased PaCO2

A patient in hypercapnic respiratory failure has a nursing diagnosis of ineffective airway clearance related to increasing exhaustion. What is an appropriate nursing intervention for this patient? a. Inserting an oral airway c. Teaching the patient huff coughing b. Performing augmented coughing d. Teaching the patient slow pursed lip breathing

b. Augmented coughing is done by applying pressure on the abdominal muscles at the beginning of expiration. This type of coughing helps to increase abdominal pressure and expiratory flow to assist the cough to remove secretions in the patient who is exhausted. An oral airway is used only if there is a possibility that the tongue will obstruct the airway. Huff coughing prevents the glottis from closing during the cough and works well for patients with chronic obstructive pulmonary disease (COPD) to clear central airways. Slow pursed lip breathing allows more time for expiration and prevents small bronchioles from collapsing.

Which patient with the following manifestations is most likely to develop hypercapnic respiratory failure? a. Rapid, deep respirations in response to pneumonia b. Slow, shallow respirations as a result of sedative overdose c. Large airway resistance as a result of severe bronchospasm d. Poorly ventilated areas of the lung caused by pulmonary edema

b. Hypercapnic respiratory failure is associated with alveolar hypoventilation with increases in alveolar and arterial carbon dioxide (CO2 ) and often is caused by problems outside the lungs. A patient with slow, shallow respirations is not exchanging enough gas volume to eliminate CO2 . Deep, rapid respirations reflect hyperventilation and often accompany lung problems that cause hypoxemic respiratory failure. Pulmonary edema and large airway resistance cause obstruction of oxygenation and result in a V/Q mismatch or shunt typical of hypoxemic respiratory failure.

When weaning a patient from a ventilator, the nurse plans a. to decrease the delivered FIO2 concentration. b. intermittent trials of spontaneous ventilation followed by ventilatory support to provide rest. c. substitution of ventilator support with a manual resuscitation bag if the patient becomes hypoxemic. d. to implement weaning procedures around the clock until the patient does not experience ventilatory fatigue.

b. Rationale: A variety of ventilator weaning methods is used, but all should provide weaning trials with adequate rest between weaning trials to prevent respiratory muscle fatigue. Weaning is usually carried out during the day, with the patient ventilated at night until there is sufficient spontaneous ventilation without excess fatigue. In all methods, patients usually require a 10% increase in fraction of inspired oxygen (FiO2) to maintain arterial oxygen tension. If the patient becomes hypoxemic, ventilator support is indicated.

Before taking hemodynamic measurements, the monitoring equipment must be referenced by a. confirming that when pressure in the system is zero, the equipment reads zero. b. positioning the stopcock nearest the transducer level with the phlebostatic axis. c. placing the transducer on the left side of the chest at the fourth intercostal space. d. placing the patient in a left lateral position with the transducer level with the top surface of the mattress.

b. Rationale: Referencing hemodynamic monitoring equipment means positioning the monitoring equipment so that the zero reference point is at the vertical level of the left atrium of the heart. The port of the stopcock nearest the transducer is placed at the phlebostatic axis, the external landmark of the left atrium. The phlebostatic axis is the intersection of two planes: a horizontal line midchest, halfway between the outermost anterior and posterior surfaces, transecting a vertical line through the fourth intercostal space at the sternum.

A nursing intervention indicated for the patient in the ICU who has a nursing diagnosis of anxiety related to ICU environment and sensory overload is a. providing flexible visiting schedules for caregivers. b. eliminating unnecessary alarms and overhead paging. c. administering sedatives or psychotropic drugs to promote rest. d. allowing the patient to do as many self-care activities as possible.

b. Rationale: When anxiety in the ICU patient is related to the environment, which has unfamiliar equipment, high noise and light levels, and an intense pace of activity, which leads to sensory overload, the nurse should eliminate as much of this source of stress as possible by muting phones, limiting overhead paging, setting alarms appropriate to the patient's condition, and eliminating unnecessary alarms. Offering flexible visiting schedules for family members and providing as much autonomy in decisions about care as possible are indicated when impaired communication and loss of control contribute to the anxiety. Use of sedation to reduce anxiety should be carefully evaluated and implemented when nursing measures are not effective.

The nurse educator is evaluating the performance of a new registered nurse (RN) who is providing care to a patient who is receiving mechanical ventilation with 15 cm H2O of peak end-expiratory pressure (PEEP). Which action indicates that the new RN is safe? a. The RN plans to suction the patient every 1 to 2 hours. b. The RN uses a closed-suction technique to suction the patient. c. The RN tapes the connection between the ventilator tubing and the ET. d. The RN changes the ventilator circuit tubing routinely every 48 hours.

b. The RN uses a closed-suction technique to suction the patient. The closed-suction technique is used when patients require high levels of PEEP (>10 cm H2O) to prevent the loss of PEEP that occurs when disconnecting the patient from the ventilator. Suctioning should not be scheduled routinely, but it should be done only when patient assessment data indicate the need for suctioning. Taping connections between the ET and ventilator tubing would restrict the ability of the tubing to swivel in response to patient repositioning. Ventilator tubing changes increase the risk for ventilator-associated pneumonia and are not indicated routinely.

25. During change-of-shift report on a medical unit, the nurse learns that a patient with aspiration pneumonia who was admitted with respiratory distress has become increasingly agitated. Which action should the nurse take first? * a. Give the prescribed PRN sedative drug. b. Offer reassurance and reorient the patient. c. Use pulse oximetry to check the oxygen saturation. d. Notify the health care provider about the patient's status.

c

The patient is being admitted to the intensive care unit (ICU) with hypercapnic respiratory failure. Which manifestations should the nurse expect to assess in the patient (select all that apply)? a. Cyanosis d. Respiratory acidosis b. Metabolic acidosis e. Use of tripod position c. Morning headache f. Rapid, shallow respirations

c, d, e, f. Morning headache, respiratory acidosis, the use of tripod position, and rapid, shallow respirations would be expected. The other manifestations are characteristic of hypoxemic respiratory failure.

The patient with a history of heart failure and acute respiratory failure has thick secretions that she is having difficulty coughing up. Which intervention would best help to mobilize her secretions? a. Administer more IV fluid c. Provide O2 by aerosol mask b. Perform postural drainage d. Suction airways nasopharyngeally

c. For the patient with a history of heart failure, current acute respiratory failure, and thick secretions, the best intervention is to liquefy the secretions with either aerosol mask or using normal saline administered by a nebulizer. Excess IV fluid may cause cardiovascular distress and the patient probably would not tolerate postural drainage with her history. Suctioning thick secretions without thinning them is difficult and increases the patient's difficulty in maintaining oxygenation. With copious secretions, this could be done after thinning the secretions.

In patients with ARDS who survive the acute phase of lung injury, what manifestations are seen when they progress to the fibrotic phase? a. Chronic pulmonary edema and atelectasis b. Resolution of edema and healing of lung tissue c. Continued hypoxemia because of diffusion limitation d. Increased lung compliance caused by the breakdown of fibrotic tissue

c. In the fibrotic phase of ARDS, diffuse scarring and fibrosis of the lungs occur, resulting in decreased surface area for gas exchange and continued hypoxemia caused by diffusion limitation. Although edema is resolved, lung compliance is decreased because of interstitial fibrosis. Long-term mechanical ventilation is required. The patient has a poor prognosis for survival.

When teaching the patient about what was happening when experiencing an intrapulmonary shunt, which explanation is accurate? a. This occurs when an obstruction impairs the flow of blood to the ventilated areas of the lung. b. This occurs when blood passes through an anatomic channel in the heart and bypasses the lungs. c. This occurs when blood flows through the capillaries in the lungs without participating in gas exchange. d. Gas exchange across the alveolar capillary interface is compromised by thickened or damaged alveolar membranes.

c. Intrapulmonary shunt occurs when blood flows through the capillaries in the lungs without participating in gas exchange (e.g., acute respiratory distress syndrome [ARDS], pneumonia). Obstruction impairs the flow of blood to the ventilated areas of the lung in a V/Q mismatch ratio greater than 1 (e.g., pulmonary embolus). Blood passes through an anatomic channel in the heart and bypasses the lungs with anatomic shunt (e.g., ventricular septal defect). Gas exchange across the alveolar capillary interface is compromised by thickened or damaged alveolar membranes in diffusion limitation (e.g., pulmonary fibrosis, ARDS).

The nurse notes that a patient's endotracheal tube (ET), which was at the 22-cm mark, is now at the 25-cm mark, and the patient is anxious and restless. Which action should the nurse take next? a. Check the O2 saturation. b. Offer reassurance to the patient. c. Listen to the patient's breath sounds. d. Notify the patient's health care provider.

c. Listen to the patient's breath sounds. The nurse should first determine whether the ET tube has been displaced into the right mainstem bronchus by listening for unilateral breath sounds. If so, assistance will be needed to reposition the tube immediately. The other actions are also appropriate, but detection and correction of tube malposition are the most critical actions.

After change-of-shift report on a ventilator weaning unit, which patient should the nurse assess first? a. Patient who failed a spontaneous breathing trial and has been placed in a rest mode on the ventilator b. Patient who is intubated and has continuous partial pressure end-tidal CO2 (PETCO2) monitoring c. Patient who was successfully weaned and extubated 4 hours ago and has no urine output for the last 6 hours d. Patient with a central venous O2 saturation (ScvO2) of 69% while on bilevel positive airway pressure (BiPAP)

c. Patient who was successfully weaned and extubated 4 hours ago and has no urine output for the last 6 hours The decreased urine output may indicate acute kidney injury or that the patient's cardiac output and perfusion of vital organs have decreased. Any of these causes would require rapid action. The data about the other patients indicate that their conditions are stable and do not require immediate assessment or changes in their care. Continuous PETCO2 monitoring is frequently used when patients are intubated. The rest mode should be used to allow patient recovery after a failed SBT, and an ScvO2 of 69% is within normal limits.

A comatose patient with a possible cervical spine injury is intubated with a nasal endotracheal tube. The nurse recognizes that in comparison with an oral endotracheal tube, a nasal tube a. requires the placement of a bite block. b. is more likely to cause laryngeal trauma. c. requires greater respiratory effort in breathing. d. provides for easier suctioning and secretion removal.

c. Rationale: A nasal endotracheal (ET) tube is longer and smaller in diameter than an oral ET tube, creating more airway resistance and increasing the work of breathing. Suctioning and secretion removal are also more difficult with nasal ET tubes, and they are more subject to kinking than are oral tubes. Oral tubes require a bite block to stop the patient from biting the tube and may cause more laryngeal damage because of their larger size.

To prevent arterial trauma during the use of the IABP, the nurse should a. reposition the patient every 2 hours. b. check the site for bleeding every hour. c. prevent hip flexion of the cannulated leg. d. cover the insertion site with an occlusive dressing.

c. Rationale: Because the IABP is inserted into the femoral artery and advanced to the descending thoracic aorta, compromised distal extremity circulation is common and requires that the cannulated extremity be extended at all times. Repositioning the patient is limited to side-lying or supine positions with the head of the bed elevated no more than 30 to 45 degrees. Assessment for bleeding is important because the IABP may cause platelet destruction, and occlusive dressings are used to prevent site infection.

In preparing the patient for insertion of a pulmonary artery catheter, the nurse a. obtains informed consent from the patient. b. places the patient in high Fowler's position. c. ensures that the patient has continuous ECG monitoring. d. performs an Allen test to confirm adequate ulnar artery perfusion.

c. Rationale: During insertion of a pulmonary artery catheter, it is necessary to monitor the ECG continuously because of the risk for dysrhythmias, particularly when the catheter reaches the right ventricle. It is the health care provider's responsibility to obtain informed consent regarding the catheter insertion. During the catheter insertion, the patient is placed supine with the head of the bed flat. An Allen test to confirm adequate ulnar artery perfusion is performed before insertion of an arterial catheter in the radial artery for arterial pressure monitoring.

A patient receiving mechanical ventilation is very anxious and agitated, and neuromuscular blocking agents are used to promote ventilation. The nurse recognizes that a. the patient will be too sedated to be aware of the details of care. b. caregivers should be encouraged to provide stimulation and diversion. c. the patient should always be addressed and explanations of care given. d. communication will not be possible with the use of neuromuscular blocking agents.

c. Rationale: Neuromuscular blocking agents produce a paralysis that facilitates ventilation, but they do not sedate the patient. It is important for the nurse to remember that the patient can hear, see, think, and feel and should be addressed and given explanations accordingly. Communication with the patient is possible, especially from the nurse, but visitors for an anxious and agitated patient should provide a calming, restful effect on the patient.

Although his oxygen saturation is above 92%, an orally intubated, mechanically ventilated patient is restless and very anxious. What intervention should be used first to decrease the risk of accidental extubation? a. Obtain an order and apply soft wrist restraints. b. Remind the patient that he needs the tube inserted to breathe. c. Administer sedatives and have a caregiver stay with the patient. d. Move the patient to an area close to the nurses' station for closer observation.

c. Rationale: Sedation may be appropriate as well as having someone the patient knows at the bedside talking to him; reassuring him may decrease his anxiety and calm him. The other methods may need to be used. Restraints will need ongoing and frequent assessment of need. Reminding the patient may help, but it may not be enough to prevent the patient from pulling the tube if the patient becomes extremely anxious.

The nurse suctions the patient's ET tube when the patient a. has peripheral crackles in all lobes. b. has not been suctioned for 2 hours. c. has coarse rhonchi over central airways. d. needs stimulation to cough and deep-breathe.

c. Rationale: Suctioning an ET tube is performed when adventitious sounds over the trachea or bronchi confirm the presence of secretions that can be removed by suctioning. Visible secretions in the ET tube, respiratory distress, suspected aspiration, increase in peak airway pressures, and changes in oxygen status are other indications. Peripheral crackles are not an indication for suctioning, and suctioning as a means of inducing a cough is not recommended because of the complications associated with suctioning.

The critical care nurse includes caregivers of the patient in the ICU as part of the health care team primarily because a. the costs of critical care will affect the entire family. b. caregivers are responsible for making health care decisions for the patient. c. the extent of the caregivers' involvement affects the patient's clinical course. d. caregivers who are ignored are more likely to question the patient's quality of care.

c. Rationale: The caregivers of the critically ill patient are very important in the recovery and well-being of the patient, and the extent to which the family is involved and supported affects the patient's clinical course. Although the cost of planning and providing critical care is a concern to caregivers, it is not the major reason caregivers are included in the patient's care. Caregivers may be responsible for making decisions about the patient's care only when the patient is unable to make personal decisions. Most caregivers have questions regarding the patient's quality of care because of anxiety and lack of information about the patient's condition.

A patient has an oral endotracheal (ET) tube inserted to relieve an upper airway obstruction and to facilitate secretion removal. The first responsibility of the nurse immediately following placement of the tube is to a. suction the tube to remove secretions. b. secure the tube to the face with adhesive tape. c. place an end tidal CO2 detector on the ET tube. d. assess for bilateral breath sounds and symmetric chest movement.

c. Rationale: The first action of the nurse is to use an end-tidal CO2 detector. If no CO2 is detected, the tube is in the esophagus. The second action by the nurse following ET intubation is to auscultate the chest to confirm bilateral breath sounds and observe to confirm bilateral chest expansion. If this evidence is present, the tube is secured and connected to an O2 source. Then the placement is confirmed immediately with x-ray, and the tube is marked where it exits the mouth. Then the patient should be suctioned as needed.

The nurse uses the minimal occluding volume to inflate the cuff on an endotracheal tube to minimize the incidence of a. infection. b. hypoxemia. c. tracheal necrosis. d. accidental extubation.

c. Rationale: The minimal occluding volume (MOV) involves adding air to the ET tube cuff until no leak is heard at peak inspiratory pressure but ensures that minimal pressure is applied to the tracheal wall to prevent pressure necrosis of the trachea. The minimal occluding volume should apply between 20 to 25 mm Hg of pressure on the trachea to prevent injury. The cuff does not secure the tube in place but rather prevents escape of ventilating gases through the upper airway.

When explaining respiratory failure to the patient's family, what should the nurse use as an accurate description? a. The absence of ventilation b. Any episode in which part of the airway is obstructed c. Inadequate gas exchange to meet the metabolic needs of the body d. An episode of acute hypoxemia caused by a pulmonary dysfunction

c. Respiratory failure results when the transfer of oxygen or carbon dioxide function of the respiratory system is impaired and, although the definition is determined by PaO2 and PaCO2 levels, the major factor in respiratory failure is inadequate gas exchange to meet tissue oxygen (O2 ) needs. Absence of ventilation is respiratory arrest and partial airway obstruction may not necessarily cause respiratory failure. Acute hypoxemia may be caused by factors other than pulmonary dysfunction

Which assessment finding obtained by the nurse when caring for a patient receiving mechanical ventilation indicates the need for suctioning? a. The patient was last suctioned 6 hours ago. b. The patient's oxygen saturation drops to 93%. c. The patient's respiratory rate is 32 breaths/min. d. The patient has occasional audible expiratory wheezes.

c. The patient's respiratory rate is 32 breaths/min. The increase in respiratory rate indicates that the patient may have decreased airway clearance and requires suctioning. Suctioning is done when patient assessment data indicate that it is needed and not on a scheduled basis. Occasional expiratory wheezes do not indicate poor airway clearance, and suctioning the patient may induce bronchospasm and increase wheezing. An O2 saturation of 93% is acceptable and does not suggest that immediate suctioning is needed.

When the V/Q lung scan result returns with a mismatch ratio that is greater than 1, which condition should be suspected? a. Pain c. Pulmonary embolus b. Atelectasis d. Ventricular septal defect

c. There will be more ventilation than perfusion (V/Q ratio greater than 1) with a pulmonary embolus. Pain and atelectasis will cause a V/Q ratio less than 1. A ventricular septal defect causes an anatomic shunt as the blood bypasses the lungs.

15. A patient with acute respiratory distress syndrome (ARDS) who is intubated and receiving mechanical ventilation develops a right pneumothorax. Which action will the nurse anticipate taking next? * a. Increase the tidal volume and respiratory rate. b. Increase the fraction of inspired oxygen (FIO2). c. Perform endotracheal suctioning more frequently. d. Lower the positive end-expiratory pressure (PEEP).

d

21. Which nursing interventions included in the care of a mechanically ventilated patient with acute respiratory failure can the registered nurse (RN) delegate to an experienced licensed practical/vocational nurse (LPN/LVN) working in the intensive care unit? * a. Assess breath sounds every hour. b. Monitor central venous pressures. c. Place patient in the prone position. d. Insert an indwelling urinary catheter.

d

Physiologic response to all types of shock include all of the following except: A) activation of the inflammatory response B) hypermetabolism C) hypoperfusion of tissues D)increased intravascular volume

d) increased intravascular volume

Which arterial blood gas (ABG) results would most likely indicate acute respiratory failure in a patient with chronic lung disease? a. PaO2 52 mm Hg, PaCO2 56 mm Hg, pH 7.4 c. PaO2 48 mm Hg, PaCO2 54 mm Hg, pH 7.38 b. PaO2 46 mm Hg, PaCO2 52 mm Hg, pH 7.36 d. PaO2 50 mm Hg, PaCO2 54 mm Hg, pH 7.28

d. In a patient with normal lung function, respiratory failure is commonly defined as a PaO2 ≤60 mm Hg or a PaCO2 >45 mm Hg or both. However, because the patient with chronic pulmonary disease normally maintains low PaO2 and high PaCO2 , acute respiratory failure in these patients can be defined as an acute decrease in PaO2 or an increase in PaCO2 from the patient's baseline parameters, accompanied by an acidic pH. The pH of 7.28 reflects an acidemia and a loss of compensation in the patient with chronic lung disease.

The nurse responds to a ventilator alarm and finds the patient lying in bed gasping and holding the endotracheal tube (ET) in her hand. Which action should the nurse take next? a. Activate the rapid response team. b. Provide reassurance to the patient. c. Call the health care provider to reinsert the tube. d. Manually ventilate the patient with 100% oxygen.

d. Manually ventilate the patient with 100% oxygen. The nurse should ensure maximal patient oxygenation by manually ventilating with a bag-valve-mask system. Offering reassurance to the patient, notifying the health care provider about the need to reinsert the tube, and activating the rapid response team are also appropriate after the nurse has stabilized the patient's oxygenation.

In caring for a patient in acute respiratory failure, the nurse recognizes that noninvasive positive pressure ventilation (NIPPV) may be indicated for which patient? a. Is comatose and has high oxygen requirements b. Has copious secretions that require frequent suctioning c. Responds to hourly bronchodilator nebulization treatments d. Is alert and cooperative but has increasing respiratory exhaustion

d. Noninvasive positive pressure ventilation (NIPPV) involves the application of a face mask and delivery of a volume of air under inspiratory pressure. Because the device is worn externally, the patient must be able to cooperate in its use and frequent access to the airway for suctioning or inhaled medications must not be necessary. It is not indicated when high levels of oxygen are needed or respirations are absent.

During hemodynamic monitoring, the nurse finds that the patient has a decreased CO with unchanged pulmonary artery wedge pressure (PAWP), HR, and SVR. The nurse identifies that the patient has a decrease in a. SV. b. preload. c. afterload. d. contractility.

d. Rationale: Cardiac output (CO) is dependent on heart rate and stroke volume, and stroke volume is determined by preload, afterload, and contractility. If CO is decreased and heart rate is unchanged, stroke volume is the variable factor. If the preload determined by pulmonary artery wedge pressure (PAWP) and the afterload determined by SVR are unchanged, the factor that is changed is the contractility of the myocardium.

The nurse recognizes that a factor commonly responsible for sodium and fluid retention in the patient on mechanical ventilation is a. increased ADH release. b. increased release of atrial natriuretic factor. c. increased insensible water loss via the airway. d. decreased renal perfusion with release of renin.

d. Rationale: Decreased CO associated with positive-pressure ventilation and positive end- expiratory pressure (PEEP) results in decreased renal perfusion, release of renin, and increased aldosterone secretion, which causes sodium and water retention. ADH may be released because of stress, but ADH is responsible only for water retention, and increased intrathoracic pressure decreases, not increases, the release of atrial natriuretic factor, causing sodium retention. There is decreased, not increased, insensible water loss via the airway during mechanical ventilation.

While suctioning the ET tube of a spontaneously breathing patient, the nurse notes that the patient develops bradycardia with premature ventricular contractions. The nurse should a. stop the suctioning and assess the patient for spontaneous respirations. b. attempt to resuction the patient with reduced suction pressure and pass time. c. stop the suctioning and ventilate the patient with slow, small-volume breaths using a bag-valve-mask (BVM) device. d. stop suctioning and ventilate the patient with a BVM device with 100% oxygen until the HR returns to baseline

d. Rationale: If serious dysrhythmias occur during suctioning, the suctioning should be stopped, and the patient should be slowly ventilated via BVM with 100% oxygen until the dysrhythmia subsides. Patients with bradycardia should not be suctioned excessively. Ventilation of the patient with slow, small-volume breaths using the BVM is performed when severe coughing results from suctioning.

A primary difference in the skills of a certified critical care nurse compared with nurses certified in medical-surgical nursing is an ability to a. diagnose and treat life-threatening diseases. b. detect and manage early complications of health problems. c. provide intensive psychologic support to the patient and family. d. use advanced technology to assess and maintain physiologic function.

d. Rationale: One of the primary characteristics of critical care nurses that is different from those of generalist medical-surgical nurses is the use of advanced technology to measure physiologic parameters accurately to manage life-threatening complications. All nursing addresses human responses to health problems and requires knowledge of physiology, pathophysiology, pharmacology, and psychologic support to the patient and family. Diagnosis and treatment of life-threatening diseases are roles of medicine.

The use of the intraaortic balloon pump (IABP) would be indicated for the patient with a. an insufficient aortic valve. b. a dissecting thoracic aortic aneurysm. c. generalized peripheral vascular disease. d. acute myocardial infarction with heart failure.

d. Rationale: The counterpulsation of the intraaortic balloon pump (IABP) increases diastolic arterial pressure, forcing blood back into the coronary arteries and main branches of the aortic arch, increasing coronary artery perfusion pressure and blood flow to the myocardium. The balloon pump also causes a drop in aortic pressure just before systole, decreasing afterload and myocardial oxygen consumption. These effects make the IABP valuable in treating unstable angina, acute myocardial infarction with heart failure, cardiogenic shock, and a variety of surgical heart situations. Its use is contraindicated in incompetent aortic valves, dissecting aortic aneurysms, and generalized peripheral vascular disease.

A patient who is hemodynamically stable has an order to wean the IABP. The nurse should a. decrease the augmentation pressure to zero. b. stop the machine since hemodynamic parameters are satisfactory. c. stop the infusion flow through the catheter when weaning is initiated. d. change the pumping ratio from 1:1 to 1:2 or 1:3 until the balloon is removed.

d. Rationale: Weaning from the IABP involves reducing the pumping to every second or third heartbeat until the IABP catheter is removed. The pumping and infusion flow are continued to reduce the risk for thrombus formation around the catheter until it is removed.

The nurse observes a PAWP waveform on the monitor when the balloon of the patient's pulmonary artery catheter is deflated. The nurse recognizes that a. the patient is at risk for embolism because of occlusion of the catheter with a thrombus. b. the patient is developing pulmonary edema that has increased the pulmonary artery pressure. c. the patient is at risk for an air embolus because the injected air cannot be withdrawn into the syringe. d. the catheter must be immediately repositioned to prevent pulmonary infarction or pulmonary artery rupture.

d. Rationale: When a pulmonary artery pressure tracing indicates a wedged waveform when the balloon is deflated, this indicates that the catheter has advanced and has become spontaneously wedged. If the catheter is not repositioned immediately, a pulmonary infarction or a rupture of a pulmonary artery may occur. If the catheter is becoming occluded, the pressure tracing becomes blunted, and pulmonary edema and increased pulmonary congestion increase the pulmonary artery waveform. Balloon leaks found when injected air does not flow back into the syringe do not alter waveforms.

The nurse assesses that a patient in respiratory distress is developing respiratory fatigue and the risk of respiratory arrest when the patient displays which behavior? a. Cannot breathe unless he is sitting upright c. Has an increased inspiratory-expiratory (I/E) ratio b. Uses the abdominal muscles during expiration d. Has a change in respiratory rate from rapid to slow

d. The increase in respiratory rate required to blow off accumulated CO2 predisposes to respiratory muscle fatigue. The slowing of a rapid rate in a patient in acute distress indicates tiring and the possibility of respiratory arrest unless ventilatory assistance is provided. A decreased inspiratory-expiratory (I/E) ratio, orthopnea, and accessory muscle use are common findings in respiratory distress but do not necessarily signal respiratory fatigue or arrest.

Prone positioning is considered for a patient with ARDS who has not responded to other measures to increase PaO2 . The nurse knows that this strategy will a. increase the mobilization of pulmonary secretions. b. decrease the workload of the diaphragm and intercostal muscles. c. promote opening of atelectatic alveoli in the upper portion of the lung. d. promote perfusion of nonatelectatic alveoli in the anterior portion of the lung.

d. When a patient with ARDS is supine, alveoli in the posterior areas of the lung are dependent and fluid-filled and the heart and mediastinal contents place more pressure on the lungs, predisposing to atelectasis. If the patient is turned prone, air-filled nonatelectatic alveoli in the anterior portion of the lung receive more blood and perfusion may be better matched to ventilation, causing less V/Q mismatch. Lateral rotation therapy is used to stimulate postural drainage and help mobilize pulmonary secretions.

The nurse is caring for a patient with a subarachnoid hemorrhage who is intubated and placed on a mechanical ventilator with 10 cm H2O of peak end-expiratory pressure (PEEP). When monitoring the patient, the nurse will need to notify the health care provider immediately if the patient develops a. O2 saturation of 93%. b. green nasogastric tube drainage. c. respirations of 20 breaths/minute. d. increased jugular venous distention.

d. increased jugular venous distention. Increases in jugular venous distention in a patient with a subarachnoid hemorrhage may indicate an increase in intracranial pressure (ICP) and that the PEEP setting is too high for this patient. A respiratory rate of 20, O2 saturation of 93%, and green nasogastric tube drainage are within normal limits.

To maintain proper cuff pressure of an endotracheal tube (ET) when the patient is on mechanical ventilation, the nurse should a. inflate the cuff with a minimum of 10 mL of air. b. inflate the cuff until the pilot balloon is firm on palpation. c. inject air into the cuff until a manometer shows 15 mm Hg pressure. d. inject air into the cuff until a slight leak is heard only at peak inflation.

d. inject air into the cuff until a slight leak is heard only at peak inflation. The minimal occluding volume technique involves injecting air into the cuff until an air leak is present only at peak inflation. The volume to inflate the cuff varies with the ET and the patient's size. Cuff pressure should be maintained at 20 to 25 mm Hg. An accurate assessment of cuff pressure cannot be obtained by palpating the pilot balloon.

The stage of shock characterized by a normal blood pressure is the?

irreversible stage

The hepatic effects of progressive shock would not include?

lowered bilirubin levels

The sequence of organ failure in multiple organ dysfunction syndrome usually begins in the?

lungs

A patient arrives in the emergency department with complaints of chest pain radiating to the jaw. What medication does the nurse anticipate administering to reduce pain and anxiety as well as reducing oxygen consumption?

morphine

The nurse is caring for a client in the irreversible stage of shock. The nurse is explaining to the client's family the poor prognosis. Which would the nurse be most accurate to explain as the rationale for imminent death?

multiple organ failure

Coronary cardiogenic shock is seen primarily in pts with?

myocardial infarction

A client with a history of depression is brought to the ED after overdosing on Valium. This client is at risk for developing which type of distributive shock?

neurogenic shock pp. 304-305.

A client is experiencing vomiting and diarrhea for 2 days. Blood pressure is 88/56, pulse rate is 122 beats/minute, and respirations are 28 breaths/minute. The nurse starts intravenous fluids. Which of the following prescribed prn mediciations would the nurse administer next?

ondansetron (Zofran) pg. 297

The nurse is caring for a client in septic shock. The nurse knows to closely monitor the client. What finding would the nurse observe when the client's condition is in its initial stages?

rapid, bounding pulse pp. 301-303.

The nurse assess a patient in compensatory shock whose longs have been decompensated. The nurse would not expect to find the following symptoms?

resp fewer than 15 bpm

Vasoactive agents are effective in treating shock if fluid administration fails because of their ability to?

reverse the cause of dehydration

The nurse anticipates that a patient who is immunosuppressed is at the greatest risk for developing which of the following types of shock?

sepetic


Related study sets

Short Story #1 Who is this person?

View Set

Maternity Exam #3: Contraception, Infertility and Abortion

View Set

APUSH Test - 1960-1990 - 5.16.16

View Set

Benchmark Fractions, Decimals, and Percents Models

View Set

Introduction and the Importance of Listening in Our Lives Week 4

View Set